Sunteți pe pagina 1din 89

BAT

COMENTARIOS DE TEXTO
Lengua castellana y literatura 1
COMENTARIOS DE TEXTO Poema de Mio Cid
BAT

POEMA DE MIO CID: CANTAR DEL DESTIERRO


A. La obra comienza presentando al Cid que, habiendo sido des- 1 Contesta estas preguntas:
terrado por el rey, abandona Castilla ultrajado y con su honor a ¿Qué dos aspectos de la
perdido. Parte de su tierra natal con gran pena y los castellanos
personalidad del Cid se
no pueden prestarle ayuda porque el rey lo ha prohibido.
muestran en los dos
1 De los sos ojos tan fuerte mientre lorando fragmentos (A y B)?
tornava la cabeça y estava los catando.1
b ¿Qué piensan los castellanos
Vio puertas abiertas e uços2 sin cañados,
del Cid? ¿Qué quieren
alcandaras3 vazias sin pielles e sin mantos
expresar en el último verso
5 e sin falcones e sin adtores mudados.
Sospiro mio Çid ca mucho avie grandes cuidados.
del primer fragmento?
Ffablo mio Çid bien e tan mesurado: c ¿En qué momentos usa el
“¡Grado a ti, señor, padre que estas en alto! autor el estilo directo? ¿Con
¡Esto me an buelto mios enemigos malos!” qué intención?
10 Alli pienssan de aguijar, alli sueltan las riendas
a la exida de Bivar ovieron la corneja diestra 2 Analiza las distintas formas
y entrando a Burgos ovieron la siniestra.
verbales usadas en los dos
Meçio mio Çid los ombros y engrameo4 la tiesta:
fragmentos y relaciónalas
“¡Albriçia, Albar Ffañez, ca echados somos de tierra!”
con el contenido.
15 Mio Çid Ruy Diaz por Burgos entrava,
en su compaña .lx. pendones levava.
Exien lo ver mugieres e varones,
burgeses e burgesas por las finiestras son,
plorando de los ojos tanto avien el dolor.
20 De las sus bocas todos dizian una razon:
“¡Dios, que buen vassalo! ¡Si oviesse buen señor!”

B. El Cid y sus hombres luchan contra los moros en Aragón. El


héroe muestra su valentía y compañerismo en una típica escena
épica.
1 A Minaya Albar Fañez mataron le el cavallo,
bien lo acorren mesnadas de christianos;
la lança a quebrada, al espada metio mano,
mager5 de pie buenos colpes va dando
5 Violo mio Çid Ruy Diaz el Castelano:
acostos a un aguazil6 que tenie buen cavallo
diol tal espadada con el so diestro braço
cortol por la çintura el medio echo en campo.
A Minaya Albar Fañez ival dar el cavallo:
10 “¡Cavalgad, Minaya, vos sodes el mio diestro braço!”

1 catando Mirando. 3 alcándaras Perchas, ganchos. 2 uços Puertas.


4 engrameo Sacudió. 5 mager Aunque. 6 aguazil Juez moro;
caudillo en general.

*La -e paragógica Según Menéndez Pidal, era normal añadir en la época en la que
fue concebido el poema una -e en palabras rima que terminan en sílaba acentuada.
Sólo el hecho de que el manuscrito conservado sea del siglo XIV explica que esa -e
no aparezca y se den rimas imperfectas. Hoy en día, no todos los críticos coinciden
con esta opinión de Menéndez Pidal.

Lengua castellana y literatura 1 2


COMENTARIOS DE TEXTO Poema de Mio Cid
BAT

POEMA DE MIO CID: CANTAR DEL DESTIERRO


C. El Cid parte al destierro y deja a su familia en el monasterio 1 Lee el primer fragmento
de Cardeña. (C) y responde:
1 Ant'el Campeador doña Ximena finco los inojos amos, a ¿Cómo es doña Jimena?
lorava de los ojos, quisol besar las manos:
b ¿Cómo se muestra el héroe
“¡Merçed, Campeador, en ora buena fuestes nado!
ante su familia? ¿Es un
Por malos mestureros1 de tierra sodes echado.
comportamiento típico
5 ¡Merçed, ya Çid, barba tan complida! de un héroe épico?
Fem ante vos yo e vuestras fijas
–iffantes son e de dias chicas
con aquestas mis dueñas de quien so yo servida
2 Lee el segundo fragmento
Yo lo veo que estades vos en ida (D) y responde:
10 e nos de vos partir nos hemos en vida: a ¿Interviene el narrador?
¡Da(n)d nos consejo por amor de Santa Maria!” ¿Cuál es su postura?
Enclino las manos [el de] la barba velida,
a las sus fijas en braço' las prendia,
legolas al coraçon ca mucho las queria. 3 Demuestra, con ejemplos
15 Lora de los ojos, tan fuerte mientre sospira: tomados de los textos, el
“¡Ya doña Ximena la mi mugier tan complida, estilo binario y formulario
commo a la mi alma yo tanto vos queria del poema.
Ya lo vedes que partir nos emos en vida,
yo ire e vos fincaredes remanida2.
20 ¡Plega a Dios e a Santa Maria
que aun con mis manos case estas mis fijas,
o que de ventura e algunos dias vida
e vos, mugier ondrada, de mi seades servida!”

POEMA DE MIO CID: CANTAR DE LA AFRENTA


DE CORPES
D. Al fin todo acaba bien para el Cid, que recupera su honor
militar y limpia y mejora su honor familiar después de la afrenta
de Corpes y la subsiguiente reparación legal.
1 “¡Grado al rey del çielo, mis fijas vengadas son!
¡Agora las ayan quitas heredades de Carrion!
Sin verguença las casare o a qui pese o a qui non.”
Andidieron en pleitos3 los de Navarra e de Aragon,
5 ovieron su ajunta con Alfonsso el de Leon;
fizieron sus casamientos con don Elvira e con doña Sol.
Los primeros fueron grandes mas aquestos son mijores;
a mayor ondra las casa que lo que primero fue:
10 ¡ved qual ondra creçe al que en buen ora naçio
quando señoras son sus fijas de Navarra e de Aragon!
Oy los reyes d'España sos parientes son;

1 mestureros Intrigantes, cizañeros.


2 remanir Quedar, estar.
3 pleitos Negociaciones.

Lengua castellana y literatura 1 3


COMENTARIOS DE TEXTO El Romancero
BAT

EL ROMANCERO
En la segunda mitad del siglo XIV, ROMANCE DEL JURAMENTO QUE TOMÓ
la épica hispánica estaba en un EL CID AL REY DON ALFONSO
periodo de evidente decadencia.
Según Menéndez Pidal, como 1 En Santa Águeda de Burgos,
consecuencia de esa fragmenta- do juran los hijosdalgo,
ción de los poemas épicos (algu- le toman jura a Alfonso
nos pasajes eran recordados y por la muerte de su hermano;
admirados por el público y otros 5 tomábasela el buen Cid,

completamente olvidados) surgen ese buen Cid castellano,


los romances, que recogen y des- sobre un cerrojo de hierro
arrollan aquellos fragmentos de y una ballesta de palo
cantares épicos que la gente que- y con unos evangelios
ría escuchar. En definitiva, como 10 y un crucifijo en la mano.
dice Pidal, “todas las gestas se Las palabras son tan fuertes
hicieron romances; la epopeya se que al buen rey ponen espanto.
hizo romancero”.
—Villanos te maten, Alonso,
El romance es una composición villanos, que no hidalgos,
poética no estrófica de versos 15 de las Asturias de Oviedo,
octosilábicos y rima asonantada que no sean castellanos;
en los versos pares. Este romance
mátente con aguijadas,1
en concreto narra el episodio de
no con lanzas ni con dardos;
la jura de Santa Gadea, que expli-
con cuchillos cachicuernos,2
caría el destierro del Cid.
20 no con puñales dorados;
abarcas traigan calzadas,
que no zapatos con lazo;
capas traigan aguaderas,
no de contray ni frisado;3
25 con camisones de estopa,
no de holanda ni labrados;
caballeros vengan en burras,
que no en mulas ni en caballos;
frenos traigan de cordel,
Imagen que reproduce la Jura de Santa 30 que no cueros fogueados.
Gadea, juramento que exigió el Cid al Mátente por las aradas,
rey de Castilla Alfonso VI de no haber
participado en la muerte de su
que no en villas ni en poblado,
hermano Sancho II en 1072. sáquente el corazón

Lengua castellana y literatura 1 4


COMENTARIOS DE TEXTO El Romancero
BAT

1 Responde a las siguientes


por el siniestro costado, preguntas después de leer
35 si no dijeres la verdad el romance:
de lo que te fuere preguntado, a ¿Qué narra el romance?
si fuiste, o consentiste ¿Es un episodio tomado del
en la muerte de tu hermano. poema épico?
Las juras eran tan fuertes b ¿Qué actitud adopta el Cid?
40 que el rey no las ha otorgado ¿Cómo reacciona el rey?
Allí habló un caballero
c ¿Qué sucesos históricos
que del rey es más privado:
podrían justificar la postura de
—Haced la jura, buen rey,
Rodrigo? ¿Te parece histórico
no tengáis de eso cuidado,
el episodio de la jura?
45 que nunca fue rey traidor,
ni papa descomulgado. d ¿Qué diferencias hay entre
Jurado había el rey el Cid del romance y el Cid
que en tal nunca se ha hallado; del poema épico?
pero allí hablara el rey e ¿Por quién toma partido el
50 malamente y enojado: narrador del romance?
—Muy mal me conjuras, Cid, ¿Es sutil su postura?
Cid, muy mal me has conjurado,
mas hoy me tomas la jura,
mañana me besarás la mano. 2 Analiza la métrica
del romance e intenta
55 —Por besar mano de rey
demostrar a partir de tu
no me tengo por honrado,
análisis la teoría que
porque la besó mi padre
relaciona los romances
me tengo por afrentado.
con los cantares de gesta.
—Vete de mis tierras, Cid,
60 mal caballero probado,
y no vengas más a ellas 3 Siendo el texto que has leído
dende este día en un año. un romance épico-histórico,
—Pláceme, dijo el buen Cid, busca en él los siguientes
pláceme, dijo de grado, rasgos estilísticos y explica
65 por ser la primera cosa qué aportan al texto:
que mandas en tu reinado. a Estilo directo y estilo indirecto.
Tú me destierras por uno, b Estilo formular: epítetos
yo me destierro por cuatro. épicos, frases binarias, frases
Ya se parte el buen Cid, apositivas.
70 sin al rey besar la mano, c Enumeraciones, antítesis
con trescientos caballeros, y anáforas.
todos eran hijosdalgo;
todos son hombres mancebos, d Variedad en el uso de las
ninguno no había cano; formas verbales.
75 todos llevan lanza en puño e La adjetivación.
y el hierro acicalado,
y llevan sendas adargas
con borlas de colorado.
1 aguijadas Herramientas de labrador.
Mas no le faltó al buen Cid 2 cachicuernos Con cacha de cuerno.
80 adonde asentar su campo. 3 contray y frisado Telas de lana y seda.

Lengua castellana y literatura 1 5


COMENTARIOS DE TEXTO El Conde Lucanor
BAT

EL CONDE LUCANOR: EJEMPLO XI


De lo que aconteció a un deán de Santiago con don Illán el gran maestro
que moraba en Toledo.
1 Otro día hablaba el conde Lucanor con Patronio, su consejero, y con-
tábale sus asuntos de esta guisa:
—Patronio, un hombre vino a rogarme que le ayudase en un hecho
en que había menester mi ayuda, y prometiome que haría por mí todas
5 las cosas que fuesen mi pro y mi honra. Y yo comencele a ayudar cuan-
to pude en aquel hecho. Y antes de que el negocio fuese acabado, cre-
yendo él que ya el negocio suyo estaba resuelto, acaeció una cosa en
que cumplía que él la hiciese por mí, y roguele que la hiciese y él púso-
me excusa. Y después acaeció otra cosa que él hubiese podido hacer por
10 mí, y púsome otrosí excusa: y esto me hizo en todo lo que yo le rogué
que hiciese por mí. Y aquel hecho por el que él me rogó, no está aún
resuelto, ni se resolverá si yo no quiero. Y por la confianza que yo he
en vos y en el vuestro entendimiento, ruégoos que me aconsejéis lo
que haga en esto.
15 —Señor conde —dijo Patronio—, para que vos hagáis en esto lo que
vos debéis, mucho querría que supieseis lo que aconteció a un deán de
Santiago con don Illán, el gran maestro que moraba en Toledo.
Y el conde le preguntó cómo había sido aquello.
—Señor conde —dijo Patronio—, en Santiago había un deán que
20 había muy gran talante de saber el arte de la nigromancia, y oyó decir
que don Illán de Toledo sabía de ello más que ninguno que viviese en
aquella sazón. Y por ello vínose para Toledo para aprender aquella
ciencia. Y el día que llegó a Toledo, enderezó luego a casa de don Illán
y hallolo que estaba leyendo en una cámara muy apartada; y luego que
25 llegó a él, recibiolo muy bien y díjole que no quería que le dijese nin-
guna cosa de aquello por lo que venía hasta que hubiesen comido.
Y cuidó muy bien de él e hízole dar muy buena posada, y todo lo que
hubo menester y diole a entender que le placía mucho con su venida.
Y después que hubieron comido, apartose con él y contole la razón
30 por la que allí había venido, y rogole muy apremiadamente que le mos-
trase aquella ciencia, que él había muy gran talante de aprenderla.
Y don Illán díjole que él era deán y hombre de gran rango y que podría
llegar a gran estado y los hombres que gran estado tienen, desde que
todo lo suyo han resuelto a su voluntad, olvidan muy deprisa lo que otro
35 ha hecho por ellos: y él, que recelaba que desde que él hubiese apren-
dido de él aquello que el quería saber, que no le haría tanto bien como
él le prometía. Y el deán le prometió y le aseguró que de cualquier bien
que él tuviese, que nunca haría sino lo que él mandase.
Y en estas hablas estuvieron desde que hubieron yantado hasta que
40 fue hora de cena. De que su pleito fue bien asosegado entre ellos, dijo
don Illán al deán que aquella ciencia no se podía aprender sino en lugar
muy apartado y que luego, esa noche, le quería mostrar do habían de estar
hasta que hubiese aprendido aquello que él quería saber. Y tomole por la
mano y llevole a una cámara. Y, en apartándose de la otra gente, llamó
45 a una manceba de su casa y díjole que tuviese perdices para que cenasen
esa noche, mas que no las pusiese a asar hasta que él se lo mandase.

Lengua castellana y literatura 1 6


COMENTARIOS DE TEXTO El Conde Lucanor
BAT

Y desde que esto hubo dicho llamó al deán; y entraron ambos por
una escalera de piedra muy bien labrada y fueron descendiendo por ella
muy gran rato de guisa que parecía que estaban tan bajos que pasaba
50 el río Tajo sobre ellos. Y desde que estuvieron al final de la escalera,

hallaron una posada muy buena, y una cámara muy adornada que allí
había, donde estaban los libros y el estudio en que había de leer. Y
desde que se sentaron, estaban parando mientes en cuáles libros ha-
bían de comenzar. Y estando ellos en esto, entraron dos hombres por
55 la puerta y diéronle una carta que le enviaba el arzobispo, su tío, en

que le hacía saber que estaba muy doliente y que le enviaba rogar que,
si le quería ver vivo, que se fuese luego para él. Al deán le pesó mucho
de estas nuevas; lo uno por la dolencia de su tío, y lo otro porque rece-
ló que había de dejar su estudio que había comenzado. Pero puso en
60 su corazón el no dejar aquel estudio tan deprisa e hizo sus cartas de

respuesta y enviólas al arzobispo su tío. Y de allí a unos tres días lle-


garon otros hombres a pie que traían otras cartas al deán, en que le
hacían saber que el arzobispo era finado, y que estaban todos los de la
iglesia en su elección y que fiaban en que, por la merced de Dios, que
65 le elegirían a él, y por esta razón que no se apresurase a ir a la iglesia.

Porque mejor era para él que le eligiesen estando en otra parte, que no
estando en la iglesia.
Y de allí al cabo de siete o de ocho días, vinieron dos escuderos muy
bien vestidos y muy bien aparejados, y cuando llegaron a él besáron-
70 le la mano y mostráronle las cartas que decían cómo le habían elegi-

do arzobispo. Y cuando don Illán esto oyó, fue al electo y díjole cómo
agradecía mucho a Dios porque estas buenas nuevas le habían llegado
en su casa; y pues Dios tanto bien le había hecho, que le pedía como
merced que el deanato que quedaba vacante que lo diese a un hijo
75 suyo. El electo díjole que le rogaba que le quisiese permitir que aquel

deanato que lo hubiese un su hermano; mas que el haría bien de guisa


que él quedase contento, y que le rogaba que se fuese con él para
Santiago y que llevase él a aquel su hijo. Don Illán dijo que lo haría.
Y fuéronse para Santiago; y cuando allí llegaron fueron muy bien
80 recibidos y muy honrosamente. Y desde que moraron allí un tiempo,
un día llegaron al arzobispo mandaderos del papa con sus cartas en
las cuales le daba el obispado de Tolosa, y que le concedía la gracia de
que pudiese dar el arzobispado a quien quisiese. Cuando Don Illán
esto oyó, recordándole muy apremiadamente lo que con él había con-
85 venido, pidiole como merced que lo diese a su hijo; y el arzobispo le
rogó que consintiese que lo hubiese un su tío, hermano de su padre.
Y don Illán dijo que bien entendía que le hacía gran tuerto, pero que
esto que lo consentía con tal de que estuviese seguro de que se lo
enmendaría más adelante. El arzobispo le prometió de toda guisa que
90 lo haría así y rogolo que fuese con él a Tolosa.

Y desde que llegaron a Tolosa, fueron muy bien recibidos de los con-
des y de cuantos hombres buenos había en la tierra. Y desde que hubie-
ron allí morado hasta dos años, llegáronle mandaderos del papa con
sus cartas en las cuales le hacía el papa cardenal y que le concedía la
95 gracia de que diese el obispado de Tolosa a quien quisiese. Entonces

Lengua castellana y literatura 1 7


COMENTARIOS DE TEXTO El Conde Lucanor
BAT

fue a él don Illán y díjole que, pues tantas veces le había fallado en lo
que con él había acordado, que ya aquí no había lugar para ponerle
excusa ninguna, que no diese alguna de aquellas dignidades a su hijo.
Y el cardenal rogole que consintiese que hubiese aquel obispado un su
100 tío, hermano de su madre que era hombre bueno y anciano; mas que,
pues él cardenal era, que se fuese con él para la corte, que asaz había
en que hacerle bien. Y don Illán quejose de ello mucho, pero consin-
tió en lo que el cardenal quiso, y fuese con él para la corte.
Y desde que allí llegaron, fueron muy bien recibidos por los carde-
105 nales y por cuantos allí estaban en la corte, y moraron allí muy gran

tiempo. Y don Illán apremiando cada día al cardenal que le hiciese


alguna gracia a su hijo, y él poníale excusas.
Y estando así en la corte, finó el papa; y todos los cardenales eligie-
ron a aquel cardenal por papa. Entonces fue a él don Illán y díjole que
110 ya no podía poner excusa para no cumplir lo que le había prometido.

Y el papa le dijo que no le apremiase tanto, que siempre habría lugar


para que le hiciese merced según fuese razón. Y don Illán se comenzó
a quejar mucho, recordándole cuántas cosas le había prometido y que
nunca le había cumplido ninguna, y diciéndole que aquello recelaba
115 él la primera vez que con él había hablado y pues que a aquel estado
era llegado y no le cumplía lo que le había prometido, que ya no le
quedaba lugar para esperar de él bien ninguno. De esta queja se quejó
mucho el papa y comenzole a maltraer diciéndole que, si más le apre-
miase, que le haría echar en una cárcel, que era hereje y mago, que bien
120 sabía él que no había otra vida ni otro oficio en Toledo donde él mora-
ba, sino vivir de aquel arte de la nigromancia.
Y desde que don Illán vio cuán mal galardonaba el papa lo que por él
había hecho, despidiose de él y ni siquiera le quiso dar el papa que
comiese por el camino. Entonces don Illán dijo al papa que pues otra
125 cosa no tenía para comer, que se habría de tornar a las perdices que había

mandado a asar aquella noche, y llamó a la mujer y díjole que asase las
perdices.
Cuando esto dijo don Illán, se halló el papa en Toledo, deán de
Santiago, como lo era cuando allí vino, y tan grande fue la vergüenza
130 que hubo, que no supo qué decirle. Y don Illán díjole que se fuese con
buena ventura y que asaz había probado lo que tenía en él, y que lo
tendría por muy mal empleado si comiese su parte de las perdices.
Y vos, señor conde Lucanor, pues veis que tanto hacéis por aquel
hombre que os demanda ayuda y no os da de ello mejores gracias,
135 tengo que no habéis por qué trabajar ni aventuraros mucho para lle-

varlo a ocasión en que os dé tal galardón como el deán dio a don Illán.
El conde tuvo éste por buen consejo, e hízolo así y hallose en ello
bien.
Y porque entendió don Juan que este ejemplo era muy bueno, hízo-
140 lo escribir en este libro e hizo de ello estos versos que dicen así:

A quien mucho ayudes y no te lo reconozca


menos ayuda habrás de él desde que a gran honra suba.

Lengua castellana y literatura 1 8


COMENTARIOS DE TEXTO El Conde Lucanor
BAT

1 Expón brevemente cuál es el problema del h ¿Qué efecto tiene la mención de las perdices
conde. que, al principio de la historia, don Illán había
mandado preparar?

2 Responde a las siguientes preguntas sobre el i ¿Cuánto tiempo real corresponde a la ilusión
cuento con el cual responde Patronio: temporal que la magia ha creado?

a ¿Por qué acude el deán de Santiago a don j ¿Qué función tiene el sortilegio? ¿Qué
Illán? ¿Cómo le recibe éste? conclusión extrae don Illán?

b ¿Qué temor expone don Illán ante la petición


del deán? 3 Localiza la comparación que Patronio esta-
c ¿De qué pequeño asunto doméstico se ocupa blece entre el problema del conde y el “ejem-
don Illán antes de iniciar el estudio? plo” que ha utilizado para responderle.

d ¿Cómo es el lugar donde se instalan para


iniciar su estudio? ¿Qué ambiente contribuye 4 Localiza la aparición de una nueva voz: la del
a crear? autor, aprobando la utilidad de la historia. In-
e ¿Qué interrumpe su actividad, al poco de dica cuál es el mensaje de los versos finales.
iniciarse? Indica qué cambio se produce en
la situación del deán. ¿Qué le pide don Illán?
¿Qué le responde el ex deán de Santiago?
f Indica los sucesivos cambios de espacio de la
acción, así como de la situación del ex deán.
g ¿Cómo reacciona el ex deán cuando don Illán
parece desesperar de no obtener ningún favor?

Vista de las antiguas murallas de Toledo, ciudad


escenario de la historia narrada en el texto.

Lengua castellana y literatura 1 9


COMENTARIOS DE TEXTO Libro de Buen Amor
BAT

PELEA CON DON AMOR


1 Si quisieres amar dueñas o otra qualquier muger,
muchas cosas avrás primero a deprender;
para que ella te quiera en amor acoger,
4 sabe primeramente la muger escoger.
Cata1 muger fermosa, donosa, et loçana,
que non sea muy luenga, nin otrosí enana;
si podieres, non quieras amar muger villana
8 que de amor non sabe, es como baüsana.2
Busca muger de talla, de cabeça pequeña,
cabellos amarillos, non sean de alheña,3
las çejas apartadas, luengas, altas, en peña,
12 ancheta de caderas; ésta es talla de dueña.
Ojos grandes, someros, pintados, reluzientes,
e de luengas pestañas, bien claras, paresçientes,4 1 cata Mira, busca.
las orejas pequeñas, delgadas, páral mientes, 2 baüsana Tonta, boba.
16 si ha el cuello alto: atal quieren las gentes. 3 alheña Planta cuyas hojas
molidas se utilizaban para teñir
La nariz afilada, los dientes menudillos, el pelo.
eguales, e bien blancos, poquillo apartadillos, 4 paresçientes Aparentes, que
las enzías bermejas; los dientes agudillos; destaquen.
20 los labros de la boca bermejos, angostillos.5 5 angostillos Estrechos.

Interior de la basílica de san Isidoro (León)

Lengua castellana y literatura 1 10


COMENTARIOS DE TEXTO Libro de Buen Amor
BAT

La su boca pequeña, así de buena guisa; 1 Lee el segundo texto.


la su faz sea blanca, sin pelos, clara, e lisa, Localiza y cita el primer
puna6 de aver muger, que la veas sin camisa, consejo de don Amor.
7
24 que la talla del cuerpo te dirá: “Esto aguisa”.

La muger que embïares de ti sea parienta,


que bien leal te sea, non sea su servienta; 2 ¿Qué estrofas ocupa la
non lo sepa la dueña, porque la otra non mienta: descripción de la mujer
28 non puede ser, quien mal casa, que non se arrepienta. ideal? Resume sus
Puña, en quanto puedas que la tu mensajera características.
sea bien razonada, sotil e costumera,8
sepa mentir fermoso e siga la carrera, 3 ¿Cómo debe ser la
32 ca más fierbe la olla con la su cobertera.
mensajera? Caracteriza a
Si parienta non tienes atal, toma unas viejas, las viejas que pueden servir
que andan las iglesias e saben las callejas:
como mensajeras de no
grandes cuentas al cuello, saben muchas consejas,
tener una parienta
36 con lágrimas de Moisés escantan las orejas.
adecuada.
Son grandes maestras aquestas pavïotas,9
andan por todo el mundo, por plaças e por cotas,
a Dios alçan las cuentas, querellando sus coitas,10 4 Indica algunos rasgos de
40 ¡aÿ! ¡quánto mal saben estas viejas arlotas!11 carácter de la mujer ideal.
Toma de unas viejas que se fazen erveras,12
andan de casa en casa e llámanse parteras;
con polvos e afeites e con alcoholeras,13 5 Di qué otros consejos le
44 echan la moça en ojo e çiegan bien de veras. da para seducir y conservar
E busca mensajera de unas negras pegatas a la mujer.
que usan muncho fraires, e monjas e beatas:
son mucho andariegas, meresçen las çapatas;
48 estas trotaconventos fazen muchas baratas.14 […] 6 puna Intenta, procura. El
mismo significado tiene la
En fin de las razones, fazle una pregunta: forma puña del verso 29.
si es muger alegre, de amor se repunta;15 7 Esto aguisa Haz esto.
si á sueras frías,16 si demanda quanto barrunta, 8 costumera Hábil, diestra,
52 al ome si dize sí, a tal mujer te ayunta. experta.
Atal es de servir e atal es de amar: 9 pavïotas Falsas.
es muy más plazentera que otras en doñear;17 10 querellando sus coitas
Quejándose de sus cuitas o
si tal saber podieres e la quieres cobrar, problemas.
56 faz mucho por servirla en dezir e en obrar. 11 arlotas Bribonas, pícaras.
De tus joyas fermosas cada que dar podieres; 12 erveras Vendedoras de hierbas
quando dar no quesieres o quando non tovieres, medicinales.
promete e manda mucho maguer non gelo dieres: 13 polvos, afeites y alcoholeras
Son todos productos
60 luego estará afuziada,18 fará lo que quisieres.
cosméticos.
Sírvela, non te enojes, sirviendo el amor crece; 14 baratas, Negocios, tratos,
el serviçio en el bueno nunca muere nin peresçe; trueques.
si se tarda, non se pierde, el amor nunca fallesçe: 15 se repunta Presume.
64 el grand trabajo siempre todas las cosas vençe. […] 16 si á sueras frías Probablemente
'si tiene sudores fríos'.
Requiere a menudo a la que bien quisieres, 17 doñear Pretender, cortejar.
non ayas miedo d’ella quando tienpo tovieres, 18 afuciada Obligada por pacto al
vergüença non te embargue do con ella estodieres. cumplimiento de alguna cosa.
68 perezoso non seas adó buena azina vieres.19 […] 19 adó Cuando; azina Ocasión.

Lengua castellana y literatura 1 11


COMENTARIOS DE TEXTO Libro de Buen Amor
BAT

DE CÓMO MORIÓ TROTACONVENTOS…


1 ¡Ay Muerte!, ¡muerta seas, muerta, e mal andante! 1 Fíjate en que el planto se abre
mataste a mi vieja, ¡matasses a mí ante! con una imprecación a la
Enemiga del mundo, que non as semejante, Muerte. Di qué le reprocha.
4 de tu memoria amarga non es que non se espante.
Muerte al que tú fieres, liévaslo de belmez,1
al bueno e al malo, al noble, e al refez,2 2 Contesta:
a todos los egualas, e los lievas por un prez, a ¿Cómo reaccionan los vivos
8 por papas e por reyes non das una vil nuez. ante el muerto?
Non catas señorío, debdo, nin amistad,
con todo el mundo tienes cotiana enemistad, b ¿Qué sucede con sus bienes?
non ay en ti mesura, amor, nin pïadad, ¿Y si deja mujer joven y guapa?
12 sinon dolor, tristeza, pena, e grand crüeldad. c ¿Qué versos prueban que se
Non puede foir omne de ti, nin se asconder, trata de un planto paródico?
nunca fue quien contigo podies´ bien contender, Explica por qué deben
la tu venida triste non se puede entender, interpretarse en clave
16 desque vienes, non quieres a omne atender. humorística.
Dexas el cuerpo yermo a gusanos en fuesa3
al alma que lo puebla, liévastela de priesa,
non es el omne çierto de tu carrera aviesa, 3 Observa los siguientes rasgos
20 de fablar en ti, Muerte, espanto me atraviesa. a fin de que tengas presente
Eres en tal manera del mundo aborrida, qué tratamiento había
que, por bien que lo amen al omne en la vida, recibido el tema de la muerte
en punto que tú vienes con tu mala venida hasta que aparecieron las
24 todos fuyen d’él luego como de res podrida. […] “Coplas a la muerte de su
Desque los sus parientes la su muerte varruntan, padre” de Jorge Manrique:
por lo heredar todo a menudo se ayuntan;
quando por su dolençia al físico4 preguntan, a ¿Aparece la idea del poder
28 si diz que sanará, todos gelo repuntan.5 […] igualador de la muerte?
Si dexa muger moça, rica o paresçiente, b ¿Cómo trata la Muerte a los
ante de misa dicha otros la han en miente: hombres?
o casa con más rico o moço más valiente,
32 muda el trentanario,6 del duelo poco s´ siente. c ¿Hay referencias a los aspectos
Muerte desmesurada, ¡matases a ti sola! más desagradables del hecho
¿Qué oviste conmigo? ¿Mi leal vieja, dola?7 de morir? Cítalos.
Tú m´la mataste, muerte; Jhesu Cristo conprola
36 por la su santa sangre, por ella perdonola.
¡Ay! Mi Trotaconventos, ¡mi leal verdadera!,
1 de belmez Probablemente el
muchos te siguién biva, muerta, yazes señera; significado de esta expresión sea
¿Adó te me han levado? Non sé cosa çertera: 'irremediablemente'.
40 nunca torna con nuevas quien anda esta carrera. 2 refez De poco valor, ruin.
Cierto, en paraíso estás tú assentada, 3 fuesa Fosa, hoyo para enterrar
con los mártires deves estar aconpañada: un cadáver.
4 físico Médico.
siempre en el mundo fuste por dios martirïada;
5 repuntan Echan en cara.
44 ¿quién te me rebató, vieja por mí lazrada?8
6 trentanario Número de treinta
A Dios merçed le pido que te dé la su gloria, días, continuados o interrumpidos,
que más leal trotera nunca fue en memoria; dedicados a un mismo objeto,
fazerte he un pitafio escrito con escoria:9 ordinariamente religioso.
48 pues que a ti non viere, veré tu triste estoria. 7 ¿dóla? ¿Dónde está?, ¿dónde la
tiene?.
Libro de Buen Amor, edición de J. L. Girón Alconchel.
Editorial Castalia. 8 por mí lazrada Atormentada por
mi culpa.
9 escrito con escoria Escrito de
forma defectuosa.

Lengua castellana y literatura 1 12


COMENTARIOS DE TEXTO Jorge Manrique
BAT

COPLAS
I Recuerde1 el alma dormida, IV Dexo las invocaciones
avive el seso e despierte, de los famosos poetas
contemplando y oradores;
cómo se passa la vida; non curo7 de sus ficciones
cómo se viene la muerte que traen yerbas secretas8
tan callando; sus sabores.
cuán presto2 se va el plazer; Aquél sólo m’encomiendo,
cómo, después de acordado,3 aquél sólo invoco yo
da dolor; de verdad,
cómo a nuestro parescer, que en este mundo viviendo,
cualquier tiempo passado el mundo non conoció
fue mejor. su deidad.

II Pues si vemos lo presente, V Este mundo es el camino


cómo en un punto s’es ido para el otro, qu’es morada
e acabado, sin pesar;
si juzgamos sabiamente mas cumple tener buen tino
daremos lo non venido para andar esta jornada
por passado. sin errar.
Non se engañe nadi,4 no, Partimos cuando nascemos,
pensando que ha de durar andamos mientras vivimos,
lo que espera e llegamos
más que duró lo que vio, al tiempo que feneçemos;9
pues que todo ha de passar assí que cuando morimos,
por tal manera. descansamos.

III Nuestras vidas son los ríos VI Este mundo bueno fue
que van a dar en el mar, si bien usásemos dél
qu’es el morir; como debemos,
allí van los señoríos porque, segund nuestra fe,
derechos a se acabar es para ganar aquél
e consumir; que atendemos.10
allí los ríos caudales,5 Aun aquel fijo11 de Dios
allí los otros medianos para sobirnos al cielo
e más chicos, descendió
allegados,6 son iguales a nascer acá entre nos,
los que viven por sus manos y a vivir en este suelo
e los ricos. do murió.

1 recuerde Vuelva en sí, despierte. 8 yerbas secretas Venenos. Se refiere a la tradición


2 presto De prisa, rápido. pagana de las invocaciones a las musas y a otros
poderes de la mitología.
3 acordado Recordado.
9 feneçemos Morimos.
4 nadi Nadie.
10 atendemos Esperamos
5 caudales Grandes.
11 Algunos arcaismos fijo, hijo; sobirnos, subirnos;
6 allegados Probablemente llegados [allí].
nescer, nacer; nos, nosotros; do, donde.
7 non curo No me preocupo.

Lengua castellana y literatura 1 13


COMENTARIOS DE TEXTO Jorge Manrique
BAT

VII Ved de cuán poco valor X Los estados16 e riqueza


son las cosas tras que andamos que nos dexan a deshora
y corremos, ¿quién lo duda?
que, en este mundo traidor, no les pidamos firmeza,
aun primero que muramos pues que son d’una señora;
las perdemos. que se muda,17
Dellas deshaze la edad, que bienes son de Fortuna
dellas casos desastrados que revuelve con su rueda
que acaeçen,12 presurosa,
dellas, por su calidad, la cual no puede ser una
en los más altos estados ni estar estable ni queda
desfallescen. en una cosa.

VIII Dezidme: La hermosura, XI Pero digo c’acompañen


la gentil frescura y tez13 e lleguen fasta la fuessa18
de la cara, con su dueño:
la color e la blancura por esso non nos engañen,
cuando viene la vejez, pues se va la vida apriessa
¿cuál se para? como sueño.
Las mañas e ligereza e los deleites de acá
e la fuerza corporal son, en que nos deleitamos,
de juventud, temporales,
todo se torna graveza e los tormentos de allá,
cuando llega el arrabal que por ellos esperamos,
de senectud.14 eternales.

IX Pues la sangre de los godos,15


y el linaje e la nobleza
tan crescida,
¡por cuántas vías e modos
se pierde su grand alteza
en esta vida!
Unos, por poco valer,
por cuán baxos e abatidos
que los tienen;
otros que, por no tener,
con oficios non debidos
se mantienen.
12 acaeçen Suceden.
13 tez Color, lisura y plenitud del rostro.
14 el arrabal de senectud La vejez.
15 Los godos han sido siempre considerados como el
origen de la más limpia y antigua nobleza.
16 estados o estamentos Hace referencia a la posición
social de los individuos.
17 se muda Cambia.
18 fuessa Fosa.

Lengua castellana y literatura 1 14


COMENTARIOS DE TEXTO Jorge Manrique
BAT

XII Los plazeres e dulçores XV Dexemos a los troyanos,


desta vida trabajada que sus males non los vimos,
que tenemos ni sus glorias;
no son sino corredores, dexemos a los romanos,
e la muerte, la çelada19 aunque oimos e leimos
en que caemos. sus hestorias;
Non mirando a nuestro daño, non curemos de saber
corremos a rienda suelta lo d’aquel siglo passado
sin parar; qué fue d’ello;
desque vemos el engaño vengamos a lo d’ayer,
y queremos dar la vuelta que también es olvidado
no hay lugar. como aquello.

XIII Si fuesse20 en nuestro poder XVI ¿Qué se hizo el rey don Juan?
hazer la cara hermosa Los Infantes d’Aragón
corporal, ¿qué se hicieron?
como podemos hazer ¿Qué fue de tanto galán?,
el alma tan glorïosa ¿qué de tanta invención
angelical, como trajeron?
¡qué diligencia, tan viva ¿Fueron sino devaneos,22
toviéramos toda hora, qué fueron sino verduras
e tan presta, de las eras,
en componer la cativa21 las justas e los torneos
dexándonos la señora paramentos,23 bordaduras
descompuesta. e çimeras?24

XIV Esos reyes poderosos XVII ¿Qué se hicieron las damas,


que vemos por escripturas sus tocados y vestidos
ya passadas sus olores?
con casos tristes, llorosos, ¿Qué se hicieron las llamas
fueron sus buenas venturas de los fuegos encendidos
trastornadas; d’amadores?
assí, que no ay cosa fuerte, ¿Qué se hizo aquel trovar,
que a papas y emperadores las músicas acordadas
e perlados, que tañían?
assí los trata la muerte ¿Qué se hizo aquel dançar
como a los pobres pastores aquellas ropas chapadas25
de ganados. que traían?

Lengua castellana y literatura 1 15


COMENTARIOS DE TEXTO Jorge Manrique
BAT

XVIII Pues el otro, su heredero XXI Pues aquel Condestable,32


don Enrique,26 ¡qué poderes maestre que conoscimos
alcançaba! tan privado,
¡Cuánd blando, cuánd halaguero27 non cumple que dél se hable,
el mundo con sus plazeres mas sólo cómo lo vimos
se le daba! degollado.
Mas verás cuánd enemigo, Sus infinitos tesoros,
cuánd contrario, cuánd cruel sus villas y sus lugares,
se le mostró; su mandar,
habiéndole sido amigo, ¿qué fueron sino lloros?
¡cuánd poco duró con él ¿qué fueron sino pesares,
lo que le dio! al dexar?

XIX Las dádivas desmedidas, XXII E los otros dos hermanos,33


los edificios reales maestres tan prosperados
llenos d’oro, como reyes,
las vaxillas tan fabridas,28 c’a los grandes e medianos
los enriques e reales29 trajeron tan sojuzgados
del tesoro, a sus leyes;
los jaezes,30 los caballos aquella prosperidad
de sus gentes e atavíos qu’en tan alto fue subida
tan sobrados y ensalzada,
¿dónde iremos a buscallos? ¿qué fue sino claridad
¿qué fueron sino rocíos que cuando más encendida
de los prados? fue amatada?

XX Pues su hermano el innocente31


qu’en su vida sucessor 19 çelada Celada, trampa.
20 si fuesse (verbo ser) Si estuviese.
se llamó
21 cativa, cautiva Es la oposición “señora/cautiva”,
¡qué corte tan excellente “esclava/dueña”, que contrapone la cara corporal
tuvo, e cuánto gran señor y el alma angelical.
22 devaneos Pasatiempos inútiles.
le siguió!
23 paramentos Adornos ricos con que se cubrían
Mas, como fuesse mortal, los caballos en los torneos.
metióle la Muerte luego 24 cimeras Penachos o adornos de plumas en que
remataban los yelmos.
en su fragua.
25 chapadas Recubiertas con láminas o con chapas.
¡Oh jüicio divinal!, 26 Don Enrique Es Enrique IV, hijo y sucesor de Juan II.
cuando más ardía el fuego, 27 halaguero Lisonjero, halagador.
echaste agua. 28 fabridas Trabajadas con primor.
29 enriques y reales Eran monedas en curso
de la época.
30 jaeces Adornos que se ponen a los caballos.
31 Se trata de don Alfonso, quien todavía niño fue
procamado rey en Ávila. Tres años más tarde, a los
catorce años de edad, moría sin llegar a reinar.
32 Es el condestable don Álvaro de Luna que asumió
todo el poder en tiempos de Juan II y acabó
decapitado en Valladolid.
33 Personajes de la época, favoritos de Enrique IV.

Lengua castellana y literatura 1 16


COMENTARIOS DE TEXTO Jorge Manrique
BAT

XXIII Tantos duques excelentes, XXVI Amigo de sus amigos,


tantos marqueses e condes ¡qué señor para criados
e varones e parientes!
como vimos tan potentes, ¡Qué enemigo d’enemigos!
dí, Muerte, ¿dó los escondes, ¡Qué maestro d’esforçados
e traspones? e valientes!
E las sus claras hazañas ¡Qué seso para discretos!
que hizieron en las guerras ¡Qué gracia para donosos!
y en las pazes, ¡Qué razón!
cuando tú, cruda, t’ensañas, ¡Qué benino a los sujetos!
con tu fuerça las atierras34 ¡A los bravos e dañosos,
e desfazes. qué león!

XXIV Las huestes35 innumerables, XXVII En ventura, Octavïano;


los pendones, estandartes Julio César en vencer
e banderas, e batallar;
los castillos impugnables,36 en la virtud, Africano;
los muros e balüartes Aníbal en el saber
e barreras, e trabajar;
la cava honda, chapada,37 en la bondad, un Trajano;
o cualquier otro reparo,38 Tito en liberalidad
¿qué aprovecha? con alegría;
Cuando tú vienes airada, en su braço, Aureliano;
todo lo passas de claro39 Marco Atilio en la verdad
con tu flecha. que prometía.

XXV Aquel de buenos abrigo, XXVIII Antoño Pío en clemencia;


amado, por virtuoso, Marco Aurelio en igualdad
de la gente, del semblante;
el maestre don Rodrigo Adriano en la elocuencia;
Manrique, tanto famoso Teodosio en humanidad
e tan valiente; e buen talante.
sus hechos grandes e claros Aurelio Alexandre fue
non cumple que los alabe, en disciplina e rigor
pues los vieron; de la guerra;
ni los quiero hazer caros,40 un Constantino en la fe,
pues qu’el mundo todo sabe Camilo, en el grand amor
cuáles fueron. de su tierra.

34 atierras Echas por tierra. 38 reparo Obstáculo.


35 huestes Ejércitos. 39 de claro De un lado a otro.
36 impugnables Inexpugnables. 40 hacer caros Encarecer.
37 la cava honda, chapada Foso o trinchera, que para
hacerla más fuerte se recubría con planchas de metal.

Lengua castellana y literatura 1 17


COMENTARIOS DE TEXTO Jorge Manrique
BAT

XXIX No dexó grandes tesoros,


ni alcançó muchas riquezas
ni vaxillas;
mas fizo guerra a los moros
ganando sus fortalezas
e sus villas;
y en las lides que venció,
cuántos moros e cavallos
se perdieron;
y en este oficio ganó
las rentas e los vasallos
que le dieron.

XXX Pues su honra y estado, XXXII E sus villas e sus tierras,


en otros tiempos pasados ocupadas de tiranos
¿cómo s’hubo?41 las halló;
Quedando desamparado, mas por çercos e por guerras
con hermanos e criados e por fuerça de sus manos
se sostuvo. las cobró.
Después que fechos famosos Pues nuestro rey natural,
fizo en esta misma guerra si de las obras que obró
que hazía, fue servido,
fizo tratos tan honrosos dígalo el de Portugal,44
que le dieron aún más tierra y, en Castilla, quién siguió
que tenía. su partido.

XXXI Estas sus viejas hestorias XXXIII Después de puesta la vida


que con su braço pintó tantas vezes por su ley
en joventud, al tablero,45
con otras nuevas victorias después de tan bien servida
agora las renovó la corona de su rey
en senectud. verdadero;
Por su gran habilidad, después de tanta hazaña
por méritos e ancianía que non puede bastar
bien gastada42 cuenta cierta,
alcançó la dignidad en la su villa de Ocaña
de la grand Caballería vino la Muerte a llamar
dell Espada.43 a su puerta,

41 ¿cómo shubo? ¿Cómo se comportó y cómo


se mantuvo?
42 bien gastada Bien empleada.
43 Don Rodrigo Manrique fue maestre de la Orden
de Caballería de Santiago de la Espada.
44 el de Portugal, El rey de Portugal, Alfonso V.
45 Después de haberse jugado tantas veces la vida.

Lengua castellana y literatura 1 18


COMENTARIOS DE TEXTO Jorge Manrique
BAT

XXXIV diziendo: “Buen caballero,


dexad el mundo engañoso
e su halago;
vuestro corazón de azero
muestre su esfuerço famoso
en este trago;
e pues de vida e salud
fezistes tan poca cuenta
por la fama;
esfuércese la virtud
para sofrir esta afruenta46
que vos llama.”

XXXV “Non se vos haga tan amarga


la batalla temerosa47
qu’esperáis,
pues otra vida más larga
de la fama glorïosa
acá dexáis.
Aunqu’esta vida d’honor XXXVII “E pues vos, claro varón,
tampoco no es eternal tanta sangre derramastes
ni verdadera; de paganos,
mas, con todo, es muy mejor esperad el galardón
que la otra temporal, que en este mundo ganastes
peresçedera.” por las manos;
e con esta confiança
XXXVI “El vivir qu’es perdurable e con la fe tan entera
non se gana con estados que tenéis,
mundanales, partid con buena esperança,
ni con vida delectable48 qu’estotra vida tercera
donde moran los pecados ganaréis.”
infernales;
mas los buenos religiosos 46 sofrir esta afruenta Sufrir esta afrenta
gánanlo con oraciones (la afrenta, humillación, de ser vencido
por la muerte).
e con lloros;
47 temerosa Temible.
los caballeros famosos, 48 delectable Deleitosa.
con trabajos y aflicciones 49 tengamos Perdamos.
contra moros.”

Lengua castellana y literatura 1 19


COMENTARIOS DE TEXTO Jorge Manrique
BAT

[Responde el Maestre:] ESTROFAS I-XIII

1 Observa que el poema empieza con una ad-


XXXVIII “No tengamos49 tiempo ya
vertencia al alma “dormida” para que “des-
en esta vida mesquina
pierte”. Di qué debe observar ésta para tomar
por tal modo, conciencia (o despertar).
que mi voluntad está
conforme con la divina
para todo; 2 Explica el sentido de la hipérbole que ocupa
la primera sextilla de la copla II.
e consiento en mi morir
con voluntad plazentera,
clara e pura, 3 Analiza la alegoría que desarrolla la copla III.
que querer hombre vivir Indica las correspondencias de términos reales
cuando Dios quiere que muera y figurados de la suma de metáforas que la
es locura.” componen. Di qué elemento destaca, en los
dos versos finales, como diferenciador social.
[Del Maestre a Jesús:]
4 Explica qué rechaza y qué reivindica Manrique
XXXIX Tú que, por nuestra maldad, en su Invocación.
tomaste forma servil
e baxo nombre;
5 Analiza la alegoría de la copla V. Explica su
tú, que a tu divinidad significado relacionándolo con la copla VI.
juntaste cosa tan vil
como es el hombre;
tú, que tan grandes tormentos 6 Explica en qué basa el poeta el poco valor que
sofriste sin resistencia atribuye a las cosas que el hombre aprecia (las
cosas tras que andamos y corremos). Observa có-
en tu persona;
mo, en la segunda sextilla, apunta ambigua-
non por mis merescimientos,
mente algunas de ellas.
mas por tu sola clemencia
me perdona.
7 Fíjate en que las coplas VIII, IX y X son más ex-
XL Así, con tal entender, plícitas. ¿Qué bien terrenal evalúa en cada una
de ellas? Indica en cada caso qué acaba con
todos sentidos humanos conservados,
éstos.
cercado de su mujer
y de sus hijos e hermanos
e criados, 8 Observa que en la copla X aparece un perso-
dio el alma a quien se la dio naje alegórico: la Fortuna. Explica cómo lo ca-
–el cual la ponga en el cielo racteriza.
en su gloria–,
que aunque esta vida perdió, 9 Fíjate en que la estrofa XI contempla la posi-
dexónos harto consuelo bilidad de que el hombre conserve sus bie-
su memoria. nes hasta la muerte. ¿Debe alimentar espe-
ranzas en este sentido? ¿Por qué? ¿Qué nos
Jorge Manrique, “Coplas de don Jorge Manrique
por la muerte de su padre”, en Poesía. recuerda el poeta en la segunda sextilla?
Editorial Cátedra. ¿Qué recursos estilísticos dan fuerza y clari-
dad a estas ideas?

Lengua castellana y literatura 1 20


COMENTARIOS DE TEXTO Jorge Manrique
BAT

10 Explica el sentido de las estrofas XII y XIII; ESTROFAS XXV A XL


fíjate en que ambas reiteran ideas expuestas
18 Indica quién protagoniza esta parte de la
utilizando imágenes nuevas.
obra.

ESTROFAS XIV-XXIV
19 Di cuál es el tema principal de las estrofas
11 Lee esta parte de la composición; ejem- XXV a XXXIII y cómo se llama esta parte de
plificará con casos concretos las ideas expues- una elegía.
tas en la parte inicial de la obra. ¿Qué perso-
najes aparecen? Observa en qué orden y el
espacio que se dedica a cada uno.
20 Distingue la parte dedicada a las virtudes del
personaje de aquella dedicada a sus hazañas.

12 Explica en qué consiste el tópico del ubi sunt


21 ¿Qué ha hecho perdurar la fama de los per-
que aparece en esta parte de la obra, espe-
sonajes que aparecen en las coplas XXVII-
cialmente claro en las coplas XVI-XVII. Ejem-
XXVIII? ¿Qué les diferencia de aquellos que
plifícalo citando fragmentos.
aparecen en el tópico del ubi sunt? Relaciona
la época a la que pertenecen con el cambio
13 Localiza los versos de la estrofa XIV en que cultural que se está produciendo a finales del
aparece con mayor claridad la idea, tan siglo xv.
medieval, del poder igualador de la muer-
te.
22 Fíjate en que la copla XXIX inaugura la evo-
cación de la vida de don Rodrigo. ¿Qué as-
14 Observa que en la copla XV Manrique pecto de ésta se destaca?
expresa su propósito de renovar el tópico
del ubi sunt. ¿Qué descarta y sobre qué
23 Localiza en las estrofas XXX-XXXII fragmentos
reclama atención?
ilustrativos del hecho de que don Rodrigo
Manrique participase activamente en las lu-
15 Lee las coplas XVI-XVII: evocan el ambiente chas políticas de su tiempo.
de la corte de Juan II. Di qué elementos pro-
pios de la vida cortesana se mencionan. 24 Lee con atención la estrofa XXXIII, que cierra
el repaso de la vida de don Rodrigo. Comenta
la función de la anáfora y di qué personaje
16 Observa que todos los personajes que apa-
aparece en escena. Después, responde:
recen en esta parte de la obra han destaca-
do en vida por su poder y su riqueza. ¿Con a ¿Qué estrofas ocupa el parlamento de este per-
qué imágenes expresa el poeta la brevedad sonaje? ¿En qué tono se dirige a don Rodrigo?
de estos bienes? ¿Con qué argumentos le anima a afrontar el
trance que se avecina?
b ¿Cuáles son las tres vidas de las que habla el
17 Localiza el receptor interno de las estrofas personaje?
XXIII y XXIV. Cita las enumeraciones que
hay en ellas y comenta su función estilística. c ¿Qué expresa la respuesta de don Rodrigo?
¿A quién dirige sus últimas palabras?
d ¿Qué hace del final de don Rodrigo un buen
morir? ¿Qué consuelo deja a los suyos?

Lengua castellana y literatura 1 21


COMENTARIOS DE TEXTO La Celestina
BAT

LA CELESTINA
A. En el acto I, que posiblemente no sea de Rojas, se produce el 1 Después de leer el primer
encuentro entre Calisto y Melibea. Él va buscando su halcón y fragmento (A), responde:
entra en una huerta donde ve a Melibea; Calisto se enamora a a ¿Qué siente Calisto al ver a
primera vista y se declara apasionadamente a la doncella, que lo Melibea? ¿Con qué la
rechaza sin contemplaciones. compara?
1 CALISTO.- En esto veo, Melibea, la grandeza de Dios. b ¿Qué términos propios del
MELIBEA.- ¿En qué, Calisto? amor cortés emplea Calisto?
CALISTO.- En dar poder a natura que de tan perfecta hermosura
c ¿Qué responde Melibea?
te dotasse, y hacer a mí, inmérito, tanta merced que verte alcan-
¿Por qué?
5 çasse, y, en tan conveniente lugar, que mi secreto dolor mani-
festarte pudiesse. Sin duda, incomparablemente es mayor tal
galardón que el servicio, sacrificio, devoción y obras pías que por 2 Después de leer el
este lugar alcançar yo tengo a Dios offrecido ni otro poder mi segundo fragmento (B),
voluntad humana puede cumplir. ¿Quién vido en esta vida cuer-
responde:
10 po glorificado de ningún hombre como agora el mío? Por cier-
to, los gloriosos santos que se deleytan en la visión divina, no a ¿En qué idea insiste Calisto
gozan más que yo agora en el acatamiento tuyo. Mas, o triste, en este fragmento? ¿Cómo
que en esto deferimos, que ellos puramente se glorifican sin ha evolucionado su estado?
temor de caer de tal bienaventurança, y yo, misto, me alegro con b ¿Con qué rasgos físicos y
15 recelo del esquivo tormento que tu absencia me ha de causar.
psicológicos caracteriza
MELIBEA.- ¿Por gran premio tienes éste, Calisto? Sempronio a Celestina?
CALISTO.- Téngolo por tanto, en verdad, que si Dios me diesse en ¿A qué se dedica la vieja?
el cielo la silla sobre sus santos, no lo ternía por tanta felicidad.
c ¿Qué se puede pensar de
MELIBEA.- Pues, ¡aún más ygual galardón te daré yo, si perseveras! Calisto después de leer los
20 CALISTO.- ¡Oh bienaventuradas orejas mías que indignamente tan dos fragmentos
gran palabra avéys oýdo! pertenecientes al comienzo
MELIBEA.- Más desventuradas de que me acabes de oýr, porque la de la obra?
paga será tan fiera qual meresce tu loco atrevimiento, y el inten-
d ¿Qué aspectos de la
to de tus palabras ha seýdo como de ingenio de tal hombre como
25 tú aver de salir para se perder en la virtud de tal mujer como yo.
conducta de Calisto hacen
prever un final trágico de la
¡Vete, vete de aý, torpe! que no puede mi paciencia tolerar que
haya subido en coraçón humano conmigo el ilícito amor comu- obra?
nicar su deleyte.
CALISTO.- Yré como aquel contra quien solamente la adversa
30 Fortuna pone su studio con odio cruel.

B. Calisto regresa angustiado a su casa y acaba confesándose con


su criado Sempronio. Ambos exponen sus opiniones sobre el
amor y las mujeres y Sempronio ofrece a su amo una solución
para conseguir saciar la pasión que lo devora: Celestina.
1 SEMPRONIO.- ¿Tú no eres christiano?
CALISTO.- ¿Yo? Melibeo só, y a Melibea adoro, y en Melibea creo,
y a Melibea amo. […]
SEMPRONIO.- […] conozco en fin desta vezindad una vieja barbuda
5 que se dize Celestina, hechizera, astuta, sagaz en quantas mal-
dades hay. Entiendo que passan de cinco mil virgos los que se
han hecho y desecho por su autoridad en esta cibdad. A las duras
peñas promeverá y provocará a luxuria, si quiere.

Lengua castellana y literatura 1 22


COMENTARIOS DE TEXTO La Celestina
BAT

C. El egoísmo individualista es propio de todos los personajes de 1 Lee el fragmento C


la obra. En el acto XII, Sempronio y Pármeno matan a Celestina y resuelve estas actividades:
porque ésta no quiere repartir el botín que ha obtenido de
Calisto con ellos. Posteriormente, los dos criados son prendidos
a ¿Qué registro emplean los
y ajusticiados. personajes? Justifica tu
respuesta.
1 SEMPRONIO.- […] ¿Quién la oyó esta vieja dezir que me llevasse yo
todo el provecho, si quisiesse, deste negocio, pensando que sería b ¿Cómo se defiende
poco? Agora que lo vee crescido, no quiere dar nada, por com- Celestina? ¿Qué reivindica?
plir el refrán de los niños que dizen: De lo poco, poco, de lo
5 mucho, nada. Relaciona la
argumentación de
PÁRMENO.- Dete lo que te prometió o tomémosselo todo. Harto
te dezía yo quién era esta vieja, si tú me creyeras. Celestina con los cambios
sociales del primer Renaci-
CELESTINA.- […] ¿Quién só yo, Sempronio? ¿Quitásteme de la
miento.
putería? Calla tu lengua; no amengües mis canas, que soy una
10 vieja qual Dios me hizo, no peor que todas. Bivo de mi officio,
como cada qual official del suyo, muy limpiamente. A quien no 2 Lee el fragmento D
me quiere, no le busco. Si bien o mal bivo, Dios es el testigo de y responde:
mi coraçón. Y no pienses con tu yra maltratarme, que justicia ay
para todos, a todos es ygual […] a ¿Quién se muestra más digno
15 PÁRMENO.- […] ¡Dale, dale, acábala, pues començaste; que nos y sutil, Calisto o Melibea?
sentirán; muera, muera, de los enemigos los menos! ¿Por qué?
CELESTINA.- ¡Confessión! b ¿Cómo demuestra Calisto
lo vulgar de su pasión por
Melibea? ¿Podemos igualar
su comportamiento y su
D. En el acto XIX, muere Calisto al dar un mal paso bajando por
lenguaje con el de los
la escalera la tapia del huerto de Melibea. Momentos antes,
criados?
Calisto goza de Melibea mientras Lucrecia, la criada de ésta, se
muere de envidia. c ¿Qué muestra Lucrecia, la
1 MELIBEA.- […] Y pues tú, señor, eres el dechado de cortesía y criada de Melibea, en su
buena criança, ¿cómo mandas a mi lengua hablar y no a tus aparte? ¿Es muy diferente
manos que estén quedas? ¿Por qué no olvidas estas mañas? […] su actitud a la de Calisto o,
Holguemos y burlemos de otros mil modos que yo te mostraré; incluso, Melibea?
5 no me destroces ni maltrates como sueles. ¿Qué provecho te trae
dañar mis vestiduras?
CALISTO.- Señora, el que quiere comer el ave, quita primero las
plumas.
LUCRECIA.- (Mala landre me mate si más lo escucho; ¿vida es ésta?
10 Que me esté yo deshaziendo de dentera y ella esquivándose por
que la rueguen […] pero también me lo haría yo si estos necios
de sus criados me fablassen entre día, pero esperan que los tengo
de yr a buscar.)

Lengua castellana y literatura 1 23


COMENTARIOS DE TEXTO La Celestina
BAT

E. Melibea se suicida, no sin antes explicar a su padre todo lo 1 Lee el fragmento E


ocurrido. La obra finaliza, en el acto XXI, con el planto de Ple- y responde:
berio.
a ¿Hay resignación cristiana
1 PLEBERIO.- ¡Ay, ay, noble mujer, nuestro gozo en el pozo; nuestro en las palabras de Pleberio?
bien todo es perdido; no queramos más bivir! […] ¡O mi hija y ¿Cómo se plantea la vida?
mi bien todo, crueldad sería que biva yo sobre ti! Más dignos
eran mis sesenta años de la sepultura, que tus veynte. […] ¿Para b ¿Qué opinión tiene del amor
5 quién edifiqué torres; para quién adquirí honrras; para quién y de los bienes terrenales?
planté árboles, para quién fabriqué navíos? […] ¡O fortuna varia- ¿Puede aparecer ahí parte
ble, ministra y mayordoma de los temporales bienes! […] ¡O del mensaje de la obra?
amor, amor, que no pensé que tenías fuerça ni poder de matar a c ¿Qué recursos retóricos se
tus sujectos! […] Del mundo me quexo porque en sí me crió, por- emplean en el fragmento?
10 que no me dando la vida no engendrara en él a Melibea; no nas-
cida, no amara; no amando, cessara mi quexosa y desconsolada d ¿Qué rasgos propios de la
postremería […] comedia humanística se
observan en los tres últimos
fragmentos que has leído
(C, D, E)?

Grabado perteneciente a la impresión de La Celestina de Burgos de 1531.

Lengua castellana y literatura 1 24


COMENTARIOS DE TEXTO Garcilaso de la Vega
BAT

ÉGLOGA I
Las églogas son poemas bucólicos o Al virrey de Nápoles.
pastoriles, ya cultivados en la Anti- 1
güedad clásica. Presentan escenas o
1 El dulce lamentar de dos pastores,
diálogos amorosos entre pastores
Salicio juntamente y Nemoroso,
idealizados. En la Égloga I, Garci-
he de cantar, sus quejas imitando;
laso pone en boca de dos pastores,
Salicio y Nemoroso, dos momen-
cuyas ovejas al cantar sabroso
tos muy distintos de su propia 5 estaban muy atentas, los amores,
experiencia amorosa: el desamor, de pacer olvidadas, escuchando. [...]
en un caso, y la muerte de la ama-
da, en otro. 4
Saliendo de las ondas encendido,
rayaba de los montes el altura
el sol, cuando Salicio, recostado
10 al pie d’una alta haya, en la verdura
por donde una agua clara con sonido
atravesaba el fresco y verde prado,
él, con canto acordado1
al rumor que sonaba
15 del agua que pasaba,
se quejaba tan dulce y blandamente
como si no estuviera de allí ausente
la que de su dolor culpa tenía,
y así como presente,
20 razonando con ella, le decía:

5
Salicio: ¡Oh más dura que mármol a mis quejas,
y al encendido fuego en que me quemo
más helada que nieve, Galatea!
Estoy muriendo, y aun la vida temo;
25 témola con razón, pues tú me dejas;
que no hay, sin ti, el vivir para que sea.
Vergüenza he2 que me
vea ninguno en tal estado,
de ti desamparado,
3
30 y de mí mismo yo me corro agora.

¿De un alma te desdeñas ser señora,


donde siempre moraste, no pudiendo
della4 salir un hora?
Salid sin duelo, lágrimas, corriendo.

Lengua castellana y literatura 1 25


COMENTARIOS DE TEXTO Garcilaso de la Vega
BAT

8
35 Por ti el silencio de la selva umbrosa5
por ti la esquividad y apartamiento
del solitario monte me agradaba;
por ti la verde hierba, el fresco viento
el blanco lirio y colorada rosa
40 y dulce primavera deseaba.

¡Ay, cuánto me engañaba!


¡Ay, cuán diferente era
y cuán de otra manera
lo que en tu falso pecho se escondía!
45 Bien claro con su voz me lo decía

la siniestra6 corneja repitiendo


la desventura mía.
Salid sin duelo, lágrimas, corriendo.

10
Tu dulce habla, ¿en cúya oreja suena?
50 Tus claros ojos ¿a quién los volviste?
¿Por quién tan sin respeto me trocaste?7
Tu quebrantada fe ¿dó la pusiste?
¿Cuál es el cuello que, como en cadena,
de tus hermosos brazos anudaste?
55 no hay corazón que baste,
aunque fuese de piedra,
viendo mi amada hiedra
de mí arrancada, en otro muro asida,
y mi parra en otro olmo entretejida,
60 que no se esté con llanto deshaciendo
hasta acabar la vida.
Salid sin duelo, lágrimas, corriendo.

12
Materia diste al mundo de esperanza
de alcanzar lo imposible y no pensado,
65 y de hacer juntar lo diferente,
dando a quien diste el corazón malvado,
quitándolo de mí con tal mudanza,
que siempre sonará de gente en gente. 1 acordado El canto armoniza con el rumor
La cordera paciente del agua.
70 con el lobo hambriento
2 he El verbo haber se utilizaba todavía en el siglo
XVI con el valor de “tener”, tengo vergüenza’.
hará su ayuntamiento,
3 yo me corro agora Yo me avergüenzo ahora.
y con las simples aves sin ruido 4 della De ella (es una contracción).
harán las bravas sierpes8 ya su nido: 5 umbrosa Sombría, oscura.
que mayor diferencia comprehendo 6 siniestra Que vuela a la izquierda. Mal augurio.
75 de ti al que has escogido 7 trocaste Cambiaste.
Salid sin duelo, lágrimas, corriendo. 8 sierpes Serpientes.

Lengua castellana y literatura 1 26


COMENTARIOS DE TEXTO Garcilaso de la Vega
BAT

18
Nemoroso: Corrientes aguas, puras, cristalinas,
árboles que os estáis mirando en ellas,
verde prado de fresca sombra lleno,
9
80 aves que aquí sembráis vuestras querellas,
hiedra que por los árboles caminas,
torciendo el paso por su verde seno:
yo me vi tan ajeno
del grave mal que siento
85 que de puro contento
con vuestra soledad me recreaba,
donde con dulce sueño reposaba,
o con el pensamiento discurría
por donde no hallaba
90 sino memorias llenas d’alegría;

19
y en este mismo valle, donde agora
me entristezco y me canso en el reposo,
estuve ya contento y descansado.
¡Oh bien caduco, vano y presuroso!
95 Acuérdome, durmiendo aquí algún hora,

que, despertando, a Elisa vi a mi lado.


¡Oh miserable hado!10
¡Oh tela delicada,
antes de tiempo dada
11
100 a los agudos filos de la muerte!

Más convenible fuera aquesta suerte


a los cansados años de mi vida,
qu’es más que’l hierro fuerte,
pues no la ha quebrantado tu partida.

20
105 ¿Dó están agora aquellos claros ojos
que llevaban tras sí, como colgada,
mi alma doquier que ellos se volvían?
¿Dó está la blanca mano delicada,
llena de vencimientos y despojos12
110 que de mí mis sentidos l’ofrecían?
Los cabellos que vían13
con gran desprecio el oro
como a menor tesoro,
¿adónde están? ¿Adónde el blanco pecho?
115 ¿Dó la columna que’l dorado techo
9 querellas Quejas. Como si el canto de las con proporción graciosa sostenía?
aves también tratara de amores. 10 hado
Suerte, destino. 11 ¡Oh tela… muerte!
Aquesto todo agora ya s’encierra,
Metafóricamente la vida es una tela o trama por desventura mía,
tejida por los hilos de una parca y cortada por en la escura, desierta y dura tierra.
otra. 12 vencimientos y despojos
Metafóricamente, pruebas del triunfo de la Garcilaso de la Vega, en Poesías castellanas completas.
dama. 13 vían Veían. Editorial Castalia.

Lengua castellana y literatura 1 27


COMENTARIOS DE TEXTO Garcilaso de la Vega
BAT

1 Haz un análisis métrico completo de la pri- 12 Resume el sentido del fragmento exclamativo.
mera estrofa, que es una estancia, combina- ¿Qué señal desatendió el pastor?
ción libre de versos endecasílabos y heptasí-
labos cuya distribución y rima se fijan al
ESTROFA 10
principio de la composición y se mantienen.
13 Indica las semejanzas formales y de contenido
ESTROFA 1 que presentan los seis primeros versos y las
figuras literarias que aparecen.
2 Prosifica los versos iniciales de la égloga des-
haciendo el hipérbaton.
14 Localiza en los versos siguientes dos alego-
rías y analízalas (indica el término real y el
3 Di qué tiene de peculiar la actitud de las ove- figurado de cada metáfora).
jas. Observa a partir de ahora la ocasional ani-
mación de elementos de la naturaleza, indicio
ESTROFA 12
de la armonía entre el hombre y ésta que el
poeta sugiere. 15 Contesta:
a ¿Por qué dice el pastor que a partir de ahora
ESTROFA 4 se podrá esperar cualquier cosa?
4 Di qué momento del día refleja el inicio de b ¿Qué ejemplos de imposibles aparecen en los
la égloga. versos siguientes? Relaciónalos con la causa de
su queja.

5 Enumera los elementos que forman el locus


ESTROFA 18 Y 19
amoenus y las características que se les atribu-
yen. 16 Localiza las invocaciones con que se abre el
canto de Nemoroso. Comprueba si es la pri-
mera vez que aparece esta comunicación entre
6 Indica qué tono domina el canto de Salicio. los pastores y la naturaleza. Observa si hay per-
sonificaciones de elementos de la naturaleza.
7 Localiza la perífrasis que nombra a la desti-
nataria de la queja. 17 Observa cómo el mismo espacio ha sido tes-
tigo de la felicidad pasada y del mal presente.
ESTROFA 5 ¿Cómo describe el pastor cada momento?
8 ¿Cómo caracteriza Salicio a Galatea? ¿Qué
sentimientos expresa él? 18 Explica el sentido de su reflexión final.

9 Describe la alternancia de modalidades ora- ESTROFA 20

cionales, recurso que se mantendrá en toda 19 Fíjate en que en la estrofa 20 se superponen


la composición como reflejo del vaivén emo- dos tópicos literarios: la descripción de la da-
cional del emisor. ma (descriptio puellae) y una peculiar versión
del ubi sunt.
10 Observa el epifonema (exclamación referida
Di cómo es la dama descrita.
a lo dicho anteriormente con la cual se cierra
el pensamiento al que se refiere). Se manten- Explica qué diferencia hay entre el sentido
drá a lo largo del canto de Salicio. original del tópico del ubi sunt y el uso que
de él hace Garcilaso.
ESTROFA 8
Observa la respuesta que se da a las preguntas
11 Localiza los epítetos que destacan las carac- retóricas y señala las antítesis entre las carac-
terísticas del paisaje idealizado. Explica por terísticas de la dama y el lugar donde se en-
qué se le hacía especialmente agradable. cuentra.

Lengua castellana y literatura 1 28


COMENTARIOS DE TEXTO Lazarillo de Tormes
BAT

LAZARILLO DE TORMES
En el tratado primero, Lázaro 1 Pues sepa Vuestra Merced, ante todas cosas, que a mí llaman Lá-
explica sus orígenes que, a dife- zaro de Tormes, hijo de Tomé González y de Antona Pérez, na-
rencia de los de los héroes ca- turales de Tejares, aldea de Salamanca. Mi nascimiento fue dentro
ballerescos, no son precisamente del río Tormes, por la cual causa tomé el sobrenombre, y fue desta
honrados. También cuenta Lá- 5 manera: mi padre, que Dios perdone, tenía cargo de proveer una
zaro cómo conoció a su primer molienda de una aceña1 que esta ribera de aquel río, en la cual
amo, el ciego. fue molinero más de quince años; y estando mi madre una noche
en la aceña, preñada de mí, tomole el parto y pariome allí; de
manera que con verdad me puedo decir nascido en el río.
10 Pues siendo yo niño de ocho años, achacaron a mi padre ciertas
sangrías mal hechas en los costales de los que allí a moler venían,
por lo cual fue preso, y confesó, y no negó, y padesció persecución
por justicia. Espero en Dios que está en la Gloria, pues el Evangelio
los llama bienaventurados. En ese tiempo se hizo cierta armada
15 contra moros, entre los cuales fue mi padre, que a la sazón estaba
desterrado por el desastre ya dicho, con cargo de acemilero de un
caballero que allá fue. Y con su señor, como leal criado, fenesció
su vida.
Mi viuda madre […] metiose a guisar de comer a ciertos estu-
20 diantes, y lavaba la ropa a ciertos mozos de caballos […].

Ella y un hombre moreno, de aquellos que las bestias curaban,


vinieron en conoscimiento […] de que vi que con su venida me-
joraba el comer, fuile queriendo bien […].
1 aceña Molino harinero de agua si-
tuado dentro del cauce de un río. De manera que, continuando la posada y conversación, mi
25 madre vino a darme un negrito muy bonito, el cual yo brincaba

y ayudaba a calentar. Y acuérdome que, estando el negro de mi


padrastro trabajando con el mozuelo, como el niño vía a mi ma-
dre y a mí blancos, y a él no, huía dél, con miedo, para mi madre,
y señalando con el dedo, decía: “¡Madre, coco!”. Respondió él
30 riendo: “¡Hideputa!”.

Yo, aunque bien mochacho, noté aquella palabra de mi her-


manico, y dije entre mí: “¡Cuántos debe de haber en el mundo
que huyen de otros porque no se veen a sí mesmos!”
[…] Y hecha pesquisa, hallose que la mitad por medio de la
35 cebada que para las bestias le daban hurtaba […] y con todo esto
acudía a mi madre para criar a mi hermanico. […] Al triste de
mi padrastro azotaron y pringaron, y a mi madre pusieron pena
por justicia, sobre el acostumbrado centenario; que en casa del
sobredicho Comendador no entrase ni al lastimado Zaide en la
40 suya acogiese.

Por no echar la soga tras el caldero, la triste se esforzó y cumplió


la sentencia; y por evitar peligro y quitarse de malas lenguas, se fue
a servir a los que al presente vivían en el mesón de Solana […].
En este tiempo vino a posar al mesón un ciego, el cual, pares-
45 ciéndole que yo sería para adestralle, me pidió a mi madre, y
ella me encomendó a él […].
—Hijo, ya sé que no te veré más. Procura de ser bueno, y Dios
te guíe. Criado te he y con buen amo te he puesto, válete por ti.

Lengua castellana y literatura 1 29


COMENTARIOS DE TEXTO Lazarillo de Tormes
BAT

1 Responde a las siguientes preguntas sobre el 2 Explica si es posible establecer paralelismos en-
fragmento anterior: tre Amadís de Gaula, el héroe caballeresco que
a ¿Quién es el narrador? ¿Qué momentos de su también nació en un río, y Lázaro. Indica si
vida narra y por qué los narra? crees que hay una voluntad por parte del au-
tor de establecerlos y justifica tu respuesta.
b ¿Cómo son sus padres? ¿A qué clase social
pertenecen?
c ¿Cómo acaba su padre? ¿Por qué?
3 Recuerda que la ironía, el humor y la parodia
son frecuentes en la novela. Observa que tam-
d ¿Por qué le gusta a Lázaro el nuevo bién aparecen en el fragmento anterior. Di
compañero de su madre? ¿Qué tema, qué recursos emplea el autor para mostrar esa
omnipresente en la obra, aparece ya aquí? ironía y provocar la sonrisa.
e ¿Qué siente el hermano de Lázaro cuando ve
el color de su padre? ¿Qué reflexión hace el
pequeño Lázaro?
4 Fíjate en que el folklore y las narraciones tra-
dicionales son fuentes que utiliza el autor. Di
f ¿Por qué acaba bruscamente la relación entre dónde se pueden observar en el texto que has
la madre de Lázaro y el negro Zaide? ¿Cómo leído.
son castigados los amantes?
g ¿Por qué le encomienda Antona Pérez su hijo
al ciego? ¿Para qué lo quiere este último?
5 ¿Cómo logra el autor mantener el decoro en
¿Cómo se muestra Lázaro a esa edad? el fragmento que has leído?

6 Redacta un comentario que analice los rasgos


ideológicos, estructurales y estilísticos del frag-
mento a la luz de lo explicado sobre el Laza-
rillo en esta unidad.

Lazarillo de Tormes. Monumento realizado en bronce,


obra de Agustín Casillas (Castilla-León).

Lengua castellana y literatura 1 30


COMENTARIOS DE TEXTO El Buscón de Quevedo
BAT

EL BUSCÓN
En el primer capítulo, Pablos 1 Yo, señor, soy de Segovia. Mi padre se llamó Clemente Pablo,
nos cuenta sus exagerada- natural del mismo pueblo (Dios le tenga en el cielo). Fue el tal,
mente deshonrosos orígenes, como todos dicen, de oficio barbero, aunque eran tan altos sus
al estilo del Lazarillo, pero pensamientos que se corría de que le llamasen así, diciendo que
empleando un humor negro 5 él era tundidor de mejillas y sastre de barbas. Dicen que era de
y sarcástico mucho más in- muy buena cepa, y, según él bebía, es cosa para creer. Estuvo ca-
tenso que el de la novela anó- sado con Aldonza de San Pedro, hija de Diego de San Juan y nie-
nima. ta de Andrés de San Cristóbal.
Sospechábase en el pueblo que no era cristiana vieja (aun vién-
10 dola con canas y rota), aunque ella, por los nombres y sobre-
nombres de sus pasados, esforzaba que descendía de la Gloria.
Tuvo muy buen parecer, y fue tan celebrada, que, en el tiempo que
ella vivió, todos los copleros de España hacían cosas sobre ella. Pa-
deció grandes trabajos recién casada, y aun después, porque ma-
15 las lenguas daban en decir que mi padre metía el dos de bastos

para sacar el as de oros. Probose que a todos los que hacía la bar-
ba a navaja, mientras le daba con el agua levantándoles la cara pa-
ra el lavatorio, un mi hermano de siete años les sacaba, muy a su
salvo, los tuétanos de las faltriqueras.1 Murió el angelico de unos
20 azotes que le dieron en la cárcel. Sintiolo mucho mi padre (buen
siglo haya), por ser tal, que robaba a todos las voluntades.
Por éstas y otras niñerías estuvo preso, aunque, según a mí
me han dicho después, salió de la cárcel con tanta honra, que le
acompañaron doscientos cardenales, sino que a ninguno se lla-
25 maba eminencia. Las damas diz que salían por verle a las ventanas,

que siempre pareció bien mi padre a pie y a caballo. No lo digo


por vanagloria, que bien saben todos cuán ajeno soy della.
¡Mi madre, pues, no tuvo calamidades! Un día, alabándomela
una vieja que me crió, decía que era tal su agrado, que hechiza-
30 ba a todos cuantos la trataban; sólo diz que le dijo no sé qué de
un cabrón y volar, lo cual la puso cerca de que la diesen plumas
con que lo hiciese en público. […] Tenía su aposento, donde so-
la ella entraba (y algunas veces yo, que, como era chico, podía),
todo rodeado de calaveras, que ella decía que eran para memo-
35 rias de la muerte, y otros, por vituperarla, que para voluntades
de la vida. Su cama estaba armada sobre sogas de ahorcado […].
Hubo grandes diferencias entre mis padres sobre a quién ha-
bía de imitar en el oficio; mas yo, que siempre tuve pensamien-
tos de caballero desde chiquito, nunca me apliqué ni a uno ni a
40 otro […]. Metilos en paz, diciendo que yo quería aprender virtud

resueltamente e ir con mis buenos pensamientos adelante; y así,


que para esto me pusiesen a la escuela, pues sin leer ni escribir
no se podía hacer nada.
Parecioles bien lo que yo decía, aunque lo gruñeron un rato
45 entre los dos. Mi madre se entró dentro y tornó a ocuparse en en-
sartar muelas, y mi padre fue a rapar a uno (así lo dijo él), no sé
1 faltriqueras Bolsillo de las prendas
si la barba o la bolsa […]. Yo me quedé solo, dando gracias a Dios
de vestir. que me hizo hijo de padres tan hábiles y celosos de mi bien.

Lengua castellana y literatura 1 31


COMENTARIOS DE TEXTO El Buscón de Quevedo
BAT

2 Compara los orígenes de Lázaro con los de


Pablos. ¿Crees que Quevedo se inspiró en el
Lazarillo? Explica, si las hay, las diferencias en-
tre las familias de los dos personajes.

3 Comenta si Pablos se muestra tan inocente en


su infancia como Lázaro. Justifica tu respuesta.

4 Recuerda que la exageración, el sarcasmo y la


caricatura son frecuentes en la obra. Di qué
recursos típicos del conceptismo emplea el es-
critor para conseguir sus objetivos.

5 Explica por qué algunos opinan que la obra


de Quevedo es el inicio del fin del camino ini-
ciado por el Lazarillo.

Grabado de la Historia de la vida del Buscón llamado don 6 Redacta un comentario que analice los rasgos
Pablos, exemplo de vagabundos y espejo de tacaños, de ideológicos, estructurales y estilísticos del frag-
una edición de 1895.
mento.

1 Responde a las siguientes preguntas sobre el


fragmento anterior:
a ¿Quién es el narrador? ¿Qué está narrando?
b ¿Cuál es el oficio del padre? ¿A qué se dedica
en realidad?
c ¿Cuál es el oficio de la madre? ¿A qué otra
cosa se dedica, según insinúa Pablos?
d ¿Cuál es el origen familiar de la madre?
¿Es un rasgo negativo más del personaje?
e ¿Cómo es la convivencia entre el padre y la
madre de Lázaro? ¿Por qué?
f ¿Qué futuro profesional quieren los padres
de Pablos para él y su hermano?
g ¿Cómo muere el hermano de Pablo?
¿Por qué lo sintió mucho su padre?
h ¿Cómo sale el padre de Pablos de la cárcel?
¿Qué calamidades sufre su madre a causa de
su oficio?
i ¿Por qué quiere Pablos aprender a leer y
escribir? ¿Qué obsesión tiene ya desde niño?
j ¿Qué nos dice de la calidad moral de Pablos
lo que cuenta sobre su familia?

Lengua castellana y literatura 1 32


COMENTARIOS DE TEXTO El Buscón de Quevedo
BAT

EL BUSCÓN
A. Pablos sirve como criado de don Diego Coronel, hijo de un 1 Lee el primer fragmento (A)
caballero llamado don Alonso Coronel. Don Diego (y con él va y resuelve las actividades:
Pablos) es enviado por su padre a estudiar a la academia del
a ¿Qué dos temas básicos
licenciado Cabra, que tiene por oficio criar a hijos de caballeros
tienen en común el Buscón
y se caracteriza por su avaricia y mezquindad.
y el Lazarillo? ¿Cómo los
1 Entramos el primer domingo después de Cuaresma en poder de trata Quevedo ?
la hambre viva, porque tal lacería no admite encarecimiento. Él
era un clérigo cerbatana, largo sólo en el talle, una cabeza peque- b Analiza el retrato del
ña, pelo bermejo: no hay más que decir para quien sabe el refrán licenciado Cabra y los
5 que dice: ni gato ni perro de aquella color; los ojos avecinados recursos literarios que
en el cogote, que parecía que miraba por cuévanos; tan hundi- emplea el autor.
dos y escuros, que era buen sitio el suyo para tiendas de merca- c ¿Por qué el personaje acaba
deres; la nariz, entre Roma y Francia, porque se le había comido siendo un fantoche y no un
de unas búas1 de resfriado, que aun no fueron de vicio, porque ser humano?
10 cuestan dinero; las barbas, descoloridas de miedo de la boca veci-
na, que, de puro hambre, parecía que amenazaban a comérselas;
los dientes, le faltaban no sé cuántos, y pienso que por holgaza- 2 Lee el segundo fragmento
nes y vagamundos se los habían desterrado; el gaznate, largo (B) y responde:
como de avestruz, con una nuez tan salida, que parecía se iba a
a ¿Cómo presenta Quevedo el
15 buscar de comer, forzada de la necesidad; los brazos, secos; las
mundo estudiantil? ¿Tiene
manos, como un manojo de sarmientos cada una […]. La sota-
intención de crear personajes
na, según decían algunos, era milagrosa, porque no se sabía de
qué color era […]. Parecía, con esto y los cabellos largos y la sota-
redondos (profundos)?
na y el botón, teatino2 lanudo. […] Pues su aposento, aun ara- b ¿Qué recursos literarios
20 ñas no había en él. Conjuraba los ratones, de miedo que no le emplea Quevedo para lograr
royesen algunos mendrugos que guardaba. La cama tenía en el la risa del lector?
suelo y dormía siempre de un lado, por no gastar las sábanas. Al
c ¿Muestra el autor algún tipo
fin, él era un archipobre y protomiseria.
de compasión por Pablos?
¿Hay algún tipo de mensaje
B. Después de casi morir de inanición con el dómine Cabra, don moral tras lo escatológico
Diego y Pablos van a estudiar a Alcalá y el pícaro sufre todo tipo de la situación? Justifica tus
de pesadas bromas que le gastan los estudiantes. En este frag- respuestas.
mento, Pablos tiene que aguantar un auténtico diluvio de gar-
gajos.
1 Yo estaba cubierto el rostro con la capa, y tan blanco, que todos
tiraban a mí, y era de ver, sin duda, cómo tomaban la puntería.
Estaba ya nevado de pies a cabeza; pero un bellacón, viéndome
cubierto y que no tenía en la cara cosa, arrancó hacia mí, dicien-
5 do con gran cólera:
—¡Basta, no le matéis!
Yo, que según me trataban, creí dellos que lo harían, desta-
peme por ver lo que era, y al mismo tiempo el que daba las voces
me enclavó un gargajo entre los dos ojos. Aquí se han de consi-
10 derar mis angustias. Levantó la infernal gente una grita que me
aturdieron, y yo, según lo que echaron sobre mí de sus estóma-
gos, pensé que, por ahorrar de médicos y boticas, aguardaban
nuevos para purgarse. Quisieron tras esto darme de pescozones;
pero no había donde sin llevarse en las manos la mitad del afei- 1 búas Bubas, tumores blandos.
15 te de mi negra capa, ya blanca por mis pecados. 2 teatino Se dice de los clérigos
regulares de San Cayetano.

Lengua castellana y literatura 1 33


COMENTARIOS DE TEXTO El Buscón de Quevedo
BAT

EL BUSCÓN
C. En el camino a Madrid, Pablos se topa con un hidalgo que le cuen- 1 Lee el primer fragmento
ta su calamitosa existencia y le explica cómo es su vida en la corte. (C) y resuelve las
1 Preguntele cómo se llamaba y adónde iba y a qué. Dijo que todos actividades:
los nombres de su padre: Don Toribio Rodríguez Vallejo Gómez a ¿Qué tienen en común el
de Ampuero y Jordán. No se vio jamás nombre tan campanudo, hidalgo del Buscón y el
porque acababa en dan y empezaba en don, como son de badajo. escudero del Lazarillo?
5 Tras esto dijo que iba a la corte, porque un mayorazgo roído ¿Qué los separa?
como él en un pueblo corto olía mal a dos días, y no se podía
b ¿Hay algún tipo de simpatía
sustentar; y que por eso se iba a la patria común, adonde caben
por parte de Quevedo hacia
todos y adonde hay mesas francas para estómagos aventureros
el personaje? ¿Cuál crees
[…]. “Lo primero has de saber que en la corte hay siempre el más
10 necio y el más sabio, el más rico y el más pobre, y los extremos
que es la intención del
de todas las cosas: que disimula los malos y esconde los buenos, autor?
y que en ella hay unos géneros de gentes, como yo, que no se c Comenta los recursos
les conoce raíz ni muebles no otra cepa de la que descienden los retóricos del fragmento y di
tales […]. Somos susto de los banquetes, polilla de los bodego- si el hidalgo aparece en él
15 nes, cáncer de las ollas y convidados por fuerza; sustentámonos como individuo o como
casi del aire y andamos contentos. Somos gente que comemos representante de un grupo
un puerro y representamos un capón […]. Es de ver uno de no- social.
sotros en una casa de juego con el cuidado que sirve y despabi-
la las velas, trae orinales, cómo ayuda a meter naipes y soleniza d ¿Cómo presenta el autor la
20 las cosas del que gana, todo por un triste real de barato […].
vida en la Corte? ¿En qué se
Tenemos de memoria, para lo que toca a vestirnos, toda la rope- está convirtiendo Madrid?
ría vieja; y como en otras partes hay hora señalada para oración,
la tenemos nosotros para remendarnos. […] Y como siempre se
2 Lee el segundo fragmento
gastan tanto las entrepiernas es de ver cómo quitamos cuchilla-
25 das de atrás para poblar lo de adelante; y solemos traer la trase-
(D) y resuelve las
ra tan pacífica por falta de cuchilladas, que se queda en las puras actividades:
bayetas […]”. a ¿Qué actividades propias
del pícaro se atribuye Pablos
en este fragmento?
D. Pablos se finge en la corte caballero y está a punto de casarse
con una linda dama. Pero don Diego lo reconoce y ordena que le b Una vez más, las
den una paliza. aspiraciones de Pablos
1 No aguardó más don Diego, y volviéndose a su casa, encontró acaban en una paliza.
con los dos caballeros del hábito y la cadena, amigos míos, junto ¿El autor sólo busca la risa
a la Puerta del Sol, y contoles lo que pasaba; y díjoles que se apa- o hay un mensaje detrás de
rejasen, y en viéndome a la noche en la calle, que me magulla- ese repetido final?
5 sen los cascos […]. En fin, a las doce, que era a la hora que solía c ¿Qué dos expresiones
hablar con mi niña, llegué a la puerta, y en emparejando, cierra propias de la jerga de
uno de los dos que me aguardaban por don Diego, con un garro- germanía emplea Pablos?
te conmigo, y dame dos palos en las piernas y derríbame al suelo; ¿En qué recursos literarios
y llega el otro, y dame un trasquilón de oreja a oreja, y quítame se basan?
10 la capa y déjanme en el suelo, diciendo:
d Localiza algún ejemplo de
—¡Así pagan los pícaros embustidores mal nacidos!
animalización o cosificación
Comencé a dar gritos y a pedir confisión; y como no sabía lo
en cualquiera de los dos
que era, aunque sospechaba por las palabras que acaso era el gües-
fragmentos.
ped1 de quien me había salido con la traza de la Inquisición, o el
15 carcelero burlado, o mis compañeros huidos, y al fin, yo esperaba
de tantas partes la cuchillada, que no sabía a quién echársela […]. 1 güesped Mesonero o amo de posada.

Lengua castellana y literatura 1 34


COMENTARIOS DE TEXTO El Quijote de Cervantes
BAT

CAPÍTULO XXI
Que trata de la alta aventura y rica ganancia del yelmo de
Mambrino, con otras cosas sucedidas a nuestro invencible
caballero.
1 De allí a poco, descubrió don Quijote un hombre a caba-
llo, que traía en la cabeza una cosa que relumbraba como si
fuera de oro, y aún él apenas le hubo visto, cuando se volvió
a Sancho y le dijo:
5 —Paréceme, Sancho, que no hay refrán que no sea verda-
dero, porque todos son sentencias sacadas de la mesma expe-
riencia, madre de las ciencias todas, especialmente aquel que
dice: “Donde una puerta se cierra, otra se abre”. Dígolo por-
que si anoche nos cerró la ventura la puerta de la que buscá-
10 bamos, engañándonos con los batanes, ahora nos abre de par
en par otra, para otra mejor y más cierta aventura; que si yo
no acertare a entrar por ella, mía será la culpa, sin que la
pueda dar a la poca noticia de batanes ni a la escuridad de la
noche. Digo esto porque, si no me engaño, hacia nosotros
15 viene uno que trae en su cabeza puesto el yelmo2 de Mam-
brino, sobre que yo hice el juramento que sabes.
—Mire vuestra merced bien lo que dice, y mejor lo que
hace —dijo Sancho—; que no querría que fuesen otros bata-
nes que nos acabasen de abatanar y aporrear el sentido.
20 —¡Válate el diablo por hombre! —replicó don Quijote—.
¿Qué va de yelmo a batanes?
—No sé nada —respondió Sancho—; mas, a fe que si yo
pudiera hablar tanto como solía, que quizá diera tales razo-
nes que vuestra merced viera que se engañaba en lo que dice.
25 —¿Cómo me puedo engañar en lo que digo, traidor escru-
puloso? —dijo don Quijote—. Dime, ¿no ves aquel caballero
que hacia nosotros viene, sobre un caballo rucio rodado, que
trae puesto en la cabeza un yelmo de oro?
—Lo que yo veo y columbro —respondió Sancho— no es
30 sino un hombre sobre un asno, pardo como el mío, que trae
sobre la cabeza una cosa que relumbra.
—Pues ése es el yelmo de Mambrino —dijo don Quijote—.
Apártate a una parte y déjame con él a solas; verás cuán sin
hablar palabra, por ahorrar del tiempo, concluyo esta aven-
35 tura, y queda por mío el yelmo que tanto he deseado.
—Yo me tengo en cuidado el apartarme —replicó San-
cho—; mas quiera Dios, torno a decir, que orégano sea, y no
batanes.
—Ya os he dicho, hermano, que no me mentéis, ni por
40 pienso, más eso de los batanes —dijo don Quijote—; que
voto..., y no digo más, que os batanee el alma.
Calló Sancho, con temor que su amo no cumpliese el voto
que le había echado, redondo como una bola. Es, pues, el
caso que el yelmo, y el caballo y caballero que don Quijote
45 veía, era esto: que en aquel contorno había dos lugares, el
uno tan pequeño que ni tenía botica ni barbero, y el otro,
que estaba junto a [él], sí; y así, el barbero del mayor servía

Lengua castellana y literatura 1 35


COMENTARIOS DE TEXTO El Quijote de Cervantes
BAT

al menor, en el cual tuvo necesidad un enfermo de sangrarse y 1 Lee el textos y responde:


otro de hacerse la barba, para lo cual venía el barbero, y traía
50 una bacía3 de azófar;4 y quiso la suerte que, al tiempo que venía, a ¿Qué tienen de irónico
comenzó a llover, y, porque no se le manchase el sombrero, que y paródico el título y el
debía de ser nuevo, se puso la bacía sobre la cabeza; y, como esta- contenido del capítulo?
ba limpia, desde media legua relumbraba. Venía sobre un asno b ¿Cómo expresan don Quijote
pardo, como Sancho dijo, y ésta fue la ocasión que a don Quijote y Sancho sus diferentes
55 le pareció caballo rucio rodado, y caballero, y yelmo de oro; que puntos de vista acerca de lo
todas las cosas que veía, con mucha facilidad las acomodaba a que están viendo? ¿Cuál se
sus desvariadas caballerías y malandantes pensamientos. Y cuan- muestra loco y cuál cuerdo?
do él vio que el pobre caballero llegaba cerca, sin ponerse con él ¿Qué crees que debió pensar
en razones, a todo correr de Rocinante le enristró con el lanzón el barbero de don Quijote?
60 bajo, llevando intención de pasarle de parte a parte; mas cuan-
do a él llegaba, sin detener la furia de su carrera, le dijo: c ¿Cómo justifica don Quijote
—¡Defiéndete, cautiva criatura, o entriégame de tu voluntad que el yelmo parezca una
lo que con tanta razón se me debe! bacía? ¿Convence a su
escudero?
El barbero, que, tan sin pensarlo ni temerlo, vio venir aque-
65 lla fantasma sobre sí, no tuvo otro remedio, para poder guardarse d ¿Qué estilo emplea el
del golpe de la lanza, si no fue el dejarse caer del asno abajo; y narrador? ¿Qué registros
no hubo tocado al suelo, cuando se levantó más ligero que un emplean los dos personajes?
gamo y comenzó a correr por aquel llano, que no le alcanzara el
e ¿Cómo demuestra Cervantes
viento. Dejose la bacía en el suelo, con la cual se contentó don
su dominio del diálogo vivaz
70 Quijote, y dijo que el pagano había andado discreto y que había
y realista?
imitado al castor, el cual, viéndose acosado de los cazadores, se
taraza y arpa con los dientes aquello por lo que él, por distinto f ¿Cómo acentúa el autor
natural, sabe que es perseguido. Mandó a Sancho que alzase el el dinamismo y el relativismo
yelmo, el cual, tomándole en las manos, dijo: tan típicos de la obra?
75 —Por Dios que la bacía es buena y que vale un real de a ocho
como un maravedí.
Y dándosela a su amo, se la puso luego en la cabeza, rodeán-
dola a una parte y a otra, buscándole el encaje; y, como no se le
hallaba, dijo:
80 —Sin duda que el pagano, a cuya medida se forjó primero esta
famosa celada,5 debía de tener grandísima cabeza; y lo peor dello
es que le falta la mitad.
Cuando Sancho oyó llamar a la bacía celada, no pudo tener
la risa; mas vínosele a las mientes la cólera de su amo, y calló en 1 batanes Máquinas hidráulicas,
compuestas de gruesos mazos de
85 la mitad della. madera, movidos por un eje, para
—¿De qué te ríes, Sancho? —dijo don Quijote. golpear, desengrasar y enfurtir los
paños.
—Ríome —respondió él— de considerar la gran cabeza que
2 yelmo Parte de la armadura antigua,
tenía el pagano dueño deste almete, que no semeja si [no] una que resguardaba la cabeza y el rostro,
bacía de barbero pintiparada. y se componía de morrión, visera y
babera. En este caso es el yelmo del
90 —¿Sabes qué imagino, Sancho? Que esta famosa pieza deste
rey moro Mambrino que, en el
encantado yelmo, por algún estraño acidente, debió de venir a Orlando enamorado de Boiardo
manos de quien no supo conocer ni estimar su valor, y, sin saber conquista Reinaldos de Montalbán.
lo que hacía, viéndola de oro purísimo, debió de fundir la otra 3 bacía Era una vasija generalmente de
mitad para aprovecharse del precio, y de la otra mitad hizo ésta, metal brillante, con una escotadura
por donde metía la barba el afeitado.
95 que parece bacía de barbero, como tú dices. Pero, sea lo que
4 azófar Latón.
fuere; que para mí que la conozco no hace al caso su trasmuta-
5 celada Pieza de la armadura que
ción; que yo la aderezaré en el primer lugar donde haya herrero servía para cubrir y defender la
[…]. cabeza.

Lengua castellana y literatura 1 36


COMENTARIOS DE TEXTO El Quijote de Cervantes
BAT

CAPÍTULO LX (2ª PARTE)


De lo que sucedió a don Quijote yendo a Barcelona.
1 […] Don Quijote salió de la venta, informándose primero
cuál era el más derecho camino para ir a Barcelona sin tocar
en Zaragoza: tal era el deseo que tenía de sacar mentiroso a
aquel nuevo historiador que tanto decían que le vituperaba
5 […]. Hízolo así don Quijote, y preguntándole qué le había
sucedido y de qué tenía miedo, le respondió Sancho que
todos aquellos árboles estaban llenos de pies y de piernas
humanas. Tentolos don Quijote y cayó luego en la cuenta de
lo que podía ser, y díjole a Sancho:
10 —No tienes de qué tener miedo, porque estos pies y pier-
nas que tientas y no vees sin duda son de algunos forajidos
y bandoleros que en estos árboles están ahorcados, que por
aquí los suele ahorcar la justicia, cuando los coge, de veinte
en veinte y de treinta en treinta; por donde me doy a enten-
15 der que debo de estar cerca de Barcelona. […]

Ya en esto amanecía, y si los muertos los habían espanta-


do, no menos los atribularon más de cuarenta bandoleros
vivos que de improviso les rodearon, diciéndoles en lengua
catalana que estuviesen quedos y se detuviesen, hasta que lle-
20 gase su capitán.

Hallose don Quijote a pie, su caballo sin freno, su lanza


arrimada a un árbol, y finalmente sin defensa alguna, y, así,
tuvo por bien de cruzar las manos e inclinar la cabeza, guar-
dándose para mejor sazón y coyuntura.
25 Acudieron los bandoleros a espulgar al rucio y a no dejar-
le ninguna cosa de cuantas en las alforjas y la maleta traía, y
avínole bien a Sancho que en una ventrera que tenía ceñida
venían los escudos del duque y los que habían sacado de su
tierra; y, con todo eso, aquella buena gente le escardara y le
30 mirara hasta lo que entre el cuero y la carne tuviera escon-
dido, si no llegara en aquella sazón su capitán, el cual mos-
tró ser de hasta edad de treinta y cuatro años, robusto, más
que de mediana proporción, de mirar grave y color morena.
Venía sobre un poderoso caballo, vestida la acerada cota y
35 con cuatro pistoletes (que en aquella tierra se llaman pedre-
ñales) a los lados. Vio que sus escuderos, que así llaman a los
que andan en aquel ejercicio, iban a despojar a Sancho Panza;
mandoles que no lo hiciesen, y fue luego obedecido, y así se
escapó la ventrera. Admirole ver lanza arrimada al árbol,
40 escudo en el suelo, y a don Quijote armado y pensativo, con
la más triste y melancólica figura que pudiera formar la
misma tristeza. Llegose a él, diciéndole:
—No estéis tan triste, buen hombre, porque no habéis
caído en las manos de algún cruel Osiris,1 sino en las de
45 Roque Guinart,2 que tienen más de compasivas que de rigu-
rosas.
—No es mi tristeza —respondió don Quijote— por haber
Entrada de don Quijote en caído en tu poder, ¡oh valeroso Roque, cuya fama no hay
Barcelona, de Gustave Doré. límites en la tierra que la encierren!, sino por haber sido tal

Lengua castellana y literatura 1 37


COMENTARIOS DE TEXTO El Quijote de Cervantes
BAT

50 mi descuido, que me hayan cogido tus soldados sin el freno, 1 Lee el texto y responde:
estando yo obligado, según la orden de la andante caballería que
profeso, a vivir contino alerta, siendo a todas horas centinela de
a ¿Cómo desmiente y
mí mismo; porque te hago saber, ¡oh gran Roque!, que si me descalifica Cervantes a
hallaran sobre mi caballo, con mi lanza y con mi escudo, no les Avellaneda sin siquiera
55 fuera muy fácil rendirme, porque yo soy don Quijote de la nombrarlo?
Mancha, aquel que de sus hazañas tiene lleno todo el orbe. b ¿Cómo consigue Cervantes
Luego Roque Guinart conoció que la enfermedad de don dar verosimilitud a esta
Quijote tocaba más en locura que en valentía; y aunque algunas aventura y al propio don
veces le había oído nombrar, nunca tuvo por verdad sus hechos, Quijote? ¿Qué diferencia a
60 ni se pudo persuadir a que semejante humor reinase en corazón Roque Guinart del resto de
de hombre, y holgose en estremo de haberle encontrado para personajes de la novela?
tocar de cerca lo que de lejos dél había oído, y, así, le dijo:
c ¿Cómo se muestra el cambio
—Valeroso caballero, no os despechéis ni tengáis a siniestra
gradual hacia la cordura
fortuna esta en que os halláis, que podía ser que en estos tro-
experimentado por don
65 piezos vuestra torcida suerte se enderezase: que el cielo, por
Quijote en la segunda parte
estraños y nunca vistos rodeos, de los hombres no imaginados,
de la novela?
suele levantar los caídos y enriquecer los pobres. […]
Halló Roque Guinart a sus escuderos en la parte donde les d ¿Cómo presenta Cervantes
había ordenado, y a don Quijote entre ellos, sobre Rocinante, al bandolero catalán Roque
70 haciéndoles una plática en que les persuadía dejasen aquel modo Guinart? ¿Tiene algo que ver
de vivir tan peligroso así para el alma como para el cuerpo; pero con el físico, la vestimenta
como los más eran gascones, gente rústica y desbaratada, no les y la fantasía de don Quijote?
entraba bien la plática de don Quijote. Llegado que fue Roque, ¿Qué tienen en común?
preguntó a Sancho Panza si le habían vuelto y restituido las alha-
e ¿Cómo justifica don Quijote
75 jas y preseas3 que los suyos del rucio le habían quitado. Sancho
su falta de reacción ante los
respondió que sí, sino que le faltaban tres tocadores […].
acontecimientos?
Mandóselos volver al punto Roque Guinart y, mandando
poner los suyos en ala, mandó traer allí delante todos los vesti- f ¿Se observa en este
dos, joyas y dineros y todo aquello que desde la última reparti- fragmento el mismo tono
80 ción habían robado; y haciendo brevemente el tanteo, volvien- paródico y humorístico que
do lo no repartible y reduciéndolo a dineros, lo repartió por toda se apreciaba en la aventura
su compañía, con tanta legalidad y prudencia, que no pasó un del yelmo? ¿Por qué?
punto ni defraudó nada de la justicia distributiva. Hecho esto,
con lo cual todos quedaron contentos, satisfechos y pagados. […]
85 Llegaron en esto los escuderos de la presa, trayendo consigo
dos caballeros a caballo y dos peregrinos a pie, y un coche de
mujeres con hasta seis criados, que a pie y a caballo las acompa-
ñaban, con otros dos mozos de mulas que los caballeros traían. 1 Osiris Se refiere a Busiris, rey egipcio
que mataba a los extranjeros que le
Cogiéronlos los escuderos en medio, guardando vencidos y ven- visitaban para ofrecerlos en sacrificio
90 cedores gran silencio, esperando a que el gran Roque Guinart a los dioses.
hablase; el cual preguntó a los caballeros que quién eran y adón- 2 Roque Guinart y el bandolerismo
de iban y qué dinero llevaban. […] catalán Roque Guinart es un
personaje rigurosamente histórico
—Vuesas mercedes, señores capitanes, por cortesía, sean servi- y contemporáneo de Cervantes.
dos de prestarme sesenta escudos, y la señora regenta ochenta, y El bandolerismo catalán fue un
95 luego puédense ir su camino libre y desembarazadamente, con un fenómeno bien conocido en toda
España y era un mal endémico en
salvoconduto que yo les daré, para que si toparen otras de algu- Cataluña contra el cual luchaban
nas escuadras mías que tengo divididas por estos contornos, no con poco éxito los virreyes. Entre sus
les hagan daño, que no es mi intención de agraviar a soldados ni miembros, además de catalanes,
a mujer alguna, especialmente a las que son principales. había un buen número de gascones
(franceses).
100 Infinitas y bien dichas fueron las razones con que los capita- 3 preseas Alhajas, joyas, telas, etc.,
nes agradecieron a Roque su cortesía y liberalidad. preciosas.

Lengua castellana y literatura 1 38


COMENTARIOS DE TEXTO Cervantes: Novelas Ejemplares
BAT

NOVELAS EJEMPLARES: EL CASAMIENTO ENGAÑOSO


En la novela El casamiento 1 —Ya vuesa merced habrá visto —dijo el alférez— dos perros que
engañoso se cuenta cómo al con dos lanternas andan de noche con los hermanos de la Capacha,
salir del hospital donde ha alumbrándoles cuando piden limosna. […]
estado convaleciente, el al- —Sí, he visto —respondió Peralta. […]
férez Campuzano encuentra 5 —Pues lo que ahora diré dellos es razón que la cause, y que sin
a un amigo suyo, el licen- hacerse cruces, ni alegar imposibles ni dificultades, vuesa merced se
ciado Peralta, con quien en- acomode a creerlo, y es que yo oí y casi vi con mis ojos a estos dos
tabla una conversación. perros, que el uno se llama Cipión y el otro Berganza, estar una
noche, que fue la penúltima que acabé de sudar, echados detrás de
10 mi cama en unas esteras viejas, y a la mitad de aquella noche, estan-
do a escuras y desvelado, pensando en mis pasados sucesos y pre-
sentes desgracias, oí hablar allí junto, y estuve con atento oído escu-
chando, por ver si podía venir en conocimiento de los que hablaban
y de lo que hablaban, y a poco rato vine a conocer, por lo que habla-
15 ban, los que hablaban y eran los dos perros Cipión y Berganza.

Apenas acabó de decir esto Campuzano, cuando, levantándose el


Licenciado dijo:
—[…] ¡Por amor de Dios!, señor alférez, que no cuente estos dis-
parates a persona alguna si ya no fuere a quien sea tan su amigo
20 como yo.

—No me tenga vuesa merced por tan ignorante —replicó


Campuzano— que no entienda que si no es por milagro no pueden
hablar los animales […]. Y así, muchas veces, después que los oí, yo
mismo no he querido dar crédito a mí mismo, y he querido tener
25 por cosa soñada lo que realmente estando despierto, con todos mis
cinco sentidos, tales cuales nuestro Señor fue servido de dármelos,
oí, escuché, noté y finalmente escribí, sin faltar palabra, por su con-
cierto de donde se puede tomar indicio bastante que mueva y per-
suada a creer esta verdad que digo. Las cosas de que trataron fueron
30 grandes y diferentes, y más para ser tratadas por varones sabios que
para ser dichas por bocas de perros; así que, pues yo no las pude
inventar de mío, a mi pesar y contra mi opinión vengo a creer que
no soñaba y que los perros hablaban.
—¡Cuerpo de mí! —replicó el Licenciado—. ¡Si se nos ha vuelto el
35 tiempo de Maricastaña,1 cuando hablaban las calabazas, o el de
Isopo cuando departía el gallo con la zorra y unos animales con
otros!
—Uno dellos sería yo, y el mayor —replicó el alférez—, si creyese
que ese tiempo ha vuelto, y aun también lo sería si dejase de creer
40 lo que oí, y lo que vi, y lo que me atreveré a jurar con juramento
que obligue, y aun fuerce, a que lo crea la misma incredulidad. Pero
puesto caso que me haya engañado, y que mi verdad sea sueño, y
el porfiarla disparate, ¿no se holgará vuesa merced, señor Peralta, de
ver escritas en un coloquio las cosas que estos perros, o sean quien
45 fueren, hablaron?

—Como vuesa merced —replicó el Licenciado— no se canse más


en persuadirme que oyó hablar a los perros, de muy buena gana oiré
ese coloquio, que por ser escrito y notado del bueno ingenio del
señor alférez, ya le juzgo por bueno.

Lengua castellana y literatura 1 39


COMENTARIOS DE TEXTO Cervantes: Novelas Ejemplares
BAT

50 —[…] El coloquio traigo en el seno;2 púselo en forma de coloquio


por ahorrar de “dijo Cipión”, “respondió Berganza”, que suele alar-
gar la escritura.
Y en diciendo esto, sacó del pecho un cartapacio y le puso en las
manos del Licenciado. […] Recostose el alférez, abrió el Licenciado el
55 cartapacio, y en el principio vio que estaba puesto este título:

1 Maricastaña e Isopo Se trata de dos referencias a tiempos antiguos. Maricastaña


es un personaje proverbial, símbolo de antigüedad e inocencia; Isopo es una
variante del nombre de Esopo, fabulista griego.
2 seno Falso bolsillo.

“EL COLOQUIO DE LOS PERROS”


Después de mostrar su asom- 1 CIPIÓN: Sea ésta la manera, Berganza amigo: que esta noche me
bro porque pueden hablar, cuentes tu vida y los trances por donde has venido al punto en que
Cipión y Berganza empiezan ahora te hallas, y si mañana en la noche estuviéremos con habla yo
a contarse su vida. te contaré la mía […].
5 BERGANZA: Paréceme que la primera vez que vi el sol fue en Sevilla,
y en su matadero, que está fuera de la Puerta de la Carne; por donde
imaginara (si no fuera por lo que después te diré) que mis padres debie-
ron de ser alanos1 de aquellos que crían los ministros de aquella con-
fusión, a quien llaman jiferos.2 El primero que conocí por amo fue uno
10 llamado Nicolás el Romo, mozo robusto, doblado3 y colérico como son
todos aquellos que ejercitan la jifería

Berganza sirve después a 1 BERGANZA: […] Con todo esto nos quedamos en el hospital aque-
otros amos. Uno de ellos es lla noche y encontrándome la vieja en el corral solo, me dijo: “¿Eres
un titiritero con el que re- tú, hijo Montiel? ¿Eres tú, por ventura, hijo?”. Alcé la cabeza, y
corre diferentes pueblos has- mirela muy de espacio; lo cual visto por ella, con lágrimas en los
ta que llega a Montilla, don- 5 ojos se vino a mí y me echó los brazos al cuello, y si la dejara me

de conoce a la Cañizares. besara en la boca; pero tuve asco y no lo consentí.


CIPIÓN: Bien hiciste; porque no es regalo, sino tormento, el besar,
ni dejar besarse de una vieja.
BERGANZA: Esto que ahora te quiero contar, te lo había de haber
10 dicho al principio de mi cuento, y así excusáramos la admiración
que nos causó el vernos con habla. Porque has de saber que la vieja
me dijo: […] “Tu madre, hijo, se llamó la Montiela, que después de
la Camacha, fue famosa; yo me llamo la Cañizares, y si no tan sabia
como las dos, a lo menos de tan buenos deseos como cualquiera
15 dellas. Verdad es que al ánimo que tu madre tenía de hacer y entrar
en un cerco y encerrarse en él con una legión de demonios no le
hacía ventaja la misma Camacha. Yo fui siempre algo medrosilla;
con conjurar media región me contentaba; pero, con paz sea dicho
de entrambas, en esto de conficionar las unturas con que las brujas
20 nos untamos, a ninguna de las dos diera ventaja, ni la daré a cuan-
tas hoy siguen y guardan nuestras reglas. Que has de saber, hijo, que

Lengua castellana y literatura 1 40


COMENTARIOS DE TEXTO Cervantes: Novelas Ejemplares
BAT

como yo he visto, y veo, que la vida que corre sobre las ligeras alas
del tiempo se acaba, he querido dejar todos los vicios de la hechi-
cería, en que estaba engolfada muchos años había, y sólo me he
25 quedado con la curiosidad de ser bruja, que es un vicio dificultosí-
simo de dejar. Tu madre hizo lo mismo, de muchos vicios se apar-
tó; muchas buenas obras hizo en esta vida; pero al fin murió bruja,
y no murió de enfermedad alguna, sino de dolor de que supo que
la Camacha, su maestra, de envidia que la tuvo porque se le iba
30 subiendo a las barbas en saber tanto como ella, o por otra penden-
zuela de celos, que nunca pude averiguar, estando tu madre preña-
da, y llegándose la hora del parto, fue su comadre la Camacha, la
cual recibió en sus manos lo que tu madre parió, y mostrole que
había parido dos perritos; y así como los vio dijo: “¡Aquí hay mal-
35 dad, aquí hay bellaquería! Pero, hermana Montiela, tu amiga soy;
yo encubriré este parto, y atiende tú a estar sana, y haz cuenta que
esta tu desgracia queda sepultada en el mismo silencio; no te dé
pena alguna este suceso, que ya sabes tú que puedo yo saber que si
no es con Rodríguez, el ganapán4 tu amigo, días ha que no tratas
40 con otro; así que este perruno parto de otra parte viene, y algún mis-
terio contiene”. Admiradas quedamos tu madre y yo, que me hallé
presente a todo, del extraño suceso. La Camacha se fue y se llevó
los cachorros; yo me quedé con tu madre para asistir a su regalo,5
la cual no podía creer lo que le había sucedido. Llegose el fin de la
45 Camacha, y estando en la última hora de su vida llamó a tu madre
y le dijo cómo ella había convertido a sus hijos en perros por cier-
to enojo que con ella tuvo; pero que no tuviese pena, que ellos vol-
verían a su ser cuando menos lo pensasen; mas que no podía ser pri-
mero que ellos por sus mismos ojos viesen lo siguiente:
50 Volverán en su forma verdadera
Cuando vieren con presta diligencia
derribar los soberbios levantados
y alzar a los humildes abatidos
Con poderosa mano para hacello.

55 Esto dijo la Camacha a tu madre al tiempo de su muerte, como


ya te he dicho. Tomolo tu madre por escrito y de memoria, y yo lo
fijé en la mía para si sucediese de poderlo decir a alguno de vosotros;
y para poder conoceros, a todos los perros que veo de tu color los
llamo con el nombre de tu madre, no por pensar que los perros han
60 de saber el nombre, sino por ver si respondían a ser llamados tan
diferentemente como se llaman los otros perros. Y esta tarde, como
te vi hacer tantas cosas, y que te llaman el perro sabio, y también
como alzaste la cabeza a mirarme cuando te llamé en el corral, he
creído que tú eres hijo de la Montiela […].”

1 alano Perro resultado de la unión del dogo y el lebrel. Es corpulento y fuerte.


2 jifero Matarife.
3 doblado Recio y de estatura mediana.
4 ganapán Mozo de cuerda.
5 regalo Cuidado.

Lengua castellana y literatura 1 41


COMENTARIOS DE TEXTO Cervantes: Novelas Ejemplares
BAT

1 Responde a las siguientes preguntas sobre el


fragmento de El casamiento engañoso:
a ¿Qué le cuenta el alférez a su amigo Peralta?
¿Cuál es la reacción de éste último?
b Peralta acepta escuchar la historia de su amigo.
¿Por qué lo hace? ¿Cree realmente que los dos
perros hablaban?
c ¿En qué se demuestra la afición por la
literatura de ambos personajes?

2 Responde a las siguientes preguntas sobre el


fragmento de El coloquio de los perros:
a ¿Qué le cuenta Berganza a Cipión? ¿Por qué?
b ¿En qué contexto social nace Berganza?
¿Quiénes son sus padres?
c ¿Quién es la Cañizares? ¿Qué le explica a
Berganza?
d ¿A qué se dedicaba la madre de Berganza?
e ¿Cuándo volverá a ser humano Berganza?
¿Será ello posible? 5 Recuerda que uno de los rasgos de la narra-
tiva de Cervantes es la intertextualidad. El es-
critor usa los géneros de la época para crear
3 Reflexiona y deduce cómo se imbrican formal-
un relato novedoso que va más allá de lo he-
mente las dos novelas cuyos fragmentos aca-
cho hasta ese momento. Teniendo esto en
bas de leer.
cuenta, ¿qué tiene de novella El coloquio de los
perros? ¿Qué tiene de ejemplar?
4 Contesta:
a ¿Qué semejanzas y diferencias tiene El coloquio 6 Lee esta afirmación:
de los perros con la novela picaresca?
“Estas dos novelas son un ejemplo de crítica
b ¿Qué visión del mundo nos ofrece Cervantes? social, tratado filosófico, tratado sobre el arte
de narrar y relato experimental.”

Analiza las voces narrativas, la cronología y


los diferentes planos del relato.

7 Observa que en las dos novelas se emplea el


diálogo como vehículo narrativo. Indica cuá-
les son las similitudes y las diferencias entre
el diálogo de El casamiento engañoso y el de El
coloquio de los perros. Explica dónde reside la
modernidad de Cervantes.

8 Redacta un comentario que analice los rasgos


ideológicos, estructurales y estilísticos de los
dos fragmentos.

Lengua castellana y literatura 1 42


COMENTARIOS DE TEXTO Luis de Góngora
BAT

1 Analiza la métrica de la
composición y sitúala en
una tradición (culta o
popular).

2 Contesta:
a ¿Qué tópico literario recrea
el poeta? Explica en qué
consiste y compáralo con
otros ejemplos.
b ¿Qué tono preside la
composición, grave o
Baile de campesinos, de Pieter Brueghel. festivo?

QUE SE NOS VA LA PASCUA (LETRILLA)


1 ¡Que se nos va la Pascua, mozas, Yo sé de una buena vieja
que se nos va la Pascua! que fue un tiempo rubia y zarca,4
35 y que al presente le cuesta
Mozuelas las de mi barrio,
loquillas y confiadas, harto caro el ver su cara;
5 mirad no os engañe el tiempo,
porque su bruñida frente
la edad y la confianza. y sus mejillas se hallan
No os dejéis lisonjear más que roquete de obispo
40 encogidas y arrugadas.
de la juventud lozana,
porque de caducas flores ¡Que se nos va la Pascua, mozas,
10 teje el tiempo sus guirnaldas.
que se nos va la Pascua!
¡Que se nos va la Pascua, mozas, Y sé de otra buena vieja
que se nos va la Pascua! que un diente que le quedaba
45 se lo dejó estotro día
Vuelan los ligeros años sepultado en unas natas;
y con presurosas alas y con lágrimas le dice:
1
15 nos roban, como harpías,
“Diente mío de mi alma.
nuestras sabrosas vïandas. yo sé cuándo fuistes perla,
La flor de la maravilla 50 aunque ahora no sois nada”.
esta verdad nos declara, ¡Que se nos va la Pascua, mozas,
porque le hurta la tarde que se nos va la Pascua!
20 lo que le dio la mañana.
¡Que se nos va la Pascua, mozas, Por eso, mozuelas locas, 1 harpía Ser fabuloso, con rostro
antes que la edad avara de mujer y cuerpo de ave de
que se nos va la Pascua! rapiña.
55 el rubio cabello de oro
Mirad que cuando pensáis 2 queda Hora de la noche fijada
convierta en luciente plata, para recogerse, de lo que se
que hacen la señal de la alba quered cuando sois queridas, avisa con la campana.
25 las campanas de la vida,
amad cuando sois amadas; 3 mayores de la marca Se
es la queda,2 y os desarma mirad, bobas, que detrás aplicaba a cualquier cosa que
de vuestro color y lustre, 60 se pinta la ocasión calva.5
excediese lo permitido.
de vuestro donaire y gracia, 4 zarca De ojos azules.
¡Que se nos va la Pascua, mozas,
y quedáis todas perdidas 5 se pinta la ocasión calva La
que se nos va la Pascua! Ocasión se representaba calva,
3
30 por mayores de la marca.
Luis de Góngora aunque con un mechón en la
¡Que se nos va la Pascua, mozas, frente, para indicar que hay que
que se nos va la Pascua! cogerla cuando viene.

Lengua castellana y literatura 1 43


COMENTARIOS DE TEXTO Luis de Góngora
BAT

SONETO 1 Lee el primer soneto con


atención. ¿Qué tópico
1 Mientras por competir con tu cabello,
oro bruñido1 al sol relumbra2 en vano, literario desarrolla el
mientras con menosprecio en medio el llano poema? Explica en qué
mira tu blanca frente el lilio3 bello; consiste y cómo se
estructura su contenido en
5 mientras a cada labio por cogello,4
siguen más ojos que al clavel temprano, el soneto.
y mientras triunfa con desdén5 lozano
del luciente cristal tu gentil cuello; 2 Indica qué rasgos físicos de
goza cuello, cabello, labio y frente, la dama se mencionan y
10 antes que lo que fue en tu edad dorada
con qué elementos se
oro, lilio, clavel, cristal luciente, comparan.
no sólo en plata o vïola6 troncada7
se vuelva, más tú y ello juntamente
en tierra, en humo, en sombra, en polvo, en nada. 3 Observa que en este
Luis de Góngora soneto se utiliza un
procedimiento
1 bruñido Lustrado, brillante. 2 relumbrar Brillar, alumbrar estructurante llamado de
mucho. 3 lilio Lirio. 4 cogello Cogerlo. 5 desdén Indiferencia diseminación/recolección.
que denota menosprecio. 6 vïola Violeta, alhelí. 7 troncada Consiste en repetir juntos
Cortada.
elementos que antes
habían aparecido
aisladamente. Di a qué
términos afecta este
SONETO LXXX recurso, dónde están
1 La dulce boca que a gustar convida diseminados y dónde los
un humor1 entre perlas destilado, recoge. Relaciónalo con la
y a no invidiar aquel licor sagrado estructura del soneto.
que a Júpiter ministra el garzón de Ida,2
5 amantes, no toquéis, si queréis vida,
4 Di qué características de la
porque entre un labio y otro colorado
composición pueden
Amor está, de su veneno armado,
relacionarse con la época
cual entre flor y flor sierpe3 escondida.
en que fue escrito. Puedes
No os engañen las rosas que, a la Aurora,4
5
compararlo con el soneto
10 diréis que aljofaradas y olorosas
XIII de Garcilaso que
se le cayeron del purpúreo seno:
aparece en la ficha de
manzanas son de Tántalo,6 y no rosas,
tópicos.
que después huyen del que incitan ahora;
y sólo del amor queda el veneno.
Luis de Góngora

1 humor Cualquiera de los líquidos del cuerpo. 2 El garzón


de Ida es Ganímedes, que servía el néctar a los dioses.
3 aurora La mitología griega la personificaba como una
muchacha cuyos dedos color de rosa abren las puertas del cielo
al carro del Sol. 4 sierpe Serpiente. 5 aljofaradas Cubiertas
de aljófares, pequeñas perlas irregulares; aquí, cubiertas de
rocío. 6 Tántalo Los dioses lo condenaron a sufrir un eterno
castigo: sufrir sed y hambre teniendo permanentemente agua
y fruta a su alcance, que se escurría de sus manos apenas
lograba tocarla.

Lengua castellana y literatura 1 44


COMENTARIOS DE TEXTO Francisco de Quevedo
BAT

REPRESÉNTASE LA BREVEDAD DE LO QUE SE


VIVE, Y CUÁN NADA PARECE LO QUE SE VIVIÓ
1 ¡Ah de la vida! ¿Nadie me responde?
Aquí de los antaños, que he vivido:
la Fortuna mis tiempos ha mordido,
las Horas mi locura las esconde.
5 ¡Que sin poder saber cómo ni adónde
la salud y la edad se hayan huido!
Falta la vida, asiste lo vivido,
y no hay calamidad que no me ronde.
Ayer se fue, mañana no ha llegado,
10 hoy se está yendo sin parar un punto;
soy un fue y un será y un es cansado.
En el hoy y mañana y ayer junto
pañales y mortaja, y he quedado
presentes sucesiones de difunto.
Óleo de Valdés Leal, representando el tema de la Francisco de Quevedo
muerte.

AMOR CONSTANTE MÁS ALLÁ DE LA MUERTE


1 Cerrar podrá mis ojos la postrera1
sombra, que me llevare2 el blanco día;
y podrá desatar esta alma mía3
hora, a su afán ansioso lisonjera;
5 mas no, de esotra parte en la ribera,4
dejará la memoria, en donde ardía:
nadar sabe mi llama la agua fría,
y perder el respeto a ley severa. 5
Alma a quien todo un Dios6 prisión ha sido,
10 venas que humor a tanto fuego han dado,
médulas7 que han gloriosamente ardido,
su cuerpo dejarán, no su cuidado;8
serán ceniza, mas tendrán sentido;
polvo serán, mas polvo enamorado.
Francisco de Quevedo

1 postrera Última.
2 llevar Cortar, separar violentamente una cosa de otra.
3 La muerte es la liberación del alma, prisionera en el cuerpo;
de ahí que la califique de hora lisonjera (agradable)
a su afán (deseo) ansioso.
4 ribera Al otro lado del río Estigia; en
el mundo de los muertos, según la mitología.
5 ley severa La que hace que los muertos olviden, cuando
beben las aguas del Leteo.
6 Dios Amor.
7 médulas Sustancia grasa que se halla dentro de algunos
huesos.
8 cuidado Preocupación o sentir amoroso.

Lengua castellana y literatura 1 45


COMENTARIOS DE TEXTO Francisco de Quevedo
BAT

Afectos varios de su corazón, 1 Leandro es amante de Hero, una sacerdotisa.


fluctuando en las ondas de los cabellos de Lisi Cada noche el joven cruzaba a nado el estrecho
del Bósforo, que separaba sus respectivas
1 En crespa tempestad del oro undoso ciudades, guiado por una lámpara que Hero
nada golfos de luz ardiente y pura encendía en lo alto de una torre. Una noche el
viento apaga la lámpara y Leandro no alcanza la
mi corazón, sediento de hermosura, costa. A la mañana siguiente las olas depositan su
si el cabello deslazas generoso. cuerpo al pie de la torre de Hero, quien se lanza,
pues no quiere sobrevivir a su amante.
5 Leandro1 en mar de fuego proceloso
2 Ícaro Prisionero en el laberinto fabricado para
su amor ostenta, su vivir apura; encerrar al Minotauro intentó huir con unas alas
Ícaro2 en senda de oro mal segura de plumas y cera. Desoyendo la advertencia de
arde sus alas por morir glorioso. su padre de que no subiera demasiado porque
el sol podía fundir la cera, cayó al mar y pereció.
Con pretensión de fénix3 encendidas 3 fénix Ave mitológica caracterizada por ser única,
10 sus esperanzas, que difuntas lloro, bellísima y sobrevivir a su propia muerte
intenta que su muerte engendre vidas. quemándose y resurgiendo de sus propias
cenizas.
Avaro y rico, y pobre en el tesoro,
4 Midas Este rey tuvo ocasión de pedirle un deseo
el castigo y la hambre imita a Midas,4 a Dioniso: que todo lo que tocase se convirtiese
Tántalo5 en fugitiva fuente de oro. en oro. Pronto comprobó su insensatez al
intentar comer o beber. Hambriento y sediento,
Francisco de Quevedo
suplicó al dios que le retirase el don, y éste
accedió.
5 Tántalo Los dioses lo condenaron a sufrir un
eterno castigo: sufrir sed y hambre teniendo
permanentemente agua y fruta a su alcance, que
se escurría de sus manos apenas lograba tocarla.

ENCARECE LOS AÑOS DE UNA VIEJA NIÑA


1 “Antes que el repelón”1 eso fue antaño:
ras con ras de Caín; o, por lo menos,
la quijada2 que cuentan los morenos
y ella, fueron quijadas en un año.
5 Secula seculorum3 es tamaño
muy niño, y el Diluvio con sus truenos;
ella y la sierpe son ni más ni menos;
y el rey que dicen que rabió,4 es hogaño.5
No había a la estaca preferido el clavo,
10 ni las dueñas usando cenojiles;6
es más vieja que “Préstame un ochavo”.7
Seis mil años le lleva a los candiles;
y si cuentan su edad de cabo a cabo,
puede el guarismo andarse a buscar miles.
Francisco de Quevedo

1 Ser más viejo que el repelón (tirón de pelo) Frase hecha.


2 La quijada con la que Caín mató a Abel.
3 Secula seculorum Expresión litúrgica que significa ‘por
los siglos de los siglos’.
4 Ser del tiempo del rey que rabió Frase hecha que alude a
la antigüedad de algo.
5 hogaño Este año.
6 cenojiles Lazo para atar las medias, ligas.
7 ochavo Moneda.

Lengua castellana y literatura 1 46


COMENTARIOS DE TEXTO Francisco de Quevedo
BAT

REPRESÉNTASE LA BREVEDAD…

1 La expresión ¡ah de la vida! está construida a 10 Observa que el segundo cuarteto empieza con
imitación de ¡ah de la casa! o ¡ah del castillo! la conjunción adversativa mas. Después de
utilizadas para anunciarse a los habitantes aclarar el sentido de cada cuarteto, explica
del recinto mencionado. Quevedo la dota de qué ideas opone esta conjunción.
un sentido existencial. ¿Qué impresión pro-
duce su grito? Relaciónalo con la interrogación
retórica que aparece a continuación. 11 Fíjate cómo, en los tercetos, cada verso del
primero tiene su continuación en el corres-
pondiente del segundo. Explica el sentido de
2 Lee el verso siguiente que reformula expre- cada pareja de versos. ¿Qué idea repiten?
siones como ¡Aquí la guardia! ¿A quién convoca ¿Qué relación tienen con el sentido de los
el poeta en su defensa? cuartetos?

3 ¿Qué lamenta el poeta en los versos 5 y 6? 12 ¿Qué conceptos oponen las numerosas antí-
¿Utiliza un tono grandilocuente o cercano y tesis? Relaciónalo con el tema del soneto.
sencillo? Comenta las antítesis del verso 7.

AFECTOS VARIOS DE SU CORAZÓN


4 ¿Qué palabras se utilizan para nombrar al pa-
sado, el presente y el futuro en el primer ter- 13 Deshaz el hipérbaton que afecta a la primera
ceto? ¿Qué afirma de cada uno de estos mo- estrofa leyendo los versos del 4 al 1.
mentos? ¿Qué pretende demostrar con ello?
14 Localiza las metáforas que forman una alego-
5 Analiza y comenta el uso que hace Quevedo ría en esta estrofa, teniendo en cuenta que los
del verbo ser en el verso 11 para referirse a la únicos términos reales son cabello y corazón,
condición humana y al paso del tiempo. Indi- mientras que los demás tienen sentido figura-
ca en qué momento parece el autor forzar las do. Explica el significado de la alegoría.
posibilidades del idioma y explica por qué.

15 Enumera los diferentes personajes mitológicos


6 ¿A qué aluden por metonimia pañales y mor- con que se identifica el corazón a lo largo del
taja? ¿Qué supone juntarlos? ¿Ha aparecido poema. Explica por qué el autor se identifica
antes esta idea? Exponla. con ellos.

7 Recuerda que el tema de una composición se 16 ¿Qué metáforas utiliza en cada caso para re-
refleja siempre en algún recurso que está a su ferirse al cabello y cómo se adaptan al mito?
servicio. Observa las reiteraciones léxicas y se- ¿Qué rasgos del cabello están presentes en
mánticas más importantes del poema y utilíza- cada una?
las para argumentar la enunciación del tema.

ENCARECE LOS AÑOS DE UNA VIEJA NIÑA


AMOR CONSTANTE MÁS ALLÁ DE LA MUERTE…
17 Lee el soneto e indica cuál es la idea principal
8 Lee el segundo poema y localiza las dos partes y de cuántas maneras distintas la expresa el
fundamentales que señala la puntuación. poeta.

9 Indica el término real de estas metáforas: pos-


18 Di cuál es la figura predominante.
trera sombra; blanco día; desatar el alma llama,
arder, fuego.

Lengua castellana y literatura 1 47


COMENTARIOS DE TEXTO Lope de Vega: Fuenteovejuna
BAT

FUENTEOVEJUNA
Fuenteovejuna es una obra basada FIN DEL ACTO I
en acontecimientos reales ocurri- 1 COMENDADOR: No es malo venir siguiendo
dos en el siglo XV, aunque Lope “re- un corcillo temeroso,
toca” los hechos para que, en lu- y topar tan bella gama.
gar de mercaderes, artesanos o
LAURENCIA: Aquí descansaba un poco
letrados, sean campesinos los que
5 de haber lavado unos paños;
protagonicen la rebelión. La trama
es la siguiente: Fernán Gómez, co- y así, al arroyo me torno,
mendador de la Orden de Calatra- si manda su señoría.
va y señor de Fuenteovejuna, es va- COMENDADOR: Aquesos desdenes toscos
liente con las armas y libertino con afrentan, bella Laurencia,
las mujeres. Fernán persigue a Lau- 10 las gracias que el poderoso
rencia, hija del alcalde, con el fin de cielo te dio, de tal suerte,
forzarla. Frondoso, novio de Lau- que vienes a ser un monstruo.
rencia, se opone al comendador, Mas si otras veces pudiste
que jura vengarse. Finalmente, el huir mi ruego amoroso,
comendador encarcela a Frondoso 15 agora no quiere el campo,
y viola a Laurencia, que insta al amigo secreto y solo;
pueblo a la venganza en defensa que tú sola no has de ser
de su honor. El pueblo mata al co- tan soberbia que tu rostro
mendador y, al ser torturados, to- huyas al señor que tienes,
dos se declaran culpables y el rey
20 teniéndome a mí en tan poco.
perdona a todo el pueblo.
¿No se rindió Sebastiana,
mujer de Pedro Redondo,
con ser casadas entrambas,
y la de Martín del Pozo,
25 habiendo apenas pasado
dos días del desposorio?
LAURENCIA: Ésas, señor, ya tenían
de haber andado con otros
el camino de agradaros;
30 porque también muchos mozos
merecieron sus favores.
Id con Dios tras vueso corzo;
que a no veros con la cruz,
os tuviera por demonio,
35 pues tanto me perseguís.
COMENDADOR: ¡Qué estilo tan enfadoso!
Pongo la ballesta en tierra
[puesto que aquí estamos solos],
y a la práctica de manos
40 reduzco melindres.
LAURENCIA: ¡Cómo!
¿Eso hacéis? ¿Estáis en vos?
Sale FRONDOSO y toma la ballesta.
COMENDADOR: No te defiendas.
45 FRONDOSO: Si tomo
la ballesta, ¡vive el cielo
que no la ponga en el hombro!

Lengua castellana y literatura 1 48


COMENTARIOS DE TEXTO Lope de Vega: Fuenteovejuna
BAT

COMENDADOR: Acaba, ríndete. 1 Lee el fragmento y responde:


LAURENCIA: ¡Cielos, a ¿Cómo caracteriza Lope al
50 ayudadme agora! comendador? ¿Qué rasgos
COMENDADOR: Solos lo definen? ¿Qué lenguaje
estamos; no tengas miedo. emplea con Laurencia?
FRONDOSO: Comendador generoso, b ¿Cómo se muestra
dejad la moza, o creed Laurencia? ¿Qué opinión
55 que de mi agravio y enojo tiene del comendador?
será blanco vuestro pecho,
aunque la Cruz me da asombro. c ¿Qué rasgos definen a
Frondoso? ¿Por qué se
COMENDADOR: ¡Perro villano!...
enfrenta al comendador?
FRONDOSO: No hay perro.
60 Huye, Laurencia. d ¿Está Lope cuestionando el
LAURENCIA: Frondoso, orden social establecido?
mira lo que haces. ¿Por qué?
FRONDOSO: Vete. e ¿Con qué personajes típicos
Vase LAURENCIA. de la comedia nueva se
pueden identificar el
65 COMENDADOR: ¡Oh, mal haya el hombre loco,
comendador, Laurencia y
que se desciñe la espada!
Frondoso?
Que de no espantar medroso
la caza, me la quité. f ¿Qué composición poética
FRONDOSO: Pues, pardiez, señor, si toco emplea Lope a lo largo de
70 la nuez, que os he de apiolar.1 todo el fragmento? ¿Para
qué decía Lope en el Arte
COMENDADOR: Ya es ida. Infame, alevoso,
nuevo que debía emplearse
suelta la ballesta luego.
esta composición?
Suéltala, villano.
FRONDOSO: ¿Cómo?
75 Que me quitaréis la vida. 1 apiolar Matar.
Y advertid que Amor es sordo, 2 correrse Avergonzarse y confundirse.
y que no escucha palabras
el día que está en su trono.
COMENDADOR: Pues, ¿la espalda ha de volver
80 un hombre tan valeroso
a un villano? Tira, infame,
tira, y guárdate; que rompo
las leyes de caballero.
FRONDOSO: Eso no. Yo me conformo
85 con mi estado, y pues me es
guardar la vida forzoso,
con la ballesta me voy.
COMENDADOR: ¡Peligro extraño y notorio!
Mas yo tomaré venganza
90 del agravio y del estorbo.
¡Que no cerrara con él!
¡Vive el cielo, que me corro!2
Lope de Vega

Lengua castellana y literatura 1 49


COMENTARIOS DE TEXTO Calderón de la Barca: La vida es sueño
BAT

LA VIDA ES SUEÑO. ACTO I


La vida es sueño (1635) presenta a 1 SEGISMUNDO: ¡Ay, mísero de mí, ay, infelice!
Segismundo preso en una torre sin Apurar, cielos, pretendo,
que sepa los motivos. El rey Basilio, ya que me tratáis así,
su padre, fue el que lo encerró para qué delito cometí
impedir que se cumpliera el vatici- 5 contra vosotros naciendo;
nio de que su hijo sería un tirano aunque si nací, ya entiendo
para sus súbditos. Un día, Basilio qué delito he cometido:
decide probar si el vaticinio era cier- bastante causa ha tenido
to y libera a su hijo, restituyéndole vuestra justicia y rigor,
su condición de príncipe. Segis- 10 pues el delito mayor
mundo se comporta violentamente del hombre es haber nacido.
con todos y acusa a su padre de qui- Sólo quisiera saber
tarle lo que le pertenecía por naci- para apurar mis desvelos
miento. El rey lo vuelve a encerrar (dejando a una parte, cielos,
en la torre y le hace creer que todo 15 el delito del nacer),
lo que ha vivido ha sido un sueño; ¿qué más os pude ofender,
Segismundo sólo recuerda a Rosau- para castigarme más?
ra como alguien real de quien se ¿No nacieron los demás?
ha enamorado. Entonces, un prín- Pues si los demás nacieron,
cipe extranjero, Astolfo, es nom- 20 ¿qué privilegios tuvieron
brado sucesor al trono y, para evitar que yo no gocé jamás?
que lo obtenga, el pueblo libera a Nace el ave, y con las galas
Segismundo y lo proclama rey. A que le dan belleza suma,
partir de aquí, Segismundo se com- apenas es flor de pluma,
porta sabiamente y perdona a su 25 o ramillete con alas,
padre, renuncia a Rosaura, a la que cuando las etéreas salas
casa con Astolfo, y contrae matri- corta con velocidad,
monio con la infanta Estrella por negándose a la piedad
razones de Estado. del nido que deja en calma;
Los temas del destino y la libertad, 30 ¿y teniendo yo más alma,
el de la realidad y la ficción, el del tengo menos libertad?
sueño y la vigilia y el del libre al- Nace el bruto, y con la piel
bedrío son los más destacados de que dibujan manchas bellas,
este drama filosófico. apenas signo es de estrellas
35 (gracias al docto pincel),
cuando, atrevida y cruel,
la humana necesidad
le enseña a tener crueldad,
monstruo de su laberinto;
40 ¿y yo, con mejor instinto,
tengo menos libertad?
Nace el pez, que no respira,
aborto de ovas y lamas,
y apenas, bajel de escamas
45 sobre las ondas se mira,
cuando a todas partes gira,
midiendo la inmensidad
de tanta capacidad
como le da el centro frío;
50 ¿y yo, con más albedrío,
tengo menos libertad?

Lengua castellana y literatura 1 50


COMENTARIOS DE TEXTO Calderón de la Barca: La vida es sueño
BAT

Nace el arroyo, culebra 1 Contesta estas preguntas


que entre flores se desata, sobre el acto I:
y apenas, sierpe de plata,
55 entre las flores se quiebra,
a ¿Cuál es el estado de ánimo
cuando músico celebra de Segismundo? ¿Qué tema
de los cielos la piedad desarrolla en su monólogo?
que le dan la majestad b ¿En qué fragmentos del
del campo abierto a su huida; texto queda recogido el
60 ¿y teniendo yo más vida tema del libre albedrío?
tengo menos libertad?
c ¿En qué modalidad textual
En llegando a esta pasión,
se puede encuadrar el
un volcán, un Etna hecho,
monólogo de Segismundo?
quisiera sacar del pecho
¿Por qué?
65 pedazos del corazón.
¿Qué ley, justicia o razón d ¿Qué siente Rosaura al oír
negar a los hombres sabe las quejas de Segismundo?
privilegios tan süave e ¿Cuál es la reacción de
excepción tan principal,
Segismundo al saberse
70 que Dios le ha dado a un cristal,
escuchado? ¿Por qué cambia
a un pez, a un bruto y a un ave?
su actitud al ver a Rosaura?
ROSAURA: Temor y piedad en mí
sus razones han causado. f ¿Qué aporta la intervención
SEGISMUNDO: ¿Quién mis voces ha escuchado?
de Clamoroso? ¿Con qué
75 ¿Es Clotaldo? personaje típico de la
comedia nueva podemos
CLAMOROSO: Di que sí.
identificarlo?
ROSAURA: No es sino un triste, ¡ay de mí!,
que en estas bóvedas frías g ¿Qué recursos estilísticos
oyó tus melancolías. propios del culteranismo y
80 SEGISMUNDO: Pues la muerte te daré del conceptismo aparecen en
porque no sepas que sé el texto? ¿Te parece un texto
que sabes flaquezas mías. formalmente complejo?
Sólo porque me has oído, h ¿Qué estrofa emplea el
entre mis membrudos brazos poeta? ¿Para qué momentos
85 te tengo de hacer pedazos. debía usarse esa estrofa,
CLAMOROSO: Yo soy sordo, y no he podido según Lope de Vega?
escucharte.
ROSAURA: Si has nacido
humano, baste el postrarme
90 a tus pies para librarme.
SEGISMUNDO: Tu voz pudo enternecerme,
tu presencia suspenderme,
y tu respeto turbarme.
¿Quién eres? Que aunque yo aquí
95 tan poco del mundo sé,
que cuna y sepulcro fue
esta torre para mí;
y aunque desde que nací
(si esto es nacer) sólo advierto
100 este rústico desierto
donde miserable vivo,
siendo un esqueleto vivo,
siendo un animado muerto.
Calderón de la Barca

Lengua castellana y literatura 1 51


COMENTARIOS DE TEXTO Calderón de la Barca: La vida es sueño
BAT

1 Lee el fragmento con


atención y responde estas
preguntas:
a ¿Qué le dice Basilio a su hijo
cuando éste tiene ya el
poder? ¿Sigue convencido
de la predestinación violenta
de Segismundo?
b ¿Qué ideas sobre el libre
Representación de La vida es sueño en 1981. albedrío y la predestinación
expone Segismundo?
LA VIDA ES SUEÑO. ACTO III (ESCENA XIV) c ¿Qué actitud adopta el hijo
con su padre, el rey?
1 CLOTALDO: ¿Qué quieres? con prudencia y con templanza. ¿Ha cambiado su forma de
BASILIO: Hacer, Clotaldo, Sirva de ejemplo este raro actuar respecto a la que
un remedio que me falta. espectáculo, esta extraña encontrábamos al comienzo
A SEGISMUNDO. admiración, este horror, de la obra? ¿Cómo responde
40 este prodigio; pues nada
5 Si a mí buscándome vas, Basilio?
es más, que llegar a ver
ya estoy, príncipe, a tus plantas, d ¿Qué tipo de composición
con prevenciones tan varias,
sea de ellas blanca alfombra poética emplea Calderón en
rendido a mis pies a mi padre
esta nieve de mis canas. el fragmento?
y atropellado a un monarca.
Pisa mi cerviz y huella
45 Sentencia del cielo fue;
10 mi corona; postra, arrastra
por más que quiso estorbarla
mi decoro y mi respeto,
él, no pudo; ¿y podré yo
toma de mi honor venganza,
que soy menor en las canas,
sírvete de mí cautivo,
en el valor y en la ciencia,
y tras prevenciones tantas,
50 vencerla? Señor, levanta.
15 cumpla el hado su homenaje,
Dame tu mano; que ya
cumpla el cielo su palabra.
que el cielo te desengaña
SEGISMUNDO: […] de que has errado en el modo
de vencerle, humilde aguarda
Mi padre, que está presente,
55 mi cuello a que tú te vengues:
por excusarse a la saña
rendido estoy a tus plantas.
20 de mi condición, me hizo
un bruto, una fiera humana; BASILIO: Hijo, que tan noble acción
de suerte que, cuando yo otra vez en mis entrañas
por mi nobleza gallarda, te engendra, príncipe eres.
por mi sangre generosa, 60 A ti el laurel y la palma
25 por mi condición bizarra se te deben; tú venciste;
hubiera nacido dócil corónente tus hazañas.
y humilde, sólo bastara TODOS: ¡Viva Segismundo, viva!
tal género de vivir,
tal linaje de crianza,
30 a hacer fieras mis costumbres:
[…] la Fortuna no se vence
con injusticia y venganza,
porque antes se incita más;
y así, quien vencer aguarda
35 a su fortuna, ha de ser

Lengua castellana y literatura 1 52


COMENTARIOS DE TEXTO Gustavo Adolfo Bécquer: Rimas
BAT

RIMA II RIMA IV

1 Saeta que voladora 1 No digáis que agotado su tesoro,


cruza arrojada al azar, de asuntos falta, enmudeció la lira.
y que no sabe dónde Podrá no haber poetas; pero siempre
4 temblando se clavará; 4 habrá poesía.

hoja que del árbol seca Mientras las ondas de la luz al beso
arrebata el vendaval, palpiten encendidas:
sin que nadie acierte el surco mientras el sol las desgarradas nubes
8 donde al polvo volverá. 8 de fuego y oro vista;

Gigante ola que el viento mientras el aire en su regazo lleve


riza y empuja en el mar, perfumes y armonías;
y rueda y pasa y se ignora mientras haya en el mundo primavera,
12 qué playa buscando va. 12 ¡habrá poesía!

Luz que en cercos temblorosos Mientras la ciencia a descubrir no alcance


brilla próxima a expirar, las fuentes de la vida,
y que no se sabe de ellos y en el mar o en el cielo haya un abismo
16 cuál el último será. 16 que al cálculo resista;

Eso soy yo, que al acaso mientras la humanidad, siempre avanzando,


cruzo el mundo sin pensar no sepa a do camina;
de dónde vengo ni a dónde mientras haya un misterio para el hombre,
20 mis pasos me llevarán. 20 ¡habrá poesía!
G. A. Bécquer
Mientras se sienta que se ríe el alma,
sin que los labios rían;
mientras se llore sin que el llanto acuda
24 a nublar la pupila;

mientras el corazón y la cabeza


batallando prosigan;
mientras haya esperanzas y recuerdos,
28 ¡habrá poesía!

Mientras haya unos ojos que reflejen


los ojos que los miran;
mientras responda el labio suspirando
32 al labio que suspira;

mientras exista una mujer hermosa


¡habrá poesía!
G. A. Bécquer

Lengua castellana y literatura 1 53


COMENTARIOS DE TEXTO Gustavo Adolfo Bécquer: Rimas
BAT

RIMA VIII RIMA LIII


1 ¡Cuando miro el azul horizonte 1 Volverán las oscuras golondrinas
perderse a lo lejos, en tu balcón sus nidos a colgar,
al través de una gasa de polvo y otra vez con el ala a sus cristales
4 dorado e inquieto, 4 jugando llamarán.

me parece posible arrancarme


Pero aquellas que el vuelo refrenaban
del mísero suelo
tu hermosura y mi dicha a contemplar,
y flotar en la niebla dorada
aquellas que aprendieron nuestros nombres…
8 en átomos leves
8 ésas… ¡no volverán!
cual ella deshecho!
Volverán las tupidas madreselvas
Cuando miro de noche en el fondo
de tu jardín las tapias a escalar
oscuro del cielo
y otra vez a la tarde aún más hermosas
12 las estrellas temblar como ardientes
12 sus flores se abrirán.
pupilas de fuego,
me parece posible a do brillan Pero aquellas cuajadas de rocío
subir en un vuelo, cuyas gotas mirábamos temblar
16 y anegarme en su luz, y con ellas y caer como lágrimas del día…
en lumbre encendido 16 ésas… ¡no volverán!

fundirme en un beso.
Volverán del amor en tus oídos
En el mar de la duda en que bogo las palabras ardientes a sonar,
20 ni aún sé lo que creo; tu corazón de su profundo sueño
sin embargo estas ansias me dicen 20 tal vez despertará.

que yo llevo algo


Pero mudo y absorto y de rodillas
divino aquí dentro.
G. A. Bécquer
como se adora a Dios ante su altar,
como yo te he querido… desengáñate,
24 así… ¡no te querrán!
G. A. Bécquer
RIMA LII

1 Olas gigantes que os rompéis bramando


En las playas desiertas y remotas,
Envuelto entre la sábana de espumas,
4 ¡llevadme con vosotras!

Ráfagas de huracán que arrebatáis


del alto bosque las marchitas hojas,
arrastrado en el ciego torbellino,
8 ¡llevadme con vosotras!

Nubes de tempestad que rompe el rayo


y en fuego ornáis las desprendidas orlas,
arrebatado entre la niebla oscura,
12 ¡llevadme con vosotras!

Llevadme por piedad a donde el vértigo


con la razón me arranque la memoria.
¡Por piedad! ¡Tengo miedo de quedarme
16 con mi dolor a solas!
G. A. Bécquer

Lengua castellana y literatura 1 54


COMENTARIOS DE TEXTO Gustavo Adolfo Bécquer: Rimas
BAT

RIMA II RIMA VIII

1 Haz un análisis métrico de la composición. 10 Describe las vivencias del poeta que evocan
las dos primeras estrofas. Analiza y expón
qué elementos formales y conceptuales tie-
2 Indica qué tienen en común el yo poético y nen en común. Después, responde:
los elementos con los que se identifica.
a ¿Qué las diferencia? ¿Responde a una oposición
entre las vivencias que expresan o pretenden
3 Explica qué recurso estilístico subrayan estas subrayar las analogías? ¿Cuáles son éstas?
analogías. b ¿Qué reflexión sobre dichas vivencias expone
el poeta en la tercera estrofa? ¿Qué tiene de
4 Relaciona el tema de la composición y las imá- romántica su convicción?
genes escogidas para expresarlo con el movi-
miento romántico. RIMA LII

11 ¿A quién se dirige el emisor en esta compo-


RIMA IV sición? Analiza la caracterización de cada
uno de estos elementos mediante la selec-
5 Cita los versos que exponen la tesis que desa-
ción del léxico. ¿Qué recursos estilísticos
rrolla esta rima. ¿Reaparecen a lo largo de la
apoyan tal caracterización?
composición? Observa dónde.

12 ¿Qué les pide? ¿Por qué? Relaciona este deseo


6 ¿Qué recursos estilísticos unifican el resto de
con el movimiento del cual el autor parte.
la composición?

13 ¿En qué medida crees que la imagen que el


7 Observa, dentro del gran bloque que desarro-
poeta ofrece de la naturaleza refleja aspectos
lla la idea principal, la subdivisión temática.
de su yo íntimo? Argumenta tu respuesta.
¿En qué ámbitos de la existencia se encuentra
la poesía, según el autor?
14 ¿Cómo se estructura la composición? Argu-
menta tu propuesta indicando en qué ele-
8 ¿Hay un equilibrio en la importancia atribuí-
mentos formales y conceptuales se basa.
da a cada uno? ¿En qué se evidencia?

15 ¿Qué recursos estilísticos reconoces como


9 Expón brevemente la idea romántica de la
propios del autor en este poema?
poesía, como realidad externa al poeta –y al
poema mismo– e independiente de él, que
esta rima refleja. RIMA LIII

16 Observa que en esta rima, el poeta opone el


tiempo cíclico y repetitivo de la naturaleza a
su irrepetible experiencia amorosa. Analiza
los recursos estructurales que utiliza para ha-
cerlo.

Lengua castellana y literatura 1 55


COMENTARIOS DE TEXTO Benito Pérez Galdós
BAT

MISERICORDIA (1897)
1 Las adversidades se estrellaban ya en el corazón de Benina, Misericordia –ambientada en Ma-
como las vagas olas en el robusto cantil. Rompíanse con drid– narra la historia de Benina,
estruendo, se quebraban, se deshacían en blancas espumas, una criada que hace lo inimagina-
y nada más. Rechazada por la familia que había sustentado ble para ayudar a todo el mundo y
5 en días tristísimos de miseria y dolores sin cuento, no tardó a su señora, doña Francisca, frívola
en rehacerse de la profunda turbación que ingratitud tan y caprichosa y empeñada en vivir
notoria le produjo; su conciencia le dio inefables consuelos: por encima de sus posibilidades, y
miró la vida desde la altura en que su desprecio de la huma- recibe como pago el olvido y el des-
na vanidad la ponía; vio en ridícula pequeñez a los seres precio. Esta novela, una de las me-
10 que la rodeaban, y su espíritu se hizo fuerte y grande. Había
jores del autor, muestra la desilu-
sión de Galdós ante el fracaso de la
alcanzado glorioso triunfo; sentíase victoriosa, después de
clase media como regeneradora del
haber perdido la batalla en el terreno material. Mas las satis-
país y su inclinación por las clases
facciones íntimas de la victoria no la privaron de su don de
bajas, en especial por los personajes
gobierno, y atenta a las cosas materiales, acudió, al poco
redentores, individualistas y de raíz
15 rato de apartarse de Juliana, a resolver lo más urgente en lo
cristiana, como posible solución a
que a la vida corporal de ambos se refería. Era indispensa- los problemas de España. En la obra
ble buscar albergue; después trataría de curar a Mordejai de se usan técnicas realistas y natura-
su sarna o lo que fuese, pues abandonarle en tan lastimoso listas de las décadas anteriores, jun-
estado no lo haría por nada de este mundo, aunque ella se to con nuevos recursos como la
20 viera contagiada del asqueroso mal. Dirigiose con él a Santa tendencia al dialogismo o el juego
Casilda, y hallando desocupado el cuartito que antes ocupó constante entre sueño y realidad.
el moro con la Petra, lo tomó. Felizmente, la borracha se
En cuanto al personaje de Mordejai
había ido con Diega a vivir en la Cava de San Miguel, detrás (o Almudena), un amigo informó a
de la Escalerilla. Instalados en aquel escondrijo, que no care- Galdós de la existencia de un ciego
25 cía de comodidades, lo primero que hizo la anciana al-
andrajoso y de origen sarraceno que
carreña fue traer agua, toda el agua que pudo, y lavarse bien pedía limosna. Galdós fue a verlo y
y jabonarse el cuerpo; costumbre antigua en ella, que siem- lo convenció, invitándolo a beber,
pre que podía practicaba en casa de Doña Francisca. Luego de que le contara la triste historia de
se vistió de limpio. El bienestar que el aseo y la frescura su vida.
30 daban a su cuerpo, se confundía en cierto modo con el des-

canso de su conciencia, en la cual también sentía algo como


absoluta limpieza y frescor confortante.
Dedicose luego al arreglo de la casa, y con el poquito
dinero que tenía hizo su compra, y le preparó a Mordejai
35 una buena comida. Pensaba llevarlo a la consulta al día
siguiente, y así se lo dijo, mostrándose el ciego conforme en
todo con lo que la voluntad de ella quisiese determinar.
Mientras comían, le entretuvo y alentó con esperanzas y
palabras dulces, ofreciéndole ir, como él deseaba, a Jerusalén
40 o un poquito más allá, en cuanto recobrara la salud. Mien-
tras no se le quitara el sarpullido, no había que pensar en
viajes. Se estarían quietos, él en casa, ella saliendo a pedir
sola todos los días para ver de sacar con qué vivir, que segu-
ramente Dios no les dejaría morir de hambre. […] Debe
45 decirse que el ingrato proceder de Doña Paca no desperta-

ba en Nina odio ni mala voluntad, y que la conformidad de


esta con la ingratitud no le quitaba las ganas de ver a la infe-
liz señora, a quien entrañablemente quería, como compa-

Lengua castellana y literatura 1 56


COMENTARIOS DE TEXTO Benito Pérez Galdós
BAT

ñera de amarguras en tantos años. Ansiaba verla, aunque fuese 1 ¿A qué época de la
50 de lejos, y llevada de esta querencia, se llegó a la calle de la producción literaria de
Lechuga para atisbar a distancia discreta si la familia estaba en Galdós pertenece esta
vías de mudanza, o se había mudado ya. ¡Qué a tiempo llegó! novela?
[…] Pero lo que la impresionó vivamente fue ver salir por el
portal a Doña Paca y a Obdulia, con Polidura y Juliana, como
55 si se fueran a la casa nueva, mientras las criadas elegantes se 2 ¿Cómo se muestra en el
quedaban en la antigua, disponiendo la recogida y transporte fragmento el cambio
de las menudencias, y de toda la morralla casera. ideológico experimentado
Turbada y confusa, Nina se escondió en un portal, para ver por el escritor?
sin ser vista. ¡Qué desmejorada encontró a Doña Francisca! Llevaba
60 un vestido nuevo; pero de tan nefanda hechura, como cortado 3 ¿Qué rasgos caracterizan
y cosido de prisa, que parecía la pobre señora vestida de limos-
a Benina? Relaciónalos con
na. Cubría su cabeza con un manto, y Obdulia ostentaba un
el cristianismo.
sombrerote con disformes ringorrangos y plumas. Andaba Doña
Paca lentamente, la vista fija en el suelo, abrumada, melancóli-
65 ca, como si la llevaran entre guardias civiles. […] “¡Pobre seño- 4 ¿Qué visión de la vida
ra mía! —dijo al ciego en cuanto se reunió con él—. La quiero co- ofrece Benina? ¿Es la del
mo hermana, porque juntas hemos pasado muchas penas. Yo Galdós maduro? ¿Por qué?
era todo para ella, y ella todo para mí. Me perdonaba mis faltas,
y yo le perdonaba las suyas... ¡Qué triste va, quizás pensando en
70 lo mal que se ha portado con la Nina! Parece que está peor del 5 ¿Qué rasgos de la novela
reúma, por lo que cojea, y su cara es de no haber comido en cua- naturalista “a la española”
tro días. Yo la traía en palmitas, yo la engañaba con buena som- se observan en el estilo, las
bra, ocultándole nuestra miseria, y poniendo mi cara en ver- situaciones y los personajes
güenza por darle de comer conforme a lo que era su gusto y del fragmento?
75 costumbre... En fin, lo pasado, como dijo el otro, pasó. Vámonos,
Almudena, vámonos de aquí, y quiera Dios que te pongas bue-
6 ¿Se muestra en el texto la
no pronto para tomar el caminito a Jerusalén, que no me asus-
gran admiración que sintió
ta ya por lejos. Andando, andando, hijo, se llega de una parte
Galdós por Cervantes y don
del mundo a otra, y si por un lado sacamos el provecho de to-
Quijote? ¿Cómo?
80 mar el aire y de ver cosas nuevas, por otro sacamos la certeza de
que todo es lo mismo, y que las partes del mundo son, un suponer,
como el mundo en junto; quiere decirse, que en donde quiera que
vivan los hombres, o verbigracia, mujeres, habrá ingratitud, ego-
ísmo, y unos que manden a los otros y les cojan la voluntad. Por
85 lo que debemos hacer lo que nos manda la conciencia, y dejar
que se peleen aquellos por un hueso, como los perros; los otros
por un juguete, como los niños, o estos por mangonear, como
los mayores, y no reñir con nadie, y tomar lo que Dios nos ponga
delante, como los pájaros... Vámonos hacia el Hospital, y no
90 te pongas triste.”
—Mí no triste —dijo Almudena—; estar tigo contentado... tú
saber como Dios cosas tudas, y yo quirier ti como ángela buni-
ta... Y si no quierer tú casar migo, ser tú madra mía, y yo niño
tuyo bunito.
95 —Bueno, hombre; me parece muy bien.
—Y tú com palmera D'sierto granda, bunita; tú com zucena
branca... llirio tú... Mí dicier ti amri: alma mía.
Benito Pérez Galdós

Lengua castellana y literatura 1 57


COMENTARIOS DE TEXTO Leopoldo Alas, Clarín
BAT

LA REGENTA
Al final de la obra, Ana acude a la ca- 1 Y por la vez primera, después de tanto tiempo, sintió dentro de
tedral a reconciliarse con Fermín. Su la cabeza aquel estallido que le parecía siempre voz sobrenatu-
marido ha muerto a consecuencia del ral, sintió en sus entrañas aquella ascensión de la ternura que
duelo con Mesía (amante de Ana), y subía hasta la garganta y producía un amago de estrangulación
la hipócrita sociedad le hace el vacío. 5 deliciosa... Salieron lágrimas a los ojos, y sin pensar más, Ana
Ana busca valientemente el perdón entró en la capilla oscura donde tantas veces el Magistral le había
del Magistral, pero éste, que ama a la hablado del cielo y del amor de las almas. “¿Quién la había traí-
Regenta y ha sido víctima de las ha- do allí? No lo sabía. Iba a confesar con cualquiera y sin saber
bladurías y de los celos, es incapaz de cómo se encontraba a dos pasos del confesionario de aquel her-
concedérselo. La novela finaliza con 10 mano mayor del alma, a quien había calumniado el mundo por
la humillación de Ana a manos de culpa de ella y a quien ella misma, aconsejada por los sofismas
Celedonio, símbolo de la corrupta y de la pasión grosera que la había tenido ciega, había calumnia-
maligna sociedad que ha destruido do también pensando que aquel cariño del sacerdote era amor
las ilusiones de Fermín y Ana, que se brutal, amor como el de Álvaro, el infame, cuando tal vez era
enfrentaron a ella. 15 puro afecto que ella no había comprendido por culpa de la pro-
pia torpeza”.
“Volver a aquella amistad ¿era un sueño? El impulso que la
había arrojado dentro de la capilla ¿era voz de lo alto o capricho
del histerismo, de aquella maldita enfermedad que a veces era lo
20 más íntimo de su deseo y de su pensamiento, ella misma?” Ana
pidió de todo corazón a Dios, a quien claramente creía ver en tal
instante, le pidió que fuera voz Suya aquélla, que el Magistral
fuera el hermano del alma en quien tanto tiempo había creído y
no el solicitante lascivo que le había pintado Mesía el infame. Ana
25 oró, con fervor, como en los días de su piedad exaltada; creyó
posible volver a la fe y al amor de Dios y de la vida, salir del limbo
de aquella somnolencia espiritual que era peor que el infierno;
creyó salvarse cogida a aquella tabla de aquel cajón sagrado que
tantos sueños y dolores suyos sabía...
30 La escasa claridad que llegaba de la nave y los destellos amari-
llentos y misteriosos de la lámpara de la capilla se mezclaban en
el rostro anémico de aquel Jesús del altar, siempre triste y pálido,
que tenía concentrada la vida de estatua en los ojos de cristal que
reflejaban una idea inmóvil, eterna... Cuatro o cinco bultos negros
35 llenaban la capilla. En el confesionario sonaba el cuchicheo de una
beata como rumor de moscas en verano vagando por el aire.
El Magistral estaba en su sitio.
Al entrar la Regenta en la capilla, la reconoció a pesar del
manto. Oía distraído la cháchara de la penitente; miraba a la
40 verja de la entrada, y de pronto aquel perfil conocido y amado
se había presentado como en un sueño. El talle, el contorno de
toda la figura, la genuflexión ante el altar otras señales que sólo
él recordaba y reconocía, le gritaron como una explosión en el
cerebro:
45 “¡Es Ana!”
La beata de la celosía continuaba el rum rum de sus pecados. El
Magistral no la oía, oía los rugidos de su pasión que vociferaban
dentro [...].
Ana había resuelto acercarse también, levantar el velo ante la
50 red de tablillas oblicuas, y a través de aquellos agujeros pedir el
perdón de Dios y el del hermano del alma, y si el perdón no era
posible, pedir la penitencia sin el perdón, pedir a fe perdida o
adormecida o quebrantada, no sabía qué, pedir la fe aunque

Lengua castellana y literatura 1 58


COMENTARIOS DE TEXTO Leopoldo Alas, Clarín
BAT

fuera con el terror del infierno... Quería llorar allí, donde había 1 ¿Cómo destaca Clarín el
55 llorado tantas veces, unas con amargura, otras sonriendo de pla- carácter indomable y la
cer entre las lágrimas; quería encontrar al Magistral de aquellos grandeza moral de Ana en
días en que ella le juzgaba emisario de Dios, quería fe, quería cari- el final de la obra? ¿Es el
dad... y después el castigo de sus pecados, si más castigo merecía personaje espejo del
que aquella oscuridad y aquel sopor del alma... El confesionario
pensamiento del escritor?
60 crujía de cuando en cuando, como si le rechinaran los huesos.
¿Por qué?
El Magistral dio otra absolución y llamó con la mano a otra
beata... […] y al fin quedaron solos la Regenta, sobre la tarima
del altar, y el Provisor dentro del confesionario. 2 ¿Acaba teniendo Fermín
Ya era tarde. La catedral estaba sola. Allí dentro ya empezaba
la misma grandeza que
65 la noche. Ana esperaba sin aliento, resuelta a acudir, la seña que
la llamase a la celosía... Pero el confesonario callaba. La mano no
Ana? ¿Por qué?
aparecía, ya no crujía la madera. Jesús de talla, con los labios páli-
dos entreabiertos y la mirada de cristal fija, parecía dominado
3 ¿Qué crees que sigue
por el espanto, como si esperase una escena trágica inminente.
70 Ana, ante aquel silencio, sintió un terror extraño... uniendo a ambos
Pasaban segundos, algunos minutos muy largos, y la mano no personajes frente a seres
llamaba... ¡La Regenta, que estaba de rodillas, se puso en pie con como Celedonio, por
un valor nervioso que en las grandes crisis le acudía... y se atre- ejemplo?
vió a dar un paso hacia el confesonario. Entonces crujió con fuer-
75 za el cajón sombrío, y brotó de su centro una figura negra, larga.
Ana vio a la luz de la lámpara un rostro pálido, unos ojos que 4 ¿Qué recursos poéticos
pinchaban como fuego, fijos, atónitos como los del Jesús del y estilísticos en general
altar... El Magistral extendió un brazo, dio un paso de asesino emplea Clarín para crear
hacia la Regenta, que, horrorizada, retrocedió hasta tropezar con ese final de la obra la
80 la tarima. Ana quiso gritar, pedir socorro y no pudo. Cayó sen- tensión progresiva que
tada en la madera, abierta la boca, los ojos espantados, las manos conduce al lector al
extendidas hacia el enemigo, que el terror le decía que iba a ase- impactante desenlace?
sinarla.
El Magistral se detuvo, cruzó los brazos sobre el vientre. No
85 podía hablar, ni quería. Temblábale todo el cuerpo; volvió a 5 ¿Dónde podemos
extender los brazos hacia Ana... dio otro paso adelante... y des- encontrar un ejemplo del
pués, clavándose las uñas en el cuello, dio media vuelta, como
estilo indirecto libre, tan
si fuera a caer desplomado, y con piernas débiles y temblonas
empleado por Clarín?
salió de la capilla. […]
90 Ana, vencida por el terror, cayó de bruces sobre el pavimento ¿Por qué lo usa el escritor?
de mármol blanco y negro; cayó sin sentido.
La catedral estaba sola. Las sombras de los pilares y de las
6 ¿En qué espacio finaliza
bóvedas se iban juntando y dejaban el templo en tinieblas.
Celedonio, el acólito afeminado, alto y escuálido, con la sota- la obra? ¿Qué personajes
95 na corta y sucia, venía de capilla en capilla cerrando verjas. Las están presentes?
llaves del manojo sonaban chocando. ¿Demuestra esto la pericia
Llegó a la capilla del Magistral y cerró con estrépito. […] estructural del autor y el
Y entonces redobló la atención y oyó un rumor como un que- uso simbólico de personajes,
jido débil, como un suspiro. animales, luces y espacios
100 Abrió, entró y reconoció a la Regenta, desmayada. por parte del mismo?
Celedonio sintió un deseo miserable, una perversión de la per- ¿Va Clarín más allá del
versión de su lascivia: y por gozar un placer extraño, o por pro-
naturalismo?
bar si lo gozaba, inclinó el rostro asqueroso sobre el de la Regenta
y le besó los labios.
105 Ana volvió a la vida rasgando las nieblas de un delirio que le
causaba náuseas. Había creído sentir sobre la boca el vientre vis-
coso y frío de un sapo.
Leopoldo Alas, Clarín

Lengua castellana y literatura 1 59


COMENTARIOS DE TEXTO Rubén Darío
BAT

LEDA1 LEDA

1 El cisne en la sombra parece de nieve; 1 Haz un análisis métrico


su pico es de ámbar, del alba al trasluz; del poema y relaciónalo
el suave crepúsculo que pasa tan breve con las innovaciones
4 las cándidas alas sonrosa de luz. métricas del Modernismo.
Y luego en las ondas del lago azulado,
después que la aurora perdió su arrebol, 2 ¿Qué motivos propios del
las alas tendidas y el cuello enarcado, Modernismo aparecen en
8 el cisne es de plata bañado de sol. el poema?
Tal es, cuando esponja las plumas de seda,
olímpico pájaro herido de amor, 3 ¿Qué tema, habitual en
y viola en las linfas2 sonoras a Leda, Darío, aborda el texto?
12 buscando su pico los labios en flor.
Suspira la bella desnuda y vencida, 4 Di en cuántas partes se
y en tanto que al aire sus quejas se van, organiza el texto. Justifica
del fondo verdoso de fronda tupida tu división.
16 chispean turbados los ojos de Pan.3
Rubén Darío, Cantos de vida y esperanza (1905) 5 Elabora una lista de
palabras del texto típicas
del Modernismo. Fíjate,
LO FATAL especialmente, en
A René Pérez aquellas que se refieren
1 Dichoso el árbol, que es apenas sensitivo,
a lo sensorial.
y más la piedra dura porque esa ya no siente,
pues no hay dolor más grande que el dolor de ser vivo, LO FATAL

4 ni mayor pesadumbre que la vida consciente. 6 Analiza la métrica de la


Ser y no saber nada, y ser sin rumbo cierto, composición.
y el temor de haber sido y un futuro terror...
Y el espanto seguro de estar mañana muerto, 7 Compara el contenido del
8 y sufrir por la vida y por la sombra y por texto con el anterior. ¿Qué
lo que no conocemos y apenas sospechamos, tema trata Rubén aquí?
y la carne que tienta con sus frescos racimos,
y la tumba que aguarda con sus fúnebres ramos, 8 ¿Qué motivos dominan
12 ¡y no saber adónde vamos, en el poema? ¿Son
ni de dónde venimos!... modernistas?
Rubén Darío, Cantos de vida y esperanza (1905)

9 Haz una lista con las


palabras clave del poema.
Comenta las diferencias
1 Leda Según la mitología griega, fue esposa de Tindáreo y madre con el texto anterior.
de Clitemnestra, Helena, Cástor y Pólux. Zeus se enamoró de ella
y se metamorfoseó en cisne para poseerla.
2 linfas Aguas. 10 Redacta un comentario
3 Pan Dios de los pastores y rebaños, pertenece al cortejo de Dioniso. de uno de los dos textos.
Su cuerpo es mitad hombre y mitad macho cabrío. Pan es un dios
lascivo que persigue a jóvenes y ninfas, y cuando éstas lo rehuyen,
satisface él solo su deseo.

Lengua castellana y literatura 1 60


COMENTARIOS DE TEXTO Antonio Machado
BAT

EL MAÑANA EFÍMERO
A Roberto Castrovido
1 La España de charanga y pandereta, 1 Haz un análisis métrico
cerrado y sacristía, del poema.
devota de Frascuelo y de María,
4 de espíritu burlón y de alma quieta,
2 Expresa si se puede
ha de tener su mármol y su día, relacionar esta composición
su infalible mañana y su poeta. con las preocupaciones
El vano ayer engendrará un mañana de los noventayochistas.
8 vacío y ¡por ventura! pasajero. ¿Utiliza Machado un
Será un joven lechuzo y tarambana, enfoque y un tono similares
un sayón con hechuras de bolero, a estos últimos o, por el
a la moda de Francia realista contrario, adopta uno
propio?
12 un poco al uso de París pagano,

y al estilo de España especialista


en el vicio al alcance de la mano. 3 Indica cuál es el tema
Esa España inferior que ora y bosteza, central del poema. ¿Qué
16 vieja y tahúr, zaragatera y triste; deseos expresa el autor?
esa España inferior que ora y embiste, ¿Se muestra optimista?
¿Por qué?
cuando se digna usar de la cabeza,
aún tendrá luengo parto de varones
20 amantes de sagradas tradiciones 4 Di en cuántas partes se
y de sagradas formas y maneras; organiza el texto. Justifica
florecerán las barbas apostólicas, tu división a partir del
y otras calvas en otras calaveras contenido y los recursos
24 brillarán, venerables y católicas.
empleados por Machado.
El vano ayer engendrará un mañana
vacío y ¡por ventura! pasajero, 5 Elabora una lista de los
la sombra de un lechuzo tarambana, recursos estilísticos más
28 de un sayón con hechuras de bolero; destacados del poema.
el vacuo ayer dará un mañana huero. Presta especial atención
Como la náusea de un borracho ahíto a las personificaciones, la
de vino malo, un rojo sol corona
adjetivación, las metáforas
y las comparaciones.
32 de heces turbias las cumbres de granito;

hay un mañana estomagante escrito


en la tarde pragmática y dulzona. 6 ¿Qué diferencias, en cuanto
Mas otra España nace, al contenido y los recursos,
36 la España del cincel y de la maza, se observan en esta
con esa eterna juventud que se hace composición con respecto
al poema “El limonero
del pasado macizo de la raza.
lánguido suspende”?
Una España implacable y redentora,
40 España que alborea

con un hacha en la mano vengadora,


España de la rabia y de la idea.
Antonio Machado, Campos de Castilla (1907-1917)

Lengua castellana y literatura 1 61


COMENTARIOS DE TEXTO Antonio Machado
BAT

I
EL CRIMEN 1 Haz un análisis métrico
A Federico García Lorca del poema. ¿Qué forma
1 Se le vio, caminando entre fusiles, métrica, habitual en su
por una calle larga, poesía, emplea el autor?
salir al campo frío,
4 aún con estrellas de la madrugada. 2 Indica qué tipo de verso
Mataron a Federico aparece en la primera
cuando la luz asomaba. parte del texto que
El pelotón de verdugos rompe con el esquema
8 no osó mirarle la cara. métrico general. ¿Tiene
Todos cerraron los ojos; relación con el poeta
rezaron: ¡ni Dios te salva! asesinado? ¿Por qué?
Muerto cayó Federico
12 —sangre en la frente y plomo en las entrañas—.
... Que fue en Granada el crimen 3 Di a qué subgénero lírico
sabed —¡pobre Granada!—, en su Granada. pertenece la composición.
Justifica tu respuesta.
II
EL POETA Y LA MUERTE
4 Resume el contenido
Se le vio caminar solo con Ella, de cada una de las tres
16 sin miedo a su guadaña. partes del poema y señala
—Ya el sol en torre y torre, los martillos cómo empieza y acaba
en yunque— yunque y yunque de las fraguas. cada una de ellas.
Hablaba Federico,
20 requebrando a la muerte. Ella escuchaba. 5 Indica si este texto es
«Porque ayer en mi verso, compañera, un simple poema de
sonaba el golpe de tus secas palmas, circunstancias. ¿Por qué?
y diste el hielo a mi cantar, y el filo ¿Qué imagen pretende
24 a mi tragedia de tu hoz de plata, dar Machado del poeta
te cantaré la carne que no tienes, asesinado?
los ojos que te faltan,
tus cabellos que el viento sacudía,
28 los rojos labios donde te besaban... 6 Elabora una lista de los
Hoy como ayer, gitana, muerte mía, recursos estilísticos más
qué bien contigo a solas, destacados en cada una
por estos aires de Granada, ¡mi Granada!» de las tres partes del
poema. Algunos de esos
recursos son un homenaje
III
a la poesía de Lorca;
32 Se le vio caminar... localízalos y coméntalos.
Labrad, amigos,
de piedra y sueño en el Alhambra,
un túmulo al poeta,
36 sobre una fuente donde llore el agua,
y eternamente diga:
el crimen fue en Granada, ¡en su Granada!
Antonio Machado, El crimen fue en Granada.

Lengua castellana y literatura 1 62


COMENTARIOS DE TEXTO Ramón del Valle-Inclán
BAT

SONATA DE INVIERNO
El marqués casi muere por la causa de don Carlos. Bradomín se 1 Indica a qué momento de
enamora de la que resulta ser su hija y sufre un desengaño al ver la vida del marqués de
que sus ilusiones (el carlismo y sus amores) se frustran. En el Bradomín pertenece este
texto hay melancolía y el primer frío de la vejez del don Juan fragmento. ¿Cómo se
feo, católico y sentimental. siente el personaje? ¿Qué
1 Yo acababa de llegar a Estella, donde el Rey tenía su Corte […]. rasgos de su vida son
Comenzaba a sentir algo hasta entonces desconocido en mi vida propios del héroe
alegre y aventurera, una vida llena de riesgos y azares, como la romántico y modernista?
de aquellos segundones hidalgos que se enganchaban en los ter-
5 cios de Italia por buscar lances de amor, de espada y de fortuna.
Yo sentía un acabamiento de todas las ilusiones, un profundo 2 Responde:
desengaño de todas las cosas. Era el primer frío de la vejez, más a ¿Cómo se muestra en el texto
triste que el de la muerte […]. Yo estaba en ese declinar de la la ideología conservadora y
vida, edad propicia para todas las ambiciones y más fuerte que antiburguesa del primer Valle?
10 la juventud misma, cuando se ha renunciado al amor de las
mujeres. ¡Ay, por qué no supe hacerlo! b ¿Qué características propias
Llegué a la Corte de Estella huyendo […]. Las campanas de del idealismo y la evasión
San Juan tocaban anunciando la misa del Rey, y quise oírla toda- modernistas se observan
vía con el polvo del camino en acción de gracias por haber sal- en el decorado y los
15 vado la vida […]. Entre aquellos bultos oscuros, sin contorno ni personajes del relato?
faz, mis ojos sólo pudieron distinguir la figura prócer del Señor, c ¿En qué parte del fragmento
que se destacaba en medio de su séquito, admirable de gallardía se detectan rasgos del
y de nobleza, como un rey de los antiguos tiempos. La arrogan- amoralismo provocador y
cia y brío de su persona, parecían reclamar una rica armadura decadente del Valle
20 cincelada por milanés orfebre, y un palafrén guerrero paramen-
modernista? ¿En qué
tado de malla […]. Todo el día estuvo lloviendo […]. Una luz tris-
consiste la provocación?
te y cenicienta amanecía sobre los montes que rodeaban la ciu-
dad santa del carlismo, donde el rumor de la lluvia en los d ¿Qué características de la
cristales es un rumor familiar […]. Un velo de niebla ondulaba más refinada y elegante
25 en las ráfagas del aire […]. Se oía la canturia monótona de los prosa modernista se observan
niños de una escuela. La tarde lívida daba mayor tristeza al vano en este fragmento?
de la plaza encharcada, desierta, sepulcral […]. Al entrar en la
saleta, donde la Señora y sus damas bordaban escapularios para
los soldados, sentí en el alma una emoción a la vez religiosa y 3 Elabora una lista con las
30 galante. Comprendí entonces todo el ingenuo sentimiento que palabras del texto que se
hay en los libros de caballerías, y aquel culto por la belleza y las refieren a lo sensorial y
lágrimas femeninas que hacía palpitar bajo la cota el corazón de comenta su relación con
Tirante el Blanco […]. el declive vital del
Con la voz vibrante de cólera y embargada de pena, me dijo: personaje donjuanesco.
35 —Ha cometido usted la mayor de sus infamias enamorando a
esa niña.
Confieso que aquella acusación sólo despertó en mi alma un
remordimiento dulce y sentimental. Yo repetí, inclinando la
cabeza:
40 —¡Pobre hija!
Sor Simona retrocedió dando un grito:
—¡Lo sabía usted!
[…]. Sentí terror de mis pecados como si estuviese próximo a
morir [...].
Ramón del Valle-Inclán

Lengua castellana y literatura 1 63


COMENTARIOS DE TEXTO Juan Ramón Jiménez
BAT

ARIAS TRISTES
1 Yo me moriré, y la noche ARIAS TRISTES
triste, serena y callada,
1 Haz un análisis métrico
dormirá el mundo a los rayos
del poema.
4 de su luna solitaria.
Mi cuerpo estará amarillo,
2 ¿Hay rasgos modernistas
y por la abierta ventana
en el poema?
entrará una brisa fresca
8 preguntando por mi alma.
3 ¿Qué obsesiones del
No sé si habrá quien solloce
poeta aparecen ya en
cerca de mi negra caja, esta composición?
o quien me dé un largo beso
12 entre caricias y lágrimas.
4 Di en cuántas partes se
Pero habrá estrellas y flores
organiza el texto.
y suspiros y fragancias,
y amor en las avenidas
16 a la sombra de las ramas.
5 ¿Cómo es el léxico del
poema? Enumera los
Y sonará ese piano
recursos estilísticos más
como en esta noche plácida,
importantes.
y no tendrá quien lo escuche
20 sollozando en la ventana.
LA SOLEDAD SONORA
Juan Ramón Jiménez
6 Haz un análisis métrico
del poema y compáralo
con el anterior.
LA SOLEDAD SONORA
1 Las antiguas arañas melodiosas, temblaban
7 Di qué rasgos
maravillosamente sobre las mustias flores…
modernistas hay en el
sus cristales, heridos por la luna, soñaban texto. ¿Qué temas
4 guirnaldas temblorosas de pálidos colores… obsesivos de Juan Ramón
se muestran en el título
Estaban los balcones abiertos al sur… Era
y el contenido?
una noche inmortal, serena y transparente…
de los campos lejanos, la nueva primavera
8 mandaba, con la brisa, su aliento, dulcemente… 8 Di en qué partes se
organiza el texto.
¡Qué silencio! Las penas ahogaban su ruido
de espectros en las rosas vagas de las alfombras…
el amor no existía… tornaba del olvido
9 Enumera los recursos del
poema y relaciónalos con
12 una ronda infinita de trastornadas sombras…
modernismo intimista.
Todo lo era el jardín… Morían las ciudades…
Las estrellas azules, con la vana indolencia
de haber visto los duelos de todas las edades,
16 coronaban de plata mi nostaljia y mi ausencia…
Juan Ramón Jiménez

Lengua castellana y literatura 1 64


COMENTARIOS DE TEXTO Valle-Inclán: Luces de Bohemia
BAT

LUCES DE BOHEMIA
Luces de bohemia dramatiza la última noche de la vida de Max 1 ¿En qué situación se
Estrella, escritor pobre y ciego. La obra –una parábola grotesca y encuentran Max Estrella
trágica de la España de la época– muestra la peregrinación de Max y su familia? ¿Qué se
y su miserable compañero don Latino por los infiernos de la so- plantea Max?
ciedad.
ESCENA I
2 ¿Qué le llama Claudinita, la
1 MAX: ¡Collet, mal vamos a vernos sin esas cuatro crónicas! ¿Dónde
gano yo veinte duros, Collet? hija de Max, a don Latino?
¿Por qué crees que lo hace?
MADAMA COLLET: Otra puerta se abrirá.
MAX: […] Podemos suicidarnos colectivamente […].
5 MAX: ¿Qué sacaste por los libros, Latino? 3 ¿Por qué no quiere
DON LATINO: ¡Tres pesetas, Max! […] ¡Un robo! Claudinita que su padre
CLAUDINITA: ¡No haberlos dejado! salga a la calle? ¿Qué
DON LATINO: […] Pero aún se puede deshacer el trato […]. crees que presiente la hija
MAX: Habría que devolver el dinero recibido. […] de Max?
10 MADAMA COLLET: Max, no debes salir.

MAX: El aire me refrescará. Aquí hace un calor de horno. […] 4 ¿Qué rasgos esperpénticos
CLAUDINITA: ¡Papá, no salgas! […] se observan en la magnífica
MAX: No quiero tolerar ese robo. ¿A quién le has llevado los libros, acotación de la escena II?
Latino?
15 DON LATINO: A Zaratustra. […]

CLAUDINITA: ¡Golfo!
5 ¿Con quiénes parece
dialogar el librero cuando
ESCENA II entran Max y Latino?
1 La cueva de ZARATUSTRA en el Pretil de los Consejos. […] En la cueva ¿Qué efecto pretende
hacen tertulia el gato, el loro, el can y el librero. ZARATUSTRA –abichado lograr Valle?
y giboso, la cara de tocino rancio y la bufanda de verde serpiente– pro-
mueve, con su caracterización de fantoche, una aguda y dolorosa diso-
5 nancia muy emotiva y muy moderna. Encogido en el roto pelote de una 6 ¿Cómo se muestra don
silla enana guarda la tienda. Latino en esta escena?
ZARATUSTRA: ¡No pienses que no te veo, ladrón! ¿Qué trato le da el autor
EL GATO: ¡Fu! ¡Fu! ¡Fu! a este personaje?
EL CAN: ¡Guau!
10 EL LORO: ¡Viva España!

Están en la puerta MAX ESTRELLA y DON LATINO. […]


MAX: ¡Mal Polonia recibe a un extranjero!
ZARATUSTRA: ¿Qué se ofrece?
MAX: Saludarte y decirte que tus tratos no me convienen.
15 ZARATUSTRA: Yo nada he tratado con usted.

MAX: Cierto. Pero has tratado con mi intendente, Don Latino.


DON LATINO interviene con ese matiz del perro cobarde que da su ladrido
entre las piernas del dueño.
DON LATINO: El maestro no está conforme con la tasa y deshace el
20 trato.
ZARATUSTRA: El trato no puede deshacerse […].
El librero recoge el atadijo y penetra en la lóbrega trastienda, cambiando
una seña con DON LATINO.
Ramón del Valle-Inclán

Lengua castellana y literatura 1 65


COMENTARIOS DE TEXTO Valle-Inclán: Luces de Bohemia
BAT

LUCES DE BOHEMIA
La escena sexta es de las más impresionantes de la obra. En ella,
Valle reduce lo burlesco y parece poner en boca de Max lo que
piensa. Muestra, además, una mirada, como afirma Buero Vallejo,
“súbitamente fraterna”. Valle, contraviniendo el esperpento,
dota de grandeza a los personajes y alude a hechos tan impor-
tantes como la Semana Trágica o la Revolución rusa.
ESCENA VI
1 El calabozo. Sótano mal alumbrado por una candileja. En la som-
bra se mueve el bulto de un hombre (blusa, tapabocas y alparga-
tas). Pasea hablando solo. Repentinamente se abre la puerta. MAX
ESTRELLA, empujado y trompicando, rueda al fondo del calabozo.
5 Se cierra de golpe la puerta. […]

Sale de la tiniebla el bulto del hombre morador del calabozo. Bajo


la luz se le ve esposado, con la cara llena de sangre.
EL PRESO: ¡Buenas noches!
MAX: ¿No estoy solo?
Semana Trágica 10 EL PRESO: Así parece.

MAX: ¿Quién eres, compañero?


EL PRESO: Un paria.
MAX: ¿Catalán?
EL PRESO: De todas partes.
15 MAX: ¡Paria!... Solamente los obreros catalanes aguijan su
rebeldía con ese denigrante epíteto. Paria, en bocas como
la tuya, es una espuela. Pronto llegará vuestra hora.
EL PRESO: Tiene usted luces que no todos tienen. Barcelona ali-
menta una hoguera de odio, soy obrero barcelonés y a
20 orgullo lo tengo.
MAX: ¿Eres anarquista?
EL PRESO: Soy lo que me han hecho las leyes.
MAX: Pertenecemos a la misma Iglesia.
EL PRESO: Usted lleva chalina.
25 MAX: ¡El dogal de la más horrible servidumbre! Me lo arran-
caré, para que hablemos.
EL PRESO: Usted no es proletario.
MAX: Yo soy el dolor de un mal sueño.
EL PRESO: Parece usted hombre de luces. Su hablar es como de
30 otros tiempos.
MAX: Yo soy un poeta ciego.
EL PRESO: ¡No es pequeña desgracia...! En España el trabajo y
la inteligencia siempre se han visto menospreciados. Aquí
todo lo manda el dinero.
35 MAX: Hay que establecer la guillotina eléctrica en la Puerta
del Sol.
EL PRESO: No basta. El ideal revolucionario tiene que ser la des-
trucción de la riqueza, como en Rusia. No es suficiente la
degollación de todos los ricos: Siempre aparecerá un here-
40 dero, y aun cuando se suprima la herencia, no podrá evi-
tarse que los despojados conspiren para recobrarla. Hay

Lengua castellana y literatura 1 66


COMENTARIOS DE TEXTO Valle-Inclán: Luces de Bohemia
BAT

que hacer imposible el orden anterior, y eso sólo se consigue 1 ¿Es comparable la acotación
destruyendo la riqueza. Barcelona industrial tiene que hun- de la escena VI con la de
dirse para renacer de sus escombros con otro concepto de la la II? ¿Qué elementos las
45 propiedad y del trabajo. En Europa, el patrono de más negra
diferencian?
entraña es el catalán, y no digo del mundo porque existen las
Colonias Españolas de América. ¡Barcelona solamente se sal-
va pereciendo! 2 ¿Qué rasgos de la escena VI
MAX: ¡Barcelona es cara a mi corazón! hacen pensar al lector que
50 EL PRESO: ¡Yo también la recuerdo! Valle prescinde del
MAX: Yo le debo los únicos goces en la lobreguez de mi ceguera. esperpento? ¿Por qué crees
Todos los días un patrono muerto, algunas veces, dos... Eso que lo hace?
consuela.
EL PRESO: No cuenta usted los obreros que caen.
55 MAX: Los obreros se reproducen populosamente, de un modo
3 ¿Qué ideología muestran
comparable a las moscas. En cambio los patronos, como los Max y el preso en la
elefantes, como todas las bestias poderosas y prehistóricas, pro- conversación que
crean lentamente. Saulo, hay que difundir por el mundo la mantienen? ¿Cómo
religión nueva. lo demostrarías?
60 EL PRESO: Mi nombre es Mateo.

MAX: Yo te bautizo Saulo. Soy poeta y tengo el derecho al alfa-


beto. Escucha para cuando seas libre, Saulo: Una buena cace-
4 ¿Qué visión de España
ría puede encarecer la piel de patrono catalán por encima del ofrece Valle por boca
marfil de Calcuta. de Max? ¿Qué soluciones
65 EL PRESO: En ello laboramos.
propone?
MAX: Y en último consuelo, aun cabe pensar que exterminando
al proletario, también se extermina al patrón. 5 ¿Qué crees que simboliza
EL PRESO: Acabando con la ciudad, acabaremos con el judaísmo el personaje del preso?
barcelonés.
¿Tiene grandeza el
70 MAX: No me opongo. Barcelona semita sea destruida, como Car-
personaje? ¿Lo presenta
tago y Jerusalén. ¡Alea jacta est! Dame la mano.
Valle desde arriba?
EL PRESO: Estoy esposado. ¿Por qué?
MAX: ¿Eres joven? No puedo verte.
EL PRESO: Soy joven. Treinta años.
75 MAX: ¿De qué te acusan?

EL PRESO: Es cuento largo. Soy tachado de rebelde... No quise


dejar el telar por ir a la guerra y levanté un motín en la fábri-
ca. Me denunció el patrón, cumplí condena, recorrí el mundo
buscando trabajo, y ahora voy por tránsitos, reclamado de no
80 sé qué jueces. Conozco la suerte que me espera: Cuatro tiros
por intento de fuga. Bueno. Si no es más que eso.
MAX: ¿Pues qué temes?
EL PRESO: Que se diviertan dándome tormento.
MAX: ¡Bárbaros!
85 EL PRESO: Hay que conocerlos.

MAX: Canallas. ¡Y esos son los que protestan de la leyenda negra!


EL PRESO: Por siete pesetas, al cruzar un lugar solitario, me saca-
rán la vida los que tienen a su cargo la defensa del pueblo. ¡Y
a esto llaman justicia los ricos canallas!
90 MAX: Los ricos y los pobres, la barbarie ibérica es unánime.

Ramón del Valle-Inclán

Lengua castellana y literatura 1 67


COMENTARIOS DE TEXTO Federico García Lorca
BAT

LA CASA DE BERNARDA ALBA


La casa de Bernarda Alba, subtitu- ACTO I
lada “Drama de mujeres en los pue- 1 MARTIRIO: Nos vamos a cambiar la ropa.
blos de España”, tuvo como origen
BERNARDA: Sí, pero no el pañuelo de la cabeza. (Entra
unos hechos ocurridos en un pue-
blo granadino. La obra comienza Adela.) ¿Y Angustias?
tras la muerte del segundo marido ADELA: (Con retintín.) La he visto asomada a la rendija
de Bernarda. Ésta impone como lu- 5 del portón. Los hombres se acababan de ir.
to a sus hijas una larga reclusión BERNARDA: ¿Y tú a qué fuiste también al portón?
en la casa familiar. En este ambien-
ADELA: Me llegué a ver si habían puesto las gallinas.
te opresivo se desarrolla una acción
que tiene como detonante la figura BERNARDA: ¡Pero el duelo de los hombres habría salido ya!
masculina, nunca presente física- ADELA: (Con intención.) Todavía estaba un grupo parado
mente, de Pepe “el Romano”. Éste 10 por fuera.
pretende a la hermana mayor,
BERNARDA: (Furiosa) ¡Angustias! ¡Angustias!
Angustias, por ser la heredera, pero
se siente atraído por la más joven, ANGUSTIAS: (Entrando) ¿Qué manda usted?
Adela, que lo ama. Por su parte, BERNARDA: ¿Qué mirabas y a quién?
Martirio también ama a Pepe. Fren- ANGUSTIAS: A nadie.
te esto, Bernarda encarna la auto-
ridad represiva, el orgullo de clase
15 BERNARDA: ¿Es decente que una mujer de tu clase vaya
y la voz de la tradición, y las hijas con el anzuelo detrás de un hombre el día de la misa
oscilan entre la sumisión y la re- de su padre? ¡Contesta! ¿A quién mirabas? […]
beldía trágica de Adela. ANGUSTIAS: ¡A nadie!
En esta escena del primer acto se BERNARDA: (Avanzando con el bastón) ¡Suave! […] (Le da)
observan ya los rasgos principales 20 LA PONCIA: (Corriendo) ¡Bernarda, cálmate! (La sujeta)
de los personajes y de sus obsesio-
(Angustias llora)
nes. Además, se presenta un diálo-
go clarificador entre Bernarda y BERNARDA: ¡Fuera de aquí todas! (Salen)
Poncia, la vieja criada de la familia, LA PONCIA: Ella lo ha hecho sin dar alcance a lo que
que siempre aporta saber popular hacía […]. ¡Ya me chocó a mí verla escabullirse
y cierta complacencia en lo sexual, 25 hacia el patio! Luego estuvo detrás de una ventana
por oposición a Bernarda. oyendo la conversación que traían los hombres […].
BERNARDA: ¡A eso vienen a los duelos! ¿De qué hablaban?
LA PONCIA: Hablaban de Paca la Roseta. Anoche ataron
a su marido a un pesebre y a ella se la llevaron a la
30 grupa del caballo hasta lo alto del olivar.
BERNARDA: ¿Y ella?
LA PONCIA: Ella, tan conforme. Dicen que iba con los
pechos fuera y Maximiliano la llevaba cogida como
si tocara la guitarra. ¡Un horror!
35 BERNARDA: ¿Y qué pasó?
LA PONCIA: Lo que tenía que pasar. Volvieron casi de
día. Paca la Roseta traía el pelo suelto y una corona
de flores en la cabeza.
BERNARDA: Es la única mujer mala que tenemos en el
40 pueblo.
LA PONCIA: Porque no es de aquí. Es de muy lejos. Y los
que fueron con ella son también hijos de forasteros.
Los hombres de aquí no son capaces de eso.

Lengua castellana y literatura 1 68


COMENTARIOS DE TEXTO Federico García Lorca
BAT

BERNARDA: No, pero les gusta verlo y comentarlo, y se 1 ¿Qué actitud muestra
45 chupan los dedos de que esto ocurra. Bernarda con sus hijas?
LA PONCIA: Contaban muchas cosas más. ¿Cómo responden ellas?
BERNARDA: (Mirando a un lado y a otro con cierto temor)
¿Cuáles? 2 ¿Qué simbolizan
LA PONCIA: Me da vergüenza referirlas. los hombres en la
50 BERNARDA: Y mi hija las oyó. conversación que
LA PONCIA: ¡Claro! mantienen las mujeres?
BERNARDA: Ésa sale a sus tías; blancas y untosas que
ponían ojos de carnero al piropo de cualquier bar- 3 ¿Qué rasgos propios
berillo. ¡Cuánto hay que sufrir y luchar para hacer de un pueblo pequeño
55 que las personas sean decentes y no tiren al monte y conservador se observan
demasiado! en el diálogo que
LA PONCIA: ¡Es que tus hijas están ya en edad de mere- mantienen Poncia
cer! Demasiada poca guerra te dan. Angustias ya y Bernarda?
debe tener mucho más de los treinta.
60 BERNARDA: Treinta y nueve justos.
4 ¿Cómo se introduce
LA PONCIA: Figúrate. Y no ha tenido nunca novio...
el sexo en la conversación?
BERNARDA: (Furiosa) ¡No, no ha tenido novio ninguna, ¿Qué actitud adoptan
ni les hace falta! Pueden pasarse muy bien. las dos mujeres?
LA PONCIA: No he querido ofenderte.
65 BERNARDA: No hay en cien leguas a la redonda quien se
pueda acercar a ellas. Los hombres de aquí no son
5 ¿Qué ideas tiene Bernarda
de su clase. ¿Es que quieres que las entregue a cual- sobre el honor y la
quier gañán? decencia? ¿Qué rasgos
de su carácter se observan
LA PONCIA: Debías haberte ido a otro pueblo.
en la escena?
70 BERNARDA: Eso, ¡a venderlas!
LA PONCIA: No, Bernarda, a cambiar... ¡Claro que en
otros sitios ellas resultan las pobres! 6 ¿Qué relación existe entre
BERNARDA: ¡Calla esa lengua atormentadora! Bernarda y la Poncia?
LA PONCIA: Contigo no se puede hablar. ¿Tenemos o no
75 tenemos confianza?
BERNARDA: No tenemos. Me sirves y te pago. ¡Nada más!
CRIADA: (Entrando) Ahí está don Arturo, que viene a
arreglar las particiones.
BERNARDA: […] Tú empieza a blanquear el patio. (A la
80 Poncia) Y tú ve guardando en el arca grande toda la
ropa del muerto.
LA PONCIA: Algunas cosas las podríamos dar...
BERNARDA: Nada. ¡Ni un botón! ¡Ni el pañuelo con que
le hemos tapado la cara!
Federico García Lorca

Lengua castellana y literatura 1 69


COMENTARIOS DE TEXTO Federico García Lorca
BAT

LA CASA DE BERNARDA ALBA


En la última escena, Adela desafía la autoridad materna. Martirio, 1 ¿Qué símbolo se quiebra
celosa de Adela, hace creer a su hermana que Bernarda ha matado al final de la obra? ¿Quién
a Pepe. Adela se suicida y acaba la obra. lo quiebra y cómo?
ACTO III
1 ADELA: (Haciéndole frente) ¡Aquí se acabaron las voces de presidio! 2 ¿Cómo se oponen a los
(Adela arrebata un bastón a su madre y lo parte en dos) Esto hago deseos de Adela sus
yo con la vara de la dominadora. No dé usted un paso más. ¡En hermanas? ¿Cuál es el
mí no manda nadie más que Pepe! […]
desencadenante final del
5 ADELA: Yo soy su mujer. (A Angustias) […]
drama?
ADELA: ¡Dios mío! Bernarda: ¡La escopeta! ¿Dónde está la escopeta?
(Sale corriendo) […]
ADELA: ¡Nadie podrá conmigo! (Va a salir) 3 ¿Qué siente Martirio?
ANGUSTIAS: (Sujetándola) De aquí no sales con tu cuerpo en triunfo, ¿Cómo lo expresa? ¿Por
10 ¡ladrona! ¡Deshonra de nuestra casa! qué la maldice la Poncia?
MAGDALENA: ¡Déjala que se vaya! […]
Suena un disparo. 4 ¿Cómo reacciona Bernarda
BERNARDA: (Entrando) Atrévete a buscarlo ahora. al suicidio de Adela?
MARTIRIO: (Entrando) Se acabó Pepe el Romano. ¿Y Martirio?
15 ADELA: ¡Pepe! ¡Dios mío! ¡Pepe! (Sale corriendo)

LA PONCIA: ¿Pero lo habéis matado? 5 ¿Cuáles son las últimas


MARTIRIO: ¡No! ¡Salió corriendo en la jaca! palabras que pronuncia
BERNARDA: No fue culpa mía. Una mujer no sabe apuntar. Bernarda en la obra?
MAGDALENA: ¿Por qué lo has dicho entonces? ¿Qué simbolizan?
20 MARTIRIO: ¡Por ella! Hubiera volcado un río de sangre sobre su ca-
beza.
6 ¿Qué temas y rasgos
LA PONCIA: Maldita.
estilísticos lorquianos se
MAGDALENA: ¡Endemoniada! ven en este fragmento
BERNARDA: Aunque es mejor así. (Se oye como un golpe) ¡Adela! ¡Adela! final?
25 LA PONCIA: (En la puerta) ¡Abre!

BERNARDA: Abre. No creas que los muros defienden de la vergüenza.


CRIADA: (Entrando) ¡Se han levantado los vecinos!
BERNARDA: (En voz baja, como un rugido) ¡Abre, porque echaré abajo
la puerta! […] ¡Adela! […] ¡Trae un martillo! (La Poncia da un
30 empujón y entra. Al entrar da un grito y sale) ¿Qué? […]
LA PONCIA: ¡No entres!
BERNARDA: No. ¡Yo no! Pepe: irás corriendo vivo por lo oscuro de
las alamedas, pero otro día caerás. ¡Descolgarla! ¡Mi hija ha
muerto virgen! Llevadla a su cuarto y vestidla como si fuera
35 doncella. ¡Nadie dirá nada! ¡Ella ha muerto virgen! Avisad que
al amanecer den dos clamores las campanas.
MARTIRIO: Dichosa ella mil veces que lo pudo tener.
BERNARDA: Y no quiero llantos. La muerte hay que mirarla cara a
cara. ¡Silencio! (A otra hija) ¡A callar he dicho! (A otra hija) Las
40 lágrimas cuando estés sola. ¡Nos hundiremos todas en un mar
de luto! Ella, la hija menor de Bernarda Alba, ha muerto virgen.
¿Me habéis oído? ¡Silencio, silencio he dicho! ¡Silencio!
Federico García Lorca

Lengua castellana y literatura 1 70


COMENTARIOS DE TEXTO Rafael Alberti
BAT

RAFAEL ALBERTI
Nace en el Puerto de Santa María en 1902. En 1917 se traslada a
Madrid y se dedica a la pintura y la poesía. En 1925 comparte con
Gerardo Diego el Premio Nacional de Literatura, que obtiene con
Marinero en tierra. En los años treinta se afilia al PCE y lucha con
la República. Después se exilia y en 1977 regresa a España, donde
recibe el Premio Cervantes en 1983 y fallece en 1999.
Alberti admiró la poesía tradicional, la del Siglo de Oro, a Bécquer,
a Juan Ramón... Frutos de su gusto por lo popular son Marinero en
tierra (1924) o El alba del alhelí (1926). El barroquismo, y las van-
guardias, se asientan en Cal y canto (1927), y Sobre los ángeles (1928)
supone la irrupción del Surrealismo en su obra. En los años treinta
cultiva la poesía social (Entre el clavel y la espada, etc.), y en el exilio
su obra muestra la variedad de la producción anterior.
1 Responde a las preguntas
sobre los poemas de
1 Se equivocó la paloma. 1 Si Garcilaso volviera,
Alberti:
Se equivocaba. yo sería su escudero;
que buen caballero era. a ¿Cómo es la métrica de los
Por ira al Norte, fue al Sur. poemas? ¿Qué diferencias
Creyó que el trigo era agua. Mi traje de marinero encuentras entre ellos?
5 Se equivocaba. 5 se trocaría en guerrera ¿Por qué crees que se dan?
ante el brillar de su acero; b ¿A qué épocas de la
Creyó que el mar era el cielo;
que buen caballero era. trayectoria del poeta
que la noche, la mañana.
Se equivocaba.
pertenecen?
¡Qué dulce oírle, guerrero,
al borde de su estribera! c Qué diferencias de contenido
Que las estrellas, rocío; se observan entre ellos?
10 En la mano mi sombrero;
10 que el calor, la nevada.
que buen caballero era. ¿Te parece que ilustran
Se equivocaba. adecuadamente la variedad
Rafael Alberti, Marinero en tierra.
de temas, tonos y estilos
Que tu falda era tu blusa;
propia de Alberti? ¿Por qué?
que tu corazón, su casa.
Se equivocaba.
15 (Ella se durmió en la orilla. 2 En el primer poema,
Tú, en la cumbre de una rama.) comprometido y social,
¿qué símbolos, y recursos
Rafael Alberti, Entre el clavel y la espada.
en general, se emplean
para expresar ese
LOS ÁNGELES COLEGIALES compromiso?
1 Ninguno comprendíamos el secreto nocturno de las pizarras
ni por qué la esfera armilar se exaltaba tan sola cuando la 3 En el segundo poema, ¿por
[mirábamos. qué escoge el poeta ser
Sólo sabíamos que una circunferencia puede no ser redonda el escudero de Garcilaso?
y que un eclipse de luna equivoca a las flores ¿Qué cambio en su estilo
5 y adelanta el reloj de los pájaros. anuncia el autor? ¿Cómo
lo anuncia?
Ninguno comprendíamos nada:
ni por qué nuestros dedos eran de tinta china
y la tarde cerraba compases para al alba abrir libros. 4 En el tercer poema,
Sólo sabíamos que una recta, si quiere, puede ser curva o ¿qué rasgos de estilo
[quebrada y métrica se relacionan
10 y que las estrellas errantes son niños que ignoran la aritmética. con el Surrealismo y las
Rafael Alberti, Sobre los ángeles. vanguardias?

Lengua castellana y literatura 1 71


COMENTARIOS DE TEXTO Luis Cernuda
BAT

SOLILOQUIO DEL FARERO


1 Cómo llenarte, soledad,
Sino contigo misma.
De niño, entre las pobres guaridas de la tierra,
Quieto en ángulo oscuro,
5 Buscaba en tí, encendida guirnalda,
Mis auroras futuras y furtivos nocturnos,
Y en ti los vislumbraba,
Naturales y exactos, también libres y fieles,
A semejanza mía,
10 A semejanza tuya, eterna soledad.
Me perdí luego por la tierra injusta
Como quien busca amigos o ignorados amantes;
Diverso con el mundo,
Fui luz serena y anhelo desbocado,
15 Y en la lluvia sombría o en el sol evidente, LUIS CERNUDA
Quería una verdad que a ti te traicionase, Nace en 1902 en Sevilla, donde
Ólvidando en mi afán cursa estudios universitarios. Vive
Cómo las alas fugitivas su propia nube crean. luego en Madrid, y más tarde es lec-
Y al velarse a mis ojos tor en la universidad de Toulouse.
20 Con nubes sobre nubes de otoño desbordado En la guerra apoya a la República,
La luz de aquellos días en tí misma entrevistos, y en 1938 se exilia. En el exilio en-
Te negué por bien poco; seña en universidades americanas.
Por menudos amores ni ciertos ni fingidos,
Muere en Méjico en 1963. Cernuda
muestra en su poesía el conflicto
Por quietas amistades de sillón y de gesto,
entre sus deseos y la realidad. Desde
25 Por un nombre de reducida cola en un mundo fantasma,
1936, reúne su obra bajo el título
Por los viejos placeres prohibidos,
de La realidad y el deseo. Sus prime-
Como los permitidos nauseabundos,
ros libros, Perfil del aire (1924-27) y
Útiles solamente para el elegante salón susurrado,
Égloga, elegía y oda (1927-28) com-
En bocas de mentira y palabras de hielo. binan la poesía pura y la clásica. De
30 Por ti me encuentro ahora en el eco de la antigua persona 1929 y 1931 son Un río, un amor y
Que yo fui, Los placeres prohibidos, en los que
Que yo mismo manché con aquellas juveniles traiciones; usa técnicas surrealistas. En Donde
Por ti me encuentro ahora, constelados hallazgos, habite el olvido (1932-33) adopta un
Limpios de otro deseo, tono más personal, que se reafirma
35 El sol, mi dios, la noche rumorosa, en Invocaciones (1934-35). En el exi-
La lluvia, intimidad de siempre, lio, en libros como Desolación de la
El bosque y su alentar pagano, quimera, etc., añade a sus temas ha-
El mar, el mar como su nombre hermoso;
bituales el de la patria perdida.
Y sobre todos ellos,
40 Cuerpo oscuro y esbelto,
Te encuentro a ti, tú, soledad tan mía,
Y tú me das fuerza y debilidad
Como el ave cansada los brazos de la piedra.
Acodado al balcón miro insaciable el oleaje,
45 Oigo sus oscuras imprecaciones,
Contemplo sus blancas caricias;
Y erguido desde cuna vigilante
Soy en la noche un diamante que gira advirtiendo a los hombres,

Lengua castellana y literatura 1 72


COMENTARIOS DE TEXTO Luis Cernuda
BAT

Por quienes vivo, aun cuando no los vea;


50 Y así, lejos de ellos,
Ya olvidados sus nombres, los amo en muchedumbres,
Roncas y violentas como el mar, mi morada,
Puras ante la espera de una revolución ardiente
O rendidas y dóciles, como el mar sabe serlo
55 Cuando toca la hora de reposo que su fuerza conquista.
Tú, verdad solitaria,
Transparente pasión, mi soledad de siempre,
Eres inmenso abrazo;
El sol, el mar,
60 La oscuridad, la estepa,
El hombre y su deseo,
la airada muchedumbre,
¿Qué son sino tú misma?
Por ti, mi soledad, los busqué un día;
65 En ti, mi soledad, los amo ahora.
Luis Cernuda, Invocaciones.

SI EL HOMBRE PUDIERA DECIR


1 Si el hombre pudiera decir lo que ama,
Si el hombre pudiera levantar su amor por el cielo
Como una nube en la luz;
Si como muros que se derrumban,
5 Para saludar la verdad erguida en medio,
Pudiera derrumbar su cuerpo, dejando solo la verdad de su amor,
La verdad de sí mismo,
Que no se llama gloria, fortuna o ambición,
Sino amor o deseo,
10 Yo sería aquel que imaginaba;
Aquel que con su lengua, sus ojos y sus manos
Proclama ante los hombres la verdad ignorada,
La verdad de su amor verdadero.

Libertad no conozco sino la libertad de estar preso en alguien


15 Cuyo nombre no puedo oír sin escalofrío;
Alguien por quien me olvido de esta existencia mezquina,
Por quien el día y la noche son para mí lo que quiera,
Y mi cuerpo y espíritu flotan en su cuerpo y espíritu
Como leños perdidos que el mar anega o levanta
20 Libremente, con la libertad del amor,
La única libertad que me exalta,
La única libertad por que muero.

Tú justificas mi existencia:
Si no te conozco, no he vivido;
25 Si muero sin conocerte, no muero, porque no he vivido.
Luis Cernuda, Los placeres prohibidos.

Lengua castellana y literatura 1 73


COMENTARIOS DE TEXTO Luis Cernuda
BAT

SOLILOQUIO DEL FARERO SI EL HOMBRE PUDIERA DECIR

1 Analiza la métrica del poema. 7 Analiza la métrica del poema. ¿Se engarza
en la tradición?

2 ¿Qué tema trata Cernuda en este poema?


¿Tiene relación con el título? 8 Di qué es lo que expresa el emisor del poema.
¿Tiene relación con los temas recurrentes del
autor? ¿Te parece una defensa de su dignidad
3 Enumera y comenta los recursos estilíticos e identidad?
fundamentales que usa el poeta para dirigirse
a esa “compañera” de toda la vida.
9 ¿Qué crees que hace de Cernuda uno de los
mejores poetas tratando el tema amoroso?
4 ¿Ha conseguido el autor dejar atrás a su Justifica tus opiniones con ejemplos del tex-
“compañera”? ¿Por qué? Comenta algunos to.
de los recursos estilísticos que expresan lo
que le ocurre a Cernuda.
10 Di en cuántas partes se organiza el poema.
Justifica tu división.
5 Basándote en el poema, explica por qué se
habla de sustrato romántico en la obra de Cer-
nuda. 11 Enumera y comenta los recursos estilísticos
más destacados. Céntrate especialmente en
aquellos que enfatizan la oposición silencio/
6 El autor expresa también su interés y amor verdad y libertad/prisión.
por la humanidad, ¿En qué versos del poema
lo hace? ¿Qué recursos emplea?

Lengua castellana y literatura 1 74


COMENTARIOS DE TEXTO Vicente Aleixandre
BAT

SE QUERÍAN
1 Se querían.
Sufrían por la luz, labios azules en la madrugada,
Labios saliendo de la noche dura,
Labios partidos, sangre, ¿sangre dónde?
5 Se querían en un lecho navío, mitad noche, mitad luz.

Se querían como las flores a las espinas hondas,


A esa amorosa gema del amarillo nuevo,
Cuando los rostros giran melancólicamente,
Giralunas que brillan recibiendo aquel beso.

10 Se querían de noche, cuando los perros hondos


Laten bajo la tierra y los valles se estiran
Como lomos arcaicos que se sienten repasados: caricia,
Seda, mano, luna que llega y toca. VICENTE ALEIXANDRE
Se querían de amor entre la madrugada, Nace en Sevilla en 1898. A partir
15 Entre las duras piedras cerradas de la noche,
de 1909 vive en Madrid, donde es-
tudia Derecho y Comercio, pero se
Duras como los cuerpos helados por las horas,
acaba dedicando por entero a la
Duras como los besos de diente a diente solo. poesía. Al acabar la Guerra Civil,
Se querían de día, playa que va creciendo, permanece en España y se convierte
en un gran referente para los nue-
Ondas que por los pies acarician los muslos,
vos poetas. En 1949 es elegido
20 Cuerpos que se levantan de la tierra y flotando...
miembro de la RAE y en 1977 se le
Se querían de día, sobre el mar, bajo el cielo. otorga el Premio Nobel. Muere en
1984. Darío o Juan Ramón marcan
Mediodía perfecto, se querían tan íntimos,
sus inicios, pero el Surrealismo es
Mar altísimo y joven, intimidad extensa,
decisivo en su obra. Su estilo es her-
Soledad de lo vivo, horizontes remotos mético y sus metáforas son gran-
25 Ligados como cuerpos en soledad cantando. diosas y proféticas. Suele usar el ver-
so libre o versículo amplio. En su
Amando. Se querían como la luna lúcida,
trayectoria destacan Ámbito (1924-
Como ese mar redondo que se aplica a ese rostro, 1927), Espadas como labios (1930-
Dulce eclipse de agua, mejilla oscurecida, 1931), La destrucción o el amor
Donde los peces rojos van y vienen sin música. (1932-33) y Sombra del paraíso
(1939-43). Después es solidario con
30 Día, noche, ponientes, madrugadas, espacios, el hombre que sufre en Historia del
Ondas nuevas, antiguas, fugitivas, perpetuas, corazón (1945-53). En sus últimos
Mar o tierra, navío, lecho, pluma, cristal, años escribe Poemas de la consuma-
Metal, música, labio, silencio, vegetal, ción (1968), entre otros libros.
Mundo, quietud, su forma. Se querían, sabedlo.
Vicente Aleixandre, La destrucción o el amor.

Lengua castellana y literatura 1 75


COMENTARIOS DE TEXTO Vicente Aleixandre
BAT

EL CUERPO Y EL ALMA
1 Pero es más triste todavía, mucho más triste.
Triste como la rama que deja caer su fruto para nadie.
Más triste, más. Como ese vaho
que de la tierra exhala después la pulpa muerta.
5 Como esa mano que del cuerpo tendido
se eleva y quiere solamente acariciar las luces,
la sonrisa doliente, la noche aterciopelada y muda.
Luz de la noche sobre el cuerpo tendido sin alma.
Alma fuera, alma fuera del cuerpo, planeando
10 tan delicadamente sobre la triste forma abandonada.
Alma de niebla dulce, suspendida
sobre su ayer amante, cuerpo inerme
que pálido se enfría con las nocturnas horas
y queda quieto, solo, dulcemente vacío.

15 Alma de amor que vela y se separa


vacilando, y al fin se aleja tiernamente fría.
Vicente Aleixandre, Sombra del paraíso.

MANO ENTREGADA
1 Pero otro día toco tu mano. Mano tibia.
Tu delicada mano silente. A veces cierro
mis ojos y toco leve tu mano, leve toque
que comprueba su forma, que tienta
5 su estructura, sintiendo bajo la piel alada el duro hueso
insobornable, el triste hueso adonde no llega nunca
el amor. Oh carne dulce que sí se empapa del amor hermoso.
Es por la piel secreta, secretamente abierta, invisiblemente entreabierta,
por donde el calor tibio propaga su voz, su afán dulce;
10 por donde mi voz penetra hasta tus venas tibias,
para rodar por ellas en tu escondida sangre,
como otra sangre que sonara oscura, que dulcemente oscura te besara
por dentro, recorriendo despacio como sonido puro
ese cuerpo que ahora resuena mío, mío poblado de mis voces profundas,
15 oh resonado cuerpo de mi amor, oh poseído cuerpo, oh cuerpo
sólo sonido de mi voz poseyéndole.
Por eso, cuando acaricio tu mano sé que sólo el hueso rehúsa
mi amor –el nunca incandescente hueso del hombre–.
Y que una zona triste de tu ser se rehúsa,
20 mientras tu carne entera llega un instante lúcido
en que total flamea, por virtud de ese leve contacto de tu mano,
de tu porosa mano suavísima que gime,
tu delicada mano silente, por donde entro
despacio, despacísimo, secretamente en tu vida,
25 hasta tus venas hondas totales donde bogo,
donde te pueblo y canto completo entre tu carne.
Vicente Aleixandre, Historia del corazón.

Lengua castellana y literatura 1 76


COMENTARIOS DE TEXTO Vicente Aleixandre
BAT

SE QUERÍAN MANO ENTREGADA

1 La métrica del poema es la habitual en Alei- 8 ¿Es la métrica de este poema típica en Alei-
xandre, analízala. xandre? ¿Por qué?

2 ¿Qué clase de amor retrata el autor en el 9 Este poema pertenece a una época más tardía
texto? ¿Con qué metáforas y recursos expresa de su obra, ¿qué idea del amor expone aquí
ese amor total? Señala ejemplos de la pre- el autor? ¿Tiene límites? ¿Es más “humano”
sencia surrealista en el poema que antes?

3 ¿Qué recurso une y da ritmo al poema? Ana- 10 ¿Con qué metáforas se expresa el poder del
liza estilísticamente la última estrofa del tex- amor, y también sus límites?
to.

11 Se ha destacado la lentitud intencionada del


EL CUERPO Y EL ALMA poema para expresar la morosidad del con-
4 ¿Es igual el amor retratado en este poema tacto amoroso. ¿Cómo logra Aleixandre ese
al del anterior? ¿Por qué? efecto de demora en el texto?

5 ¿Qué concepto aparece asociado aquí al


amor?

6 ¿Qué otro recurso, además de las bellísimas


metáforas, destaca en el poema?

7 Busca en el texto las palabras relacionadas


con los sentidos y comenta el uso que el
poeta hace de ellas.

Lengua castellana y literatura 1 77


COMENTARIOS DE TEXTO Blas de Otero
BAT

A LA INMENSA MAYORÍA
1 Aquí tenéis, en canto y alma, al hombre
aquel que amó, vivió, murió por dentro
y un buen día bajó a la calle: entonces
4 comprendió: y rompió todos sus versos.

Así es, así fue. Salió una noche


echando espuma por los ojos, ebrio
de amor, huyendo sin saber adónde:
8 a donde el aire no apestase a muerto.

Tiendas de paz, brizados pabellones,


BLAS DE OTERO eran sus brazos, como llama al viento;
olas de sangre contra el pecho, enormes
Nació en Bilbao en 1916. Cursó la
12 olas de odio, ved, por todo el cuerpo.
carrera de Derecho, aunque nun-
ca ejerció. Se dedicó durante algún ¡Aquí! ¡Llegad! ¡Ay! Ángeles atroces
tiempo a la enseñanza, que aban-
en vuelo horizontal cruzan el cielo;
donó para entregarse por entero a
su obra cuando ya era un poeta de horribles peces de metal recorren
prestigio internacional. Visitó mu- 16 las espaldas del mar, de puerto a puerto.
chos países dando conferencias y
recitales y residió temporadas en Yo doy todos mis versos por un hombre
París, Rusia, China y Cuba. en paz. Aquí tenéis, en carne y hueso,
Su trayectoria es similar a la de mi última voluntad. Bilbao, a once
otros poetas que escriben después 20 de abril, cincuenta y uno.
de la traumática experiencia de la
Blas de Otero, Pido la paz y la palabra (1955).
Guerra Civil. Se inicia con una po-
esía existencial, a menudo religio-
sa, marcada por la angustia. A esta
etapa pertenecen sus primeros li-
bros: Ángel fieramente humano
(1950) y Redoble de conciencia EN EL PRINCIPIO
(1951). En un segundo momento,
inaugurado por su obra Pido la paz
1 Si he perdido la vida, el tiempo, todo
y la palabra (1955), el poeta se abre lo que tiré como un anillo al agua,
a un enfoque más social de la rea- si he perdido la voz en la maleza,
lidad; su poesía busca ser una for- 4 me queda la palabra.
ma de solidaridad y de lucha. A par-
tir de 1965, su poesía se renueva Si he sufrido la sed, el hambre, todo
formal y temáticamente, como la lo que era mío y resultó ser nada,
de otros poetas. Murió en Madrid,
si he segado las sombras en silencio,
en 1979.
8 me queda la palabra.

Si abrí los labios para ver el rostro


puro y terrible de mi patria,
si abrí los labios hasta desgarrármelos,
12 me queda la palabra.

Blas de Otero, Pido la paz y la palabra (1955).

Lengua castellana y literatura 1 78


COMENTARIOS DE TEXTO Blas de Otero
BAT

A LA INMENSA MAYORÍA 6 Relaciona la conclusión a la que llega el poeta


1 Observa la apertura y el cierre de la composi- con lo expresado en las estrofas anteriores.
ción y localiza en ellos las referencias, directas ¿Qué tipo de texto simula el final de la com-
o indirectas, al propio poeta. Justifica la dife- posición? Relaciónalo con el principio del
rencia de persona gramatical para referirse a poema.
sí mismo entre la primera y la última estrofa.
7 Comprueba que tanto el título como el con-
2 ¿Qué permite al poeta comprender? ¿Cuál es tenido del poema dan fe de una inflexión en
la consecuencia de esta súbita lucidez? la obra del poeta. Explica en qué consistió di-
cho cambio.

3 Fíjate en que las estrofas centrales relatan esa


crisis del poeta, y son de un gran dramatismo. EN EL PRINCIPIO
¿Qué provoca la huida que menciona en la se- 8 Observa cómo, en la primera estrofa, el poeta
gunda estrofa? opone aquello que ha perdido a lo que aún le
queda. Explica qué es en cada caso. ¿Qué apor-
ta la comparación como un anillo al agua a la
4 ¿Cómo caracteriza sus brazos en la tercera es- idea de pérdida? Comenta la función del en-
trofa? ¿Qué se estrella contra ellos? cabalgamiento entre los dos primeros versos.

5 ¿Qué expresan los gritos que abren la cuarta 9 Observa cómo, en la segunda estrofa, se des-
estrofa? ¿Cómo interpretas las dos imágenes taca el sufrimiento y reaparece la idea de pér-
que ocupan esta estrofa? dida. Localízala. ¿Qué connotan las imágenes
del tercer verso? Observa la aliteración.

10 Indica con qué acción se asocia normalmente


abrir los labios. ¿Con qué lo asocia el poeta?
Relaciónalo con el último verso, que se ha ido
repitiendo a lo largo del poema.

11 ¿Qué recursos estilísticos estructuran la com-


posición? Explícalo.

Lengua castellana y literatura 1 79


COMENTARIOS DE TEXTO Gabriel Celaya
BAT

LA POESÍA ES UN ARMA CARGADA DE FUTURO


1 Cuando ya nada se espera personalmente exaltante,
mas se palpita y se sigue más acá de la conciencia,
fieramente existiendo, ciegamente afirmando,
4 como un pulso que golpea las tinieblas,

cuando se miran de frente


los vertiginosos ojos claros de la muerte,
se dicen las verdades:
8 las bárbaras, terribles, amorosas crueldades.

Se dicen los poemas


que ensanchan los pulmones de cuantos, asfixiados,
GABRIEL CELAYA
piden ser, piden ritmo,
12 piden ley para aquello que sienten excesivo. […] Nació en Hernani (Guipúz-
Poesía para el pobre, poesía necesaria coa) en 1911. Vivió en la Re-
sidencia de Estudiantes, en
como el pan de cada día,
Madrid, donde conoció a al-
como el aire que exigimos trece veces por minuto,
gunos poetas de la Genera-
16 para ser y en tanto somos dar un sí que glorifica.
ción del 27. Publica su pri-
Porque vivimos a golpes, porque apenas si nos dejan mer libro, Marea de silencio,
decir que somos quien somos, en 1935. La Guerra Civil y
nuestros cantares no pueden ser sin pecado un adorno. sus consecuencias lo llevan
20 Estamos tocando el fondo. a un largo periodo sin pu-
Maldigo la poesía concebida como un lujo blicar, aunque no sin escri-
bir, que concluye en 1947
cultural por los neutrales
con Tranquilamente hablan-
que, lavándose las manos, se desentienden y evaden.
do, de temática existencial.
24 Maldigo la poesía de quien no toma partido hasta mancharse.
Sus obras siguientes, Las car-
Hago mías las faltas. Siento en mí a cuantos sufren tas boca arriba (1951), Cantos
y canto respirando. íberos (1955) y Episodios na-
Canto, y canto, y cantando más allá de mis penas cionales (1962), lo convierten
28 personales, me ensancho. en uno de los máximos re-
Quiero daros vida, provocar nuevos actos, presentantes de la poesía so-
cial. Posteriormente, como
y calculo por eso con técnica qué puedo.
otros poetas, intenta otros
Me siento un ingeniero del verso y un obrero
caminos.
32 que trabaja con otros a España en sus aceros.

Tal es mi poesía: poesía-herramienta 1 Lee esta composición:


a la vez que latido de lo unánime y ciego. es un verdadero manifiesto
Tal es, arma cargada de futuro expansivo de la poesía social.
36 con que te apunto al pecho.
¿Para quién dice escribir
No es una poesía gota a gota pensada. el poeta? ¿Por qué?
No es un bello producto. No es un fruto perfecto.
Es algo como el aire que todos respiramos
40 y es el canto que espacia cuanto dentro llevamos.
2 ¿Qué poesía rechaza y por
qué razones? Observa la
Son palabras que todos repetimos sintiendo
identificación del poeta
como nuestras, y vuelan. Son más que lo mentado.
con aquéllos a quienes la
Son lo más necesario: lo que tiene nombre.
dedica. ¿En qué imágenes
44 Son gritos en el cielo, y en la tierra, son actos.
se refleja? ¿Cómo
Gabriel Celaya (1954)
caracteriza su propia
poesía?

Lengua castellana y literatura 1 80


COMENTARIOS DE TEXTO José Ángel Valente
BAT

EL CRIMEN
1 Hoy he amanecido
como siempre, pero
con un cuchillo
en el pecho. Ignoro
5 quién ha sido,

y también los posibles móviles del delito.

Estoy aquí
tendido
y pesa vertical
JOSÉ ÁNGEL VALENTE
10 el frío.
Nació en Orense en 1929. Cursó es-
He sido asesinado. tudios de Derecho y Filología romá-
(Descarto la posibilidad del suicidio.) nica. Su primer libro, A modo de es-
La noticia se divulga peranza, aparece en 1954 y recibe el
premio Adonais. Punto cero (1972)
con relativo sigilo.
recoge su poesía hasta esta fecha.
15 El doctor estuvo brillante, pero Posteriormente ha seguido publican-
do y evolucionando en su quehacer
el interrogatorio ha sido
literario, hasta convertirse en una
confuso. El hecho de las figuras más importantes de su
carece de testigos. generación.
(Llamada la portera,
20 dijo

que el muerto no tenía antecedentes políticos.


1 Fíjate en que la primera frase del
poema adelanta sus características
Es una obsesión que la persigue
principales: narración fantástica e
desde la muerte del marido.) ironía. ¿Qué narra el poema?
¿Qué lo aproxima al género
Por mi parte no tengo
fantástico, de no suponer un
25 nada que declarar.
sentido figurado a la narración?
Se busca al asesino;
sin embargo,
2 Caracteriza la actitud del emisor
tal vez no hay asesino,
frente a su propio homicidio y
aunque se enrede así el final de la trama.
apóyalo en citas del texto.
30 Sencillamente yazgo

aquí, con un cuchillo…


3 Localiza y comenta los fragmentos
Oscila pendular y
en que el distanciamiento del
solemne, el frío.
emisor se tiñe especialmente de
No hay pruebas contra nadie. Nadie ironía.
35 ha consumado mi homicidio.

José Ángel Valente, A modo de esperanza (1955).


4 ¿Crees que hay que atribuir un
sentido alegórico a la narración
o que es un puro juego literario
en el que simplemente hay que
entrar? Argumenta tu respuesta.

Lengua castellana y literatura 1 81


COMENTARIOS DE TEXTO Carmen Laforet: Nada
BAT

NADA
1 El cuarto de mi tía comunicaba con el comedor y tenía un bal- 1 Lee el texto de Carmen
cón a la calle. Ella estaba de espaldas, sentada frente al peque- Laforet. Indica quién es
ño escritorio. Me paré, asombrada, a mirar la habitación por- el narrador y qué persona
que aparecía limpia y en orden como si fuera un mundo aparte gramatical utiliza.
5 en aquella casa. Había un armario de luna y un gran crucifijo ta-
piando otra puerta que comunicaba con el recibidor; al lado de
la cabecera de la cama, un teléfono. 2 ¿Quiénes son
La tía volvía la cabeza para mirar mi asombro con cierta com- los personajes?
placencia.
10 Estuvimos un rato calladas y yo inicié desde la puerta una
3 ¿Dónde se desarrolla la
sonrisa amistosa.
acción? Localiza la
—Ven, Andrea —me dijo ella—. Siéntate.
descripción del espacio
Observé que con la luz del día Angustias parecía haberse hin- y di qué función tiene.
chado, adquiriendo bultos y formas bajo su guardapolvo verde,
15 y me sonreí pensando que mi imaginación me jugaba malas pa-
sadas en las primeras impresiones. 4 ¿Cómo se reproduce el
—Hija mía, no sé cómo te han educado… diálogo? Localiza las
(Desde los primeros momentos, Angustias estaba empezando precisiones del narrador
a hablar como si se preparase para hacer un discurso.) Yo abrí la sobre la gestualidad de los
20 boca para contestarle, pero me interrumpió con un gesto de su personajes. ¿Qué indica
dedo. sobre éstos y su relación?
—Ya sé que has hecho parte de tu Bachillerato en un colegio
de monjas y que has permanecido allí durante casi toda la guerra.
Eso, para mí, es una garantía. Pero... esos dos años junto a tu pri- 5 ¿Qué averiguamos sobre
25 ma —la familia de tu padre ha sido siempre muy rara—, en el cada una de las
ambiente de un pueblo pequeño, ¿cómo habrán sido? No te ne- protagonistas a través
garé, Andrea, que he pasado la noche preocupada por ti, pen- del diálogo?
sando... Es muy difícil la tarea que se me ha venido a las manos.
La tarea de cuidar de ti, de moldearte en la obediencia... ¿Lo con-
30 seguiré? Creo que sí. De ti depende facilitármelo.

No me dejaba decir nada y yo tragaba sus palabras por sorpresa,


sin comprenderlas bien.
—La ciudad, hija mía, es un infierno. Y en toda España no
hay una ciudad que se parezca más al infierno que Barcelona...
35 Estoy preocupada con que anoche vinieras sola desde la esta-
ción. Te podía haber pasado algo. Aquí vive la gente aglomera-
da, en acecho unos contra otros. Toda prudencia en la conduc-
ta es poca, pues el diablo reviste tentadoras formas... Una joven
en Barcelona debe ser como una fortaleza. ¿Me entiendes?
40 —No, tía.
Angustias me miró.
—No eres muy inteligente, nenita.
Otra vez nos quedamos calladas.
—Te lo diré de otra forma: eres mi sobrina; por lo tanto, una
45 niña de buena familia, modosa, cristiana e inocente. Si yo no me
ocupara de ti para todo, tú en Barcelona encontrarías multitud
de peligros. Por lo tanto, quiero decirte que no te dejaré dar un
paso sin mi permiso. ¿Entiendes ahora?
Carmen Laforet

Lengua castellana y literatura 1 82


COMENTARIOS DE TEXTO Rafael Sánchez Ferlosio: El Jarama
BAT

EL JARAMA
1 Tito encendió el cigarrillo de Sebas y después el suyo; miraba a 1 Lee este fragmento de la
Lucita un momento en la luz de la llama. Sopló la cerilla y vol- novela “El Jarama” y
vía a sentarse junto a Luci. Paulina dijo: explica la situación en que
—¿Qué te pasa, Luci? se hallan los personajes.
5 —Nada, ¿por qué? ¿Por qué se encuentra mal
—No hablas. Lucita? ¿Cómo lo sabemos,
—Tengo una pizca de mareo. por el diálogo o por la
—Os ponéis a beber. ¿Por qué no te echas?, échate, anda. narración?
—Deja a la chica —dijo Sebas.
10 Valles abajo del Jarama, se veían las tierras difusas, como nie-
blas yacentes, a la luz imprecisa de la luna; más lejos, los perfi- 2 Observa en los fragmentos
les de lomas sucesivas, jorobas o espinazos nevados de blanco en que habla el narrador si
mortecino, contra el fondo de la noche, como un alejarse de gru- predomina la narración o
pas errabundas, gigantescos carneros de un rebaño fabuloso. Tito la descripción. ¿Se trata de
15 le puso a Lucita una mano en la nuca.
un observador externo o de
—¿Vas mejor? —le preguntaba por lo bajo.
un narrador omnisciente?
Ella sacó una voz cansada:
—Me defiendo.
Cambió de postura. Miraba allá abajo, por entremedias de los 3 ¿Cómo se resuelve la
20 troncos, en el agua embalsada de la presa, el reflejo de la luz que escena? ¿Cómo se insinúa
venía de las bombillas de los merenderos, la sombra enorme de la posibilidad de peligro?
alguien que se había asomado al malecón. El mismo malecón no
se veía, oculto a la derecha tras el morro del ribazo, ni las terra-
zas cuajadas de gente, ni las bombillas bailando en los cables 4 Compara la extrema
25 debajo del gran árbol; sólo las sombras y las luces que proyecta-
sencillez de los diálogos
ban hacia el agua. Llegaba el alboroto, las voces de juerga, la
con la riqueza léxica de
música incesante de las radios, el fragor de la esclusa, de allá
abajo, al final de los árboles, enfrente del puntal.
los fragmentos narrativos.
Luego el ojo blanquísimo del tren asomó de repente al fondo
¿En qué aspecto del paisaje
30 de los llanos; se acercaba, rodante y fragoroso dando alaridos por
se centra cada uno?
la recta elevada que cruzaba el erial. Entraba al puente del Localiza las comparaciones
Jarama, sorprendía instantáneas figuras de novios aplastadas de en el primero. Observa la
miedo contra los pretiles, en la luz violentísima, que se cegó acto selección de elementos en
seguido tras las casas de la margen derecha, hacia el paso a nivel el segundo y en el tercero
35 y la estación de Coslada y San Fernando de Henares. Lucita se y justifícala.
estremecía y se pasaba las manos por los brazos y los hombros;
luego dijo:
—Chico, estoy más molesta... Tengo grima, con tanto polvo
encima de la piel. Tanta tierra pegada por todo el cuerpo. Te
40 pones perdida de tierra, no se puede soportar.

—Lleva razón —dijo Sebas—, se llena uno hasta los pelos, a


fuerza de estarse revolcando todo el día. Para darse otro baño.
Yo me lo daba. ¿Eh?, ¿qué os parece?, ¿qué tal darnos ahora un
chapuzón?
45 —¿Pero a estas horas? —dijo Paulina—. Tú no estás bien de
la cabeza. Yo creo que...
—Más emocionante, ya verás.
—Por mí desde luego —dijo Lucita—. Yo me apunto. Has
tenido una idea.
50 —Bien por Lucita, así me gusta. Anda, Tito, y tú también,
vamos todos, hale.
Rafael Sánchez Ferlosio

Lengua castellana y literatura 1 83


COMENTARIOS DE TEXTO Luis Martín-Santos: Tiempo de silencio
BAT

TIEMPO DE SILENCIO
1 Cada una de las rejas, rastrillos y cerrojos que Pedro iba en-
contrando en su camino descendente, poseía un gnomo gris
que, a su paso, los hacía transitables, como si no estuvieran
fabricados de un apenas oxidado hierro sino de alguna mate-
5 ria fluida y deformable.

Que en el ínterin Similiano le hubiera pedido algunas recetas


gratuitas con destino a sus bien descritos padecimientos, que
otros sujetos amables y oportunos le hubieran entretenido con
diálogos referentes a su nombre, apellidos, estado civil, profe-
10 sión, domicilio, que la naturalidad más cotidiana presidiera
los gestos y actitudes de cuantos en aquellas oficinas se afa-
naban no habían sido datos suficientemente tranquilizadores
para que el desasosiego hubiera abandonado su pecho a la fa-
tiga, al sueño ni al hastío. Permanecía, pues, despierto, senta-
15 do en uno de aquellos sillones rotatorios, mientras que algu-
nos empleados se acercaban y le miraban como diciéndose:
“Es éste”, y se alejaban después tras recoger un papel eviden-
temente inútil o teclear al desgaire en una de las máquinas
profusamente repartidas por todo el ámbito de locales unidos
20 entre sí por el pasillo donde guardias, presos, oficinistas y al-
gún que otro perdido camarero con su chaquetilla blanca tran-
sitaban. La pequeña porquería que imperceptiblemente iba ca-
yendo se depositaba sobre cuantos objetos eran asequibles a
los dedos dándoles tacto rasposo y aspecto amarillento. Tal vez
25 esta sensación no fuera debida, a decir verdad, al polvo cuya
realidad es siempre cuestionable (como la de los otros entes
invisibles), sino el miedo que parecía reinar con dominio ab-
soluto en tales zonas, habitadas además de por los regidores y
manufactureros de la angustia, por ciertos sutiles seres de co-
30 lor verdoso y barba crecida, nacidos de una raza todavía no
antropológicamente clasificada, en cuyos rostros, al ser con-
templados atentamente, resplandecía aquel reino absoluto, an-
te los que Pedro podía inclinarse como ante un espejo que
mostrara la naturaleza de la metamorfosis por él mismo sufri-
35 da, de la que aún no tenía total conocimiento. Así pues, lo que
él notaba como pequeña sensación de cansancio en ambas cor-
vas, tensión de la bolsa del párpado inferior, picores prolongados
a lo largo de ambas hendiduras palpebrales, ausencia absolu-
ta de hambre sobre superficie seca de lengua vuelta objeto ex-
40 traño en cavidad bucal repentinamente contraída, incapaci-
dad para comprensión de preguntas sencillas, fuerte deseo de
ser amable con todo el mundo, suciedad pegajosa en axilas y
en pies no por falta de jabón sino por sudor nuevo nunca an-
tes eliminado, mirar agitado y vertiginoso hacia todos (abso-
45 lutamente todos) los rostros de los empleados intentando es-
crutar en ellos los signos de una lejana simpatía que, por lo
demás, indiferentes prodigaban, proximidad excesiva de los
zapatos a los pies que han perdido aparentemente toda utili-
dad traslatoria ya que no se es movido a impulsos de una vo-
50 luntad que se transmite a los músculos de las piernas sino por
una fuerza magnética que emana de los hábiles ordenadores de

Lengua castellana y literatura 1 84


COMENTARIOS DE TEXTO Luis Martín-Santos: Tiempo de silencio
BAT

la circulación en tales pistas, proximidad excesiva del cuello 1 ¿Dónde se sitúa el


de la camisa al de la carne que ha perdido también sus natu- episodio? Señala algunos
rales propiedades transportadoras de aire, alimentos, etc., con- elementos que te han
55 servando sólo la de servir de pivote al movimiento circular pre- permitido identificarlo.
ciso para captar con la mayor frecuencia posible las muestras
de simpatía de los rostros circundantes, temblor o bien rigidez
a lo largo de las vértebras lumbares, no eran sino los indicios 2 Observa la extensión de la
internos de ese mismo terror que deformaba los rostros de los frase que empieza “Así
60 que él podía ver, hijos de esa raza despreciable en la que todo pues, lo que él notaba …”.
hombre puede ser trasmutado por la culpa públicamente des- ¿Qué se describe en ella?
cubierta, hecha patente y en ruta hacia el castigo. ¿Cómo se realiza la
—Vamos a acabar en seguida. Usted es un hombre inteli- descripción?
gente –dijo uno de los omnipotentes habitantes de las ofi-
65 cinas que precisamente mostraba hacia él una simpatía más
desbordante, una sonrisa especialmente acogedora, una mag- 3 ¿De qué crees que trata
nanimidad más fina y providente. el diálogo que mantienen
Pedro se volvió hacia él interrumpiendo la búsqueda de otras Pedro y su interlocutor?
fuentes de simpatía ya que ésta, al parecer más decisiva, con tan ¿Qué tiene de novedoso?
70 especial abundancia sobre él se derramaba.

—Así que usted... (suposición capciosa y sorprendente). 4 Localiza elementos de


—No. Yo no... (refutación indignada y sorprendida). la realidad descrita cuya
—Pero no querrá usted hacerme creer que... (hipótesis in- presentación haya
verosímil y hasta absurda). dificultado su identificación
75 —No, pero yo... (reconocimiento consternado). (técnicas de extrañamiento).
—Usted sabe perfectamente... (lógica, lógica, lógica).
—Yo no he... (simple negativa a todas luces insuficiente).
—Tiene que reconocer usted que... (lógica).
—Pero... (adversativa apenas si viable).
80 —Quiero que usted comprenda... (cálidamente humano).
—No.
—De todos modos es inútil que usted... (afirmación de su-
perioridad basada en la experiencia personal de muchos ca-
sos).
85 —Pero... (apenas adversativa con escasa convicción).
—Claro que si usted se empeña... (posibilidad de recurren-
cia a otras vías abandonando el camino de la inteligencia y la
amistosa comprensión).
—No, nada de eso... (negativa alarmada).
90 —Así que estamos de acuerdo... (superación del apenas apa-
rente obstáculo).
—Bueno... (primer peligroso comienzo de reconocimien-
to).
—Perfectamente. Entonces usted… (triunfal).
—¿Yo?... (horror ante las deducciones imprevistas).
95 —¡¡ Ya me estoy cansando!!
Luis Martín-Santos

Lengua castellana y literatura 1 85


COMENTARIOS DE TEXTO Julio Cortázar
BAT

NO SE CULPE A NADIE
Los cuentos fantásticos de Julio Cortázar parten de una realidad coti-
diana familiar al lector. Cuando ésta se resquebraja para dar entrada al
elemento fantástico, éste resulta especialmente inquietante. El elemen-
to fantástico parece proceder del mundo de la pesadilla, o incluso del
trastorno mental.

1 El frío complica siempre las cosas, en verano se está tan cerca del
mundo, tan piel contra piel, pero ahora a las seis y media su mujer lo
espera en una tienda para elegir un regalo de casamiento, ya es tarde
y se da cuenta de que hace fresco, hay que ponerse el pulóver azul,
5 cualquier cosa que vaya bien con el traje gris, el otoño es un ponerse
y sacarse pulóveres, irse encerrando, alejando. Sin ganas silba un tango
mientras se aparta de la ventana abierta, busca el pulóver en el armario
y empieza a ponérselo delante del espejo. No es fácil, a lo mejor por
culpa de la camisa que se adhiere a la lana del pulóver, pero le cuesta
10 hacer pasar el brazo, poco a poco va avanzando la mano hasta que al
fin asoma un dedo fuera del puño de lana azul, pero a la luz del atar-
decer el dedo tiene un aire como de arrugado y metido para adentro,
con una uña negra y terminada en punta. De un tirón se arranca la
manga del pulóver y se mira la mano como si no fuese suya, pero ahora
15 que está fuera del pulóver se ve que es su mano de siempre y él la deja
caer al extremo del brazo flojo y se le ocurre que lo mejor será meter
el otro brazo en la otra manga a ver si así resulta más sencillo. Parece-
ría que no lo es porque apenas la lana del pulóver se ha pegado otra
vez a la tela de la camisa, la falta de costumbre de empezar por la otra
20 manga dificulta todavía más la operación, y aunque se ha puesto a sil-
bar de nuevo para distraerse siente que la mano avanza apenas y que
sin alguna maniobra complementaria no conseguirá hacerla llegar
nunca a la salida. Mejor todo al mismo tiempo, agachar la cabeza para
calzarla a la altura del cuello del pulóver a la vez que mete el brazo libre
25 en la otra manga enderezándola y tirando simultáneamente con los
dos brazos y el cuello. En la repentina penumbra azul que lo envuelve
parece absurdo seguir silbando, empieza a sentir como un calor en la
cara aunque parte de la cabeza ya debería estar afuera, pero la frente y
toda la cara siguen cubiertas y las manos andan apenas por la mitad
30 de las mangas, por más que tira nada sale afuera y ahora se le ocurre
pensar que a lo mejor se ha equivocado en esa especie de cólera iróni-
ca con que reanudó la tarea, y que ha hecho la tontería de meter la
cabeza en una de las mangas y una mano en el cuello del pulóver. Si
fuese así su mano tendría que salir fácilmente pero aunque tira con
35 todas sus fuerzas no logra hacer avanzar ninguna de las dos manos
aunque en cambio parecería que la cabeza está a punto de abrirse paso
porque la lana azul le aprieta ahora con una fuerza casi irritante la nariz
y la boca, lo sofoca más de lo que hubiera podido imaginarse, obli-
gándolo a respirar profundamente mientras la lana se va humede-
40 ciendo contra la boca, probablemente desteñirá y le manchará la cara
de azul. Por suerte en ese mismo momento su mano derecha asoma al
aire, al frío de afuera, por lo menos ya hay una afuera aunque la otra
siga apresada en la manga, quizá era cierto que su mano derecha esta-

Lengua castellana y literatura 1 86


COMENTARIOS DE TEXTO Julio Cortázar
BAT

ba metida en el cuello del pulóver, por eso lo que él creía el cuello le


45 está apretando de esa manera la cara sofocándolo cada vez más, y en
cambio la mano ha podido salir fácilmente. De todos modos y para
estar seguro lo único que puede hacer es seguir abriéndose paso, res-
pirando a fondo y dejando escapar el aire poco a poco, aunque sea
absurdo porque nada le impide respirar perfectamente salvo que el aire
50 que traga está mezclado con pelusas de lana del cuello o de la manga
del pulóver, y además hay el gusto del pulóver, ese gusto azul de la lana
que le debe estar manchando la cara ahora que la humedad del alien-
to se mezcla cada vez más con la lana, y aunque no puede verlo por-
que si abre los ojos las pestañas tropiezan dolorosamente con la lana,
55 está seguro de que el azul le va envolviendo la boca mojada, los agu-
jeros de la nariz, le gana las mejillas, y todo eso lo va llenando de ansie-
dad y quisiera terminar de ponerse de una vez el pulóver sin contar que
debe ser tarde y su mujer estará impacientándose en la puerta de la
tienda. Se dice que lo más sensato es concentrar la atención en su
60 mano derecha, porque esa mano por fuera del pulóver está en contac-
to con el aire frío de la habitación, es como un anuncio de que ya falta
poco y además puede ayudarlo, ir subiendo por la espalda hasta afe-
rrar el borde inferior del pulóver con ese movimiento clásico que ayuda
a ponerse cualquier pulóver tirando enérgicamente hacia abajo. Lo
65 malo es que aunque la mano palpa la espalda buscando el borde de
lana, parecería que el pulóver ha quedado completamente arrollado
cerca del cuello y lo único que encuentra la mano es la camisa cada vez
más arrugada y hasta salida en parte del pantalón, y de poco sirve
traer la mano y querer tirar de la delantera del pulóver porque sobre
70 el pecho no se siente más que la camisa, el pulóver debe haber pasa-
do apenas por los hombros y estará ahí arrollado y tenso como si
él tuviera los hombros demasiado anchos para ese pulóver, lo que
en definitiva prueba que realmente se ha equivocado y ha meti-
do una mano en el cuello y la otra en la manga, con lo cual la
75 distancia que va del cuello a una de las mangas es exactamen-
te la mitad de la que va de una manga a otra, y eso explica
que él tenga la cabeza un poco ladeada a la izquierda, del
lado donde la mano sigue prisionera en la manga, si es la
manga, y que en cambio su mano derecha que ya está
80 afuera se mueva con toda libertad en el aire aunque no
consiga hacer bajar el pulóver que sigue como arrolla-
do en lo alto de su cuerpo. Irónicamente se le ocurre
que si hubiera una silla cerca podría descansar y respi-
rar mejor hasta ponerse del todo el pulóver, pero ha
85 perdido la orientación después de haber girado tantas
veces con esa especie de gimnasia eufórica que inicia
siempre la colocación de una prenda de ropa y que
tiene algo de paso de baile disimulado, que nadie puede
reprochar porque responde a una finalidad utilitaria y no
90 a culpables tendencias coreográficas. En el fondo la ver-
dadera solución sería sacarse el pulóver puesto que no ha
podido ponérselo, y comprobar la entrada correcta de cada

Lengua castellana y literatura 1 87


COMENTARIOS DE TEXTO Julio Cortázar
BAT

mano en las mangas y de la cabeza en el cuello, pero la mano derecha


desordenadamente sigue yendo y viniendo como si ya fuera ridículo
95 renunciar a esa altura de las cosas, y en algún momento hasta obede-
ce y sube a la altura de la cabeza y tira hacia arriba sin que él com-
prenda a tiempo que el pulóver se le ha pegado en la cara con esa
gomosidad húmeda del aliento mezclado con el azul de la lana, y cuan-
do la mano tira hacia arriba es un dolor como si le desgarraran las ore-
100 jas y quisieran arrancarle las pestañas. Entonces más despacio, enton-
ces hay que utilizar la mano metida en la manga izquierda, si es la
manga y no el cuello, y para eso con la mano derecha ayudar a la mano
izquierda para que pueda avanzar por la manga o retroceder y zafarse,
aunque es casi imposible coordinar los movimientos de las dos manos,
105 como si la mano izquierda fuese una rata metida en una jaula y desde
afuera otra rata quisiera ayudarla a escaparse, a menos que en vez de
ayudarla la está mordiendo porque de golpe le duele la mano prisio-
nera y a la vez la otra mano se hinca con todas sus fuerzas en eso que
debe ser su mano y que le duele, le duele a tal punto que renuncia a
110 quitarse el pulóver, prefiere intentar un último esfuerzo para sacar la

cabeza fuera del cuello y la rata izquierda fuera de la jaula y lo inten-


ta luchando con todo el cuerpo, echándose hacia adelante y hacia
atrás, girando en medio de la habitación, si es que está en el medio por-
que ahora alcanza a pensar que la ventana ha quedado abierta y que
115 es peligroso seguir girando a ciegas, prefiere detenerse aunque su mano
derecha siga yendo y viniendo sin ocuparse del pulóver, aunque su
mano izquierda le duela cada vez más como si tuviera los dedos mor-
didos o quemados, y sin embargo esa mano le obedece, contrayendo
poco a poco los dedos lacerados alcanza a aferrar a través de la manga
120 el borde del pulóver arrollado en el hombro, tira hacia abajo casi sin
fuerza, le duele demasiado y haría falta que la mano derecha ayudara
en vez de trepar o bajar inútilmente por las piernas, en vez de pelliz-
carle el muslo como lo está haciendo, arañándolo y pellizcándolo a tra-
vés de la ropa sin que pueda impedírselo porque toda su voluntad
125 acaba en la mano izquierda, quizá ha caído de rodillas y se siente como
colgado de la mano izquierda que tira una vez más del pulóver y de
golpe es el frío en las cejas y en la frente, en los ojos, absurdamente no
quiere abrir los ojos pero sabe que ha salido fuera, esa materia fría, esa
delicia es el aire libre, y no quiere abrir los ojos y espera un segundo,
130 dos segundos, se deja vivir en un tiempo frío y diferente, el tiempo de

fuera del pulóver, está de rodillas y es hermoso estar así hasta que poco
a poco agradecidamente entreabre los ojos libres de la baba azul de la
lana de adentro, entreabre los ojos y ve las cinco uñas negras suspen-
didas apuntando a sus ojos, vibrando en el aire antes de saltar contra
135 sus ojos, y tiene el tiempo de bajar los párpados y echarse atrás cubrién-

dose con la mano izquierda que es su mano, que es todo lo que le


queda para que lo defienda desde dentro de la manga, para que tire
hacia arriba el cuello del pulóver y la baba azul le envuelva otra vez la
cara mientras se endereza para huir a otra parte, para llegar por fin a
140 alguna parte sin mano y sin pulóver, donde solamente haya un aire fra-
goroso que lo envuelva y lo acompañe y lo acaricie y doce pisos.

Julio Cortázar, Final de juego.

Lengua castellana y literatura 1 88


COMENTARIOS DE TEXTO Julio Cortázar
BAT

1 Describe la situación inicial de este relato de 6 ¿Qué sucede cuando finalmente logra sacar
Julio Cortázar. la cabeza fuera? ¿Por qué se vuelve a meter
dentro del pulóver?

2 ¿Qué dificultad tiene el protagonista? ¿Se tra-


ta de algo raro o corriente? 7 Explica el desenlace de la historia. ¿Se narra di-
rectamente o se debe deducir?

3 ¿En qué momento sucede algo que ya no es


normal? ¿Qué hace el personaje? 8 Observa que la narración, aunque está en ter-
cera persona, parece surgir por momentos di-
rectamente de la mente del personaje. ¿Qué
4 Describe los problemas del personaje, que tie- nombre recibe este recurso técnico?
ne la mano derecha fuera, y la izquierda y la
cabeza dentro del pulóver. ¿Cómo se com-
porta cada una de sus manos? ¿Con qué las 9 ¿Qué dificultad crea la extensión anormal-
compara? mente larga de las frases? ¿Crea alguna sen-
sación similar a las del protagonista?

5 ¿Qué temor le asalta, girando a ciegas en me-


dio de la habitación?

Lengua castellana y literatura 1 89

S-ar putea să vă placă și